Chapter 8: Nitrogen Metabolism

¡Supera tus tareas y exámenes ahora con Quizwiz!

*The answer is E.* Oculocutaneous albinism (OCA) is a genetically heterogeneous congenital disorder characterized by reduced or absent melanin synthesis in the melanocytes. Individuals with OCA1 have a recessive mutation in tyrosinase, and thus have absent or impaired, depending on mutation subtypes, ability to convert tyrosine to melanin in melanocytes. Tyrosinase is specifically responsible for converting tyrosine into dopaquinone, a preliminary product used in the formation of melanin. This is a key, rate-limiting step in the ultimate production of melanin from phenylalanine. Similarly, patients with phenylketonuria (PKU) and other disorders that reduce tyrosine will also have a fair complexion due to decreased available substrate for the production of melanin. Individuals with OCA often present with snow-white hair, white or pink skin color and abnormal irides. Squamous cell carcinomas are almost inevitable without appropriate photoprotection; basal cell carcinomas and melanomas are common as well. Ocular problems are severe and f requently include impaired visual acuity, nystagmus, photophobia, and strabismus.

A 1-week-old term African-American boy presents to his pediatrician for a well-child visit. The parents report that the infant is breastfeeding well and is producing the expected amount of wet diapers and stools per day. Pregnancy and birth were unremarkable and he has passed all newborn screening tests to date. However, his parents remark that his skin and hair are very light in color. On physical exam, you appreciate an infant with snow-white hair and pale skin. His irides are grayish-tan. Exam is otherwise unremarkable. Which enzyme defect best accounts for this disorder? A. α-Keto acid dehydrogenase B. Galactokinase C. Homogentisic acid oxygenase D. Methionine synthase E. Tyrosinase

*The answer is B.* The patient has Lesch-Nyhan syndrome, a deficiency in HGPRT activity. HGPRT utilizes as substrates hypoxanthine, guanine, and PRPP, converting the free base to a nucleoside monophosphate (IMP and GMP). The enzyme does not utilize adenine, adenosine, guanosine, or GMP as a substrate. The reason for the aberrant behavior and developmental delay observed in this disorder has not yet been elucidated.

A 1-year-old boy was brought to the pediatrician due to a developmental delay, biting of his lips and fingers, and the presence of orange crystals in his diapers. Enzymatic analysis shows loss of 99% of the activity of a particular enzyme. The defective enzyme in this disorder would normally utilize which of the following as a substrate? (A) Adenine (B) Guanine (C) Adenosine (D) Guanosine (E) GMP

*The answer is C.* The disease is maple-syrup urine disease, which is also known as branched-chain ketoaciduria. This is a rare genetic disease with an incidence of 1 in 200,000 in most of the United States, although a Mennonite community in Lancaster County Pennsylvania has been found with an incidence of 1 in 176. The defective enzyme branched-chain ketoacid dehydrogenase. normally is involved in the catabolism of branched-chain amino acids that have been previously transaminated to form their ketoacid derivatives. Leucine. isoleucine, and valine should be minimized in the diet.

A 5-day-old newborn develops poor feeding and sleepiness that progresses to convulsions, coma. and apnea. Urinalysis shows ketones but no acetoacetate or 3-hydroxybutyrate. Specialized laboratory studies show a deficiency of branched-chain ketoacid dehydrogenase. Which of the following amino acids should be eliminated from this baby's diet? A. Histidine B. Isoleucine C. Methionine D. Phenylalanine D. Tyrosine

*The answer is B.* A folate deficiency leads to a megaloblastic anemia since DNA synthesis is inhibited in the absence of folate. Large cells are generated in the bone marrow because the cell enlarges in preparation for division, but the DNA does not replicate and the cell does not divide. All of the other answer choices suggested create a microcytic, hypochromic anemia. Lead poisoning interferes with heme synthesis, as does an iron deficiency. Under these conditions, the red cells released are small in size, as the final size is, in part, dependent on the intracellular concentration of heme (so if heme levels are low, the cell will be small). Sideroblastic anemia also results from a disruption in heme synthesis, for a variety of causes. Hereditary spherocytosis is due to mutations in red cell membrane proteins, which lead to early removal of these cells from the spleen.

A patient presents with fatigue, and a blood count reveals a macrocytic, hyperchromic anemia. Which one of the following may account for this type of anemia? (A) Lead poisoning (B) Folate deficiency (C) Hereditary spherocytosis (D) Sideroblastic anemia (E) Iron deficiency

*The answer is E.* These transaminases convert amino acids to their corresponding α-keto acids in reactions that are readily reversible. α Ketoglutarate and glutamate serve as the other α-keto acid/amino acid pair. Pyruvate (the end product of glycolysis) is the α-keto acid corresponding to alanine, and oxaloacetate (an intermediate of the TCA cycle) is the partner of aspartate. PLP is the cofactor. Thus, AST will convert aspartate and α-ketoglutarate to oxaloacetate and glutamate, and ALT will convert alanine and α-ketoglutarate to pyruvate and glutamate.

In liver disease, the enzymes AST and ALT leak into the blood from damaged liver cells. Both of these enzymes have which one of the following in common? (A) They both transfer ammonia to α-keto acids to form amino acids. (B) They both form intermediates of glycolysis from amino acids. (C) They both require thiamine pyrophosphate as a cofactor. (D) They both catalyze irreversible reactions. (E) They both convert α-ketoglutarate to glutamate.

*The answer is D.* IMP is a feedback inhibitor of PRPP amidophosphoribosyl transferase, the first reaction in the biosynthesis of purines. IMP is formed by the HPRT reaction in the salvage of hypoxanthine.

Patients with Lesch-Nyhan syndrome have hyperuricemia, indicating an increased biosynthesis of purine nucleotides, and markedly decreased levels of hypoxanthine phosphoribosyl transferase (HPRT). The hyperuricemia can be explained on the basis of a decrease in which regulator of purine biosynthesis? A. ATP B. GDP C. Glutamine D. IMP E. PRPP

*The answer is C.* Serine donates a carbon to THF to form N5,N10-methylene THF. This is oxidized to form N5, N10-methenyl THF, which is then hydrolyzed with water to form N10-formyl THF. As such, glycine, FAD, B12, and B6 are not required for these conversions to take place.

The primary route of carbon entry into the tetrahydrofolate (THF) pool is via the serine hydroxymethyltransferase reaction. Which of the following is required to convert that initial form of the THF into the form that can donate carbons to de novo purine synthesis? (A) Glycine (B) FAD (C) Water (D) B12 (E) B6

*The answer is A.* Glutamate cannot produce histidine, as histidine is an essential amino acid in humans. Glutamate can fix ammonia to form glutamine in a reaction catalyzed by glutamine synthetase. Glutamate can be synthesized from α-ketoglutarate either through a transamination reaction or by glutamate dehydrogenase (which fixes ammonia into α ketoglutarate). Glutamine donates nitrogens for purine ring synthesis, but glutamate does not. Glutamine is a nitrogen carrier in the blood, whereas glutamate is not.

The properties of glutamate are best represented by which one of the following?

*The answer is A.* Acetylcholine mediates the release of epinephrine from the chromaffin granules of the adrenal medulla into the bloodstream in response to neural stimulation.

The release of epinephrine from the chromaffin granules of the adrenal medulla into the bloodstream in response to neural stimulation is mediated by which of the following? (A) Acetylcholine (B) γ-Aminobutyric acid (GABA) (C) Cortisol (D) Dopamine (E) Serotonin

*The answer is A.* Arginine is a basic amino acid that may be synthesized by the body. However, during periods of growth, such as childhood, biosynthesis levels are not high enough to allow adequate protein synthesis and the amino acid becomes a necessary part of dietary intake.

A 5-year-old child is brought to the clinic for his check-up. On physical exam no apparent abnormalities are found. Before leaving, the mother states that she has been having difficulty at work, but has been attempting to make sure her son is being adequately fed. She specifically asks about protein nutrition, as she has heard that all amino acids are not present in foods that are labeled as being protein-rich. Which amino acid should she be sure to include in her son's diet? A. Arginine B. Aspartate C. Glutamate D. Serine

*The answer is A.* Adults with alcaptonuria show a high prevalence of ochronotic arthritis due to deficiency of homogentisate oxidase.

A 56-year-old man with a history of genetic disease undergoes hip replacement surgery for arthritis. During the operation the surgeon notes a dark pigmentation (ochronosis) in the cartilage. His ochronotic arthritis is most likely caused by oxidation and polymerization of excess tissue A. homogentisic acid B. orotic acid C. methylmalonic acid D. uric acid E. ascorbic acid

*The answer is E.* Ornithine transcarbamylase (OTC) deficiency, the most common of the urea cycle disorders, is a rare metabolic disorder that occurs in 1 out of every 80,000 births. OTC deficiency is an X-linked genetic disorder affecting predominantly males although there is significant, though milder, symptomatology in heterozygous females. Like other urea cycle disorders, OTC deficiency affects the body's ability to eliminate ammonia, a toxic byproduct of the body's use of protein. As a result, ammonia accumulates in the blood causing hyperammonemia. Ammonia travels to the various organs of the body, including the brain, where it can cause coma, brain damage, and death. OTC deficiency often becomes evident in the first few days of life. An infant with OTC deficiency may be lacking in energy (lethargic) or unwilling to eat, and may have a poorly controlled breathing rate or body temperature. Some babies with this disorder may experience seizures or unusual body movements or may go into a coma. The main criterion for the diagnosis of urea cycle defects is hyperammonemia. Patients with a deficiency of OTC or carbamoyl phosphate synthetase (CPS) (choice D) have no abnormalities of plasma amino acids, except for increased levels of glutamine secondary to the hyperammonemia. Also in OTC deficiency, there is a marked increase in urinary excretion of erotic acid due to excess carbamoyl phosphate, the substrate of OTC, stimulating pyrimidine synthesis. Orotic acid is an early intermediate in the pyrimidine pathway. Orotic acid in urine is low in CPS deficiency Although the patient in this question is a newborn, symptoms may not appear until later in childhood or even in an adult.

A newborn born at 37 weeks' gestation is admitted to the neonatal intensive care unit because of a spontaneous pneumothorax. Two days later, the pneumothorax resolves and the newborn is transferred to the nursery on bottle feedings of a standard cow's-milk formula, within 24 hours, the baby vomits all feeds, is tachypneic, and has progressive lethargy. Laboratory studies show: Arterial pH 7.41 HCO3- 25 Plasma NH3 300 micrometers Plasma amino acids Elevated glutamine Citrulline Absent Urine orotic acid High A. Arginase deficiency B. Argininosuccinic acid lyase deficiency C. Arginosuccinic acid synthetase deficiency D. Carbamoyl phsophate synthetase deficiency E. Ornithine transcarbamylase

*The answer is B.* The child 's presentation is characteristic of phenylketonuria (PKU), an autosomal recessive disorder caused by a mutation in the gene coding for the enzyme phenylalanine hydroxylase Phenylalanine hydroxylase converts phenylalanine to tyrosine using the cofactor tetrahydrobiopterin. The deficiency of the enzyme leads to increased phenylalanine levels in the serum, which cause progressive intellectual disability. In the United States PKU is normally screened for at birth, but this child did not benefit from perinatal screening. PKU can be effectively treated by decreasing the dietary intake of phenylalanine.

An 11-month-old girl is brought to her pediatrician because of severe developmental delay. The mother reports that she is a recent immigrant and did not have access to perinatal screening tests. The pediatrician orders a genetic test, which reveals that the child has two mutated copies of an enzyme responsible for the hydroxylation of an amino acid to form another amino acid. Had this mutation been discovered at birth, what would have most effectively prevented the development of intellectual disability in this child? A. A diet supplemented with a cofactor for the deficient enzyme B. A diet with reduced amounts of the substrate amino acid for the deficient enzyme C. Nothing could have prevented the development of intellectual disability D. Recombinant enzyme delivered by injection

*The answer is A.* The child has a mutation in the shared E3 subunit of pyruvate dehydrogenase, α-ketoglutarate dehydrogenase, and the branched-chain α-ketoacid dehydrogenase. All three reactions are oxidative decarboxylation reactions and utilize a three-component enzyme complex, designated as E1, E2, and E3. The E1 subunit binds thiamine pyrophosphate and catalyzes the decarboxylation reaction. The E2 subunit is a transacylase and is involved in the oxidation-reduction part of the reaction. The E3 component (dihydrolipoyl dehydrogenase) is shared among all three enzymes, and a mutation in this subunit will affect the activity of all three enzymes. This subunit reduces NAD+, using electrons obtained from reduced lipoic acid. The key to solving the problem is the recognition that lactic acidosis occurs, which would happen when pyruvate dehydrogenase was defective. None of the other dehydrogenases listed (isocitrate dehydrogenase, malate dehydrogenase, and succinate dehydrogenase) require the E3 subunit for their activity, nor do they catalyze oxidative decarboxylation reactions. Acetyl-CoA carboxylase catalyzes a carboxylation reaction, and does not share subunits with the enzymes that catalyze oxidative decarboxylations.

Another routine newborn screening identified a child with elevated levels of the branched-chain amino acids and their α-ketoacid derivatives. In addition, the child also exhibited lactic acidosis. Which enzyme listed below would you expect to be negatively affected (reduced activity) by this disorder? (A) α-ketoglutarate dehydrogenase (B) Isocitrate dehydrogenase (C) Malate dehydrogenase (D) Succinate dehydrogenase (E) Acetyl-CoA carboxylase

*The answer is E.* Homocystinuria is an autosomal recessive condition caused by deficiencies of various enzymes involved in the pathway that converts methionine to cysteine. This results in the accumulation of ho mocysteine, which is then excreted in urine. In this condition, cysteine becomes an essential amino acid. Clinically, homocystinuria is manifested by mental retardation, osteoporosis, tall stature, kyphosis, lens subluxation, and atherosclerosis (causing premature stroke and myocardial infarction). Diagnosis is based on a positive nitroprusside cyanide test. Marfan syndrome is the primary differential diagnosis. Clinical features of homocystinuria, such as ectopia lentis, tall and thin body habitus, and chest and spinal deformities, are similar to the features found in patients with Marfan syndrome. However, generalized osteoporosis, arterial and venous thrombosis, premature atherosclerosis, changes in hair, and the disorders of mental development are absent in patients with Marfan syndrome.

At a routine check-up, a 7-year-old boy is found to have osteoporosis. The patient is tall and thin with pale skin, fair hair, and flushed cheeks. He has arachnodactyly, pes cavus, and bilaterally dislocated lenses, and demonstrates developmental delay with mild mental retardation. His mother is told that her child might benefit from folic acid supplementation. Which is the most appropriate test to confirm the diagnosis? (A) Enzymatic assay for the enzyme HGPRT (B) Genetic studies demonstrating a mutation in type I collagen (C) Genetic studies indicating >200 copies of the CGG trinucleotide repeat on the X chromosome (D) Nitroblue tetrazolium test (E) Nitroprusside cyanide test

1. *The answer is C.* Maple syrup urine disease; substrates are branched chain α-ketoacids derived from the branched chain amino acids. 2. *The answer is E.* The child has PKU; aspartame contains phenylalanine. These children may be blond, blue-eyed, and pale complected because of deficient melanin production from tyrosine. 3. *The answer is F.* The only biotin-dependent reaction in the diagram. The enzyme is propionyl-CoA carboxylase.

For each of the conditions below, link the missing substrate or enzyme. 1. A 9-week-old boy, healthy at birth, begins to develop symptoms of ketoacidosis, vomiting, lethargy, seizures and hypertonia. Urine has characteristic odor of maple syrup. 2. A child with white-blond hair, blue eyes, and pale complexion is on a special diet in which one of the essential amino acids is severely restricted. He has been told to avoid foods artificially sweetened with aspartame. 3. A chronically ill patient on long-term (home) parenteral nutrition develops metabolic acidosis, a grayish pallor, scaly dermatitis, and alopecia (hair loss). These symptoms subside upon addition of the B vitamin biotin to the alimentation fluid.

*The answer is B.* Deficiency of dihydrobiopterin reductase, the enzyme responsible for the reduction of dihydrobiopterin (BH2) to BH4, is the most common cause for a deficiency of BH4. This results in what is known as atypical or malignant phenylketonuria.

During normal screening for phenylketonuria, a male newborn has a serum phenylalanine concentration of 35 mg/dL (greater than 20 mg/dL is considered a positive test). Signs of tyrosine deficiency also are apparent. Enzymatic analysis using cultured fibroblasts, obtained after circumcision, shows normal activity of phenylalanine hydroxylase. A possible explanation for these findings is a deficiency in function of which of the following coenzymes? (A) Adenosylcobalamin (B) Biopterin (C) Dihydroquinone (D) Pyridoxal phosphate (E) Tetrahydrofolic acid

*The answer is A.* Most forms of gout are probably X-linked recessive with deficiencies in phosphoribosyl pyrophosphate (PRPP) synthase, the first step of purine synthesis. Some patients may have a partial deficiency of hypoxanthine-guanine phosphoribosyl transferase (HGPRTase), which salvages hypoxanthine and guanine by transferring the purine ribonucleotide of PRPP to the bases and forming iosinate and guanylate, respectively. In all of these patients, the hypoxanthine analogue allopurinol has two actions: (1) it inhibits xanthine oxidase, which catalyzes the oxidation of hypoxanthine to xanthine and then to uric acid stones and tissue deposits; and (2) it forms an inactive allopurinol ribonucleotide from PRPP in a reaction catalyzed by HGPRTase, thereby decreasing the rate of purine synthesis. In contrast, because of the total loss of HGPRTase activity in Lesch-Nyhan patients, the allopurinol ribonucleotide cannot be formed. Thus, PRPP levels are not decreased and de novo purine synthesis continues unabated. The gouty arthritis caused by urate crystal formation is relieved in Lesch-Nyhan patients, but their neurological symptoms are not.

In most patients with gout as well as those with Lesch-Nyhan syndrome, purines are overproduced and overexcreted. Yet the hypoxanthine analogue allopurinol, which effectively treats gout, has no effect on the severe neurological symptoms of Lesch-Nyhan patients because it does not a. Decrease de novo purine synthesis b. Decrease de novo pyrimidine synthesis c. Diminish urate synthesis d. Increase phosphoribosylpyrophosphate (PRPP) levels e. Inhibit xanthine oxidase

*The answer is E.* Glycine reacts with succinyl-CoA in the first step of heme synthesis and with arginine in the first step of creatine synthesis. The entire glycine molecule is incorporated into the growing purine ring. Serine is not utilized for the biosynthesis of either heme, creatine, or purines. Aspratate is used for purine ring synthesis only (one nitrogen of the purine ring is derived from aspartate). Neither cysteine nor glutamate is directly involved in the synthesis of heme, creatine, or purines.

In the biosynthetic pathways for the synthesis of heme, creatine, and guanine, which one of the following amino acids directly provides carbon atoms that appear in the final product? (A) Serine (B) Aspartate (C) Cysteine (D) Glutamate (E) Glycine

*The answer is C.* In humans and other land mammals, excess NH4+ is converted into urea in the liver for excretion by the kidneys. Malfunctions of the urea cycle can lead to hyperammonemia and result in brain damage. Urea is composed of two nitrogen groups and a carbonyl group. One nitrogen and the carbon are derived from free NH4+ and CO2 condensed to form carbamoyl phosphate (compound E). The other nitrogen is derived from the amino group of aspartate.

In the urea cycle diagram below, which compound is considered the end product?

*The answer is C.* Folate derivatives are crucial in the synthesis of DNA and in the conversion of vitamin B12 to one of its coenzyme forms. As summarized in the following chemical reaction, thymidylate synthetase catalyzes the methylation of dUMP (deoxyuridine monophosphate) to dTMP (deoxythymidine monophosphate) while converting the folate derivative 5,10-methylenetetrahydrofolate to dihydrofolate: 5,10-methylenetetrahydrofolate + dUMP = dihydrofolate + dTMP. This is the only de novo pathway for dTMP production. Thymidylate synthetase is therefore essential in its regulation of the supply of the four nucleotide precursors of DNA replication. DNA synthesis is also impaired when a deficit of 5,10-methylenetetrahydrofolate develops. One of the common consequences of reduced DNA synthesis is megaloblastosis. There is a salvage pathway using thymidine kinase that normally accounts for 5-10% of dTMP synthesis. Therefore, activation of this pathway with thymidine supplementation can partially compensate for diminished dTMP synthesis. *Educational Objective:* Folate deficiency inhibits the formation of deoxythymidine monophosphate (dTMP), which limits DNA synthesis and promotes megaloblastosis and erythroid precursor cell apoptosis. Because thymidine supplementation can moderately increase dTMP levels, it can reduce erythroid precursor cell apoptosis.

Laboratory animals deprived of folic acid experience a marked increase in marrow erythroid precursor cell production. Subsequently, many of these erythroid precursor cells undergo apoptosis without further maturation. Provision of which of the following supplements would reduce erythroid precursor cell apoptosis in these animals? (A) Homocysteine (B) Cytosine (C) Thymidine (D) Cobalamin (E) Glutamine

*The answer is D.* Amino acid degradation ultimately leads to the formation of ammonium ion (NH4+), which is toxic in significant amounts. In the liver of humans, as in most terrestrial vertebrates, NH4+ is produced and converted into urea for excretion. For many amino acids, the conversion of α-amino groups into ammonium ion and then into urea is carried out by two groups of enzymes. Transaminases (aminotransferases) transfer α-amino groups to α-ketoglutarate to form glutamate, which is then oxidatively deaminated by glutamate dehydrogenase to release free ammonium ion that can be converted to urea.

Liver aminotransferases, which are also called transaminases, catalyze the transfer of α-amino groups from many different amino acids to α-ketoglutarate. The intermediate produced is deaminated back to α-ketoglutarate with the formation of ammonium ion. The structure of α-ketoglutarate is shown below. What is the intermediate produced? a. Aspartate b. Alanine c. Oxaloacetate d. Glutamate e. Pyruvate

*The answer is B.* The key to answering this question correctly is understanding that phosphatidylcholine is formed by donation of methyl groups. Methionine is the only amino acid listed that can donate methyl groups. The activated form, S-adenosyl-L-methionine, is a very common methyl group donor. A mnemonic for remembering all ten essential amino acids is PVT TIM HALL (PriVaTe TIM HALL): Phenylalanine, Valine, Threonine, Tryptophan, Isoleucine, Methionine, Histidine, Arginine, Leucine, Lysine.

Phosphatidylcholine is a major component of red cell membranes, myelin, surfactant, and lipoproteins. Phosphatidylcholine is made from dietary choline or from choline released from phospholipid turnover. De novo synthesis of choline requires addit ion of methyl groups. Without the phospholipid turnover component as a source of choline, a deficiency in which essential amino acid would make dietary choline essential for phosphatidylcholine synthesis? A. Glycine B. Methionine C. Serine D. Threonine E. Valine

*The answer is C.* Deoxyguanosine would be first acted on by purine nucleoside phosphorylase, which would produce guanine and deoxyribose-1-phosphate. The guanine would be converted to GMP by HGPRT, and the GMP phosphorylated to GDP. The GDP would be reduced by ribonucleotide reductase to dGDP, which is then phosphorylated again to produce dGTP. Since guanine is a purine, pyrimidine nucleoside phosphorylase is not required in this pathway. There is no deoxyguanosine kinase (the only purine nucleosides that can be phosphorylated by adenosine kinase are adenosine and deoxyadenosine). APRT only works for the adenine base, not guanine. The 5′-nucleotidase is not required as there are no dephosphorylation events in the pathway outlined.

Many anticancer drugs are given to patients in their nucleoside form, rather than the nucleotide form. Which enzyme below will be required in the conversion of deoxyguanosine to dGTP? (A) Pyrimidine nucleoside phosphorylase (B) Deoxyguanosine kinase (C) Ribonucleotide reductase (D) Adenine phosphoribosyltransferase (E) 5′-nucleotidase

*The answer is A.* Most drugs used to treat depression do so by elevating serotonin levels, and serotonin is derived from tryptophan . Tyrosine is the precursor for catecholamines, while glutamate is the precursor of GABA. Histidine is the precursor for histamine, while glycine itself acts as a neurotransmitter in the brain.

Mr Smith had been prescribed a drug to treat his depression. One of the effects of the drug is to maintain elevated levels of a particular neurotransmitter that has been derived from which of the following amino acids? (A) Tryptophan (B) Tyrosine (C) Glutamate (D) Histidine (E) Glycine

*The answer is A.* Methionine and tetrahydrofolate are formed when methyl-tetrahydrofolate donates a methyl group to homocysteine. Tetrahydrofolate rapidly accepts one-carbon moieties to re-enter the cycle. The conversion of homocysteine to methionine requires the cofactor vitamin B12 (cobalamin). In vitamin B12 deficiency, tetrahydrofolate cannot be regenerated. Folate metabolism is consequently impaired. Defective DNA synthesis and its resultant megaloblastic erythropoiesis is seen in both vitamin B12 and folate deficiency. Homocysteine levels are elevated in both conditions as well. Elevated homocysteine is a risk factor for arterial and venous thrombosis. Vitamin B12 is alone responsible for the conversion of methylmalonyl CoA to succinyl CoA. Thus, in vitamin B12 deficiency, methylmalonyl CoA levels are elevated. The result is the incorporation of nor physiologic fatty acids into neuronal lipids. This contributes to the neurologic dysfunction in vitamin B12 deficiency as well. importantly, whereas homocysteine is elevated in both folic acid and vitamin B12 deficiency, methylmalonyl CoA s elevated in vitamin B12 deficiency only.

Recent studies have suggested that increased serum levels of homocysteine may predispose to thrombosis, and that decreasing one's homocysteine levels may benefit patients at risk for coronary or cerebral artery thrombosis. Folic acid and vitamin B12 supplementation can decrease serum homocysteine levels according to the following mechanism: (A) Methionine (B) Cysteine (C) Serine (D) Succinyl-CoA (E) Methylmalonyl-CoA (F) Glutamic acid

*The answer is D.* The formation of glutamate from glucose involves the TCA cycle intermediate α-ketoglutarate, which is formed from isocitrate in a reaction catalyzed by isocitrate dehydrogenase. α-Ketoglutarate is converted to glutamate either by glutamate dehydrogenase or a transaminase. The formation of serine, alanine, aspartate, and cysteine from glucose does not require the activity of isocitrate dehydrogenase. Serine is derived from 3-phosphoglycerate; alanine from pyruvate; aspartate from oxaloacetate; and cysteine from methionine (only the sulfur) and serine (the carbon atoms). The oxaloacetate needed for aspartate synthesis can be generated from pyruvate via the pyruvate carboxylase reaction.

Starting with glucose, the synthesis of which one of the following would require the participation of isocitrate dehydrogenase? (A) Serine (B) Alanine (C) Aspartate (D) Glutamate (E) Cysteine

*The answer is C.* Because allopurinol inhibits xanthine oxidase, the 6-mercaptopurine will not be deactivated as rapidly.

The anticancer drug 6-mercaptopurine is deactivated by the enzyme xanthine oxidase. A cancer patient being treated with 6-mercaptopurine develops hyperuricemia, and the physician decides to give the patient allopurinol. What effect will allopurinol have on the activity of 6-mercaptopurine? A. Enhanced deactivation of 6-mercaptopurine B. Enhanced elimination of 6- mercaptopurine as uric acid C. Enhanced retention and potentiation of activity D. Decreased inhibition of PRPP glutamylamidotransferase

*The answer is E.* The final thiolytic cleavage in β-oxidation of odd-chain fatty acids yields propionyl CoA. Propionyl CoA is also formed during the breakdown of methionine and isoleucine. It is carboxylated to form D-methylmalonyl CoA, which is in equilibrium with 1-methylmalonyl CoA. Valine forms methylmalonyl CoA during its degradation. The 1-isomer of methylmalonyl CoA is converted to succinyl CoA through the action of the B12 coenzyme-containing methylmalonyl CoA mutase. Thus, succinyl CoA serves as the entry point into the citric acid cycle for three amino acids and the last three carbons of odd-chain fatty acids. The amino acids and fatty acid carbons introduced in this manner may either be catabolized in the cycle for energy production or utilized for gluconeogenesis.

The entry point into the citric acid cycle for isoleucine, valine, and the product of odd-chain fatty acids is a. Fumarate b. Pyruvate c. Oxaloacetate d. Citrate e. Succinyl CoA

*The answer is E.* Given these symptoms, the defect is in the urea cycle and the elevated orotate suggests deficiency of ornithine transcarbamoylase.

Two days after a full-term normal delivery, a neonate begins to hyperventilate, develops hypothermia and cerebral edema, and becomes comatose. Urinalysis reveals high levels of glutamine and orotic acid. BUN is below normal. Which enzyme is most likely to be deficient in this child? A. Cytoplasmic glutaminase B. Cytoplasmic carbamoyl phosphate synthetase C. Cytoplasmic orotidylate decarboxylase D. Mitochondrial carbamoyl phosphate synthetase E. Mitochondrial ornithine transcarbamoylase

*The answer is B.* The nitrogenous bases are aromatic compounds. The pyrimidines—uracil, thymine, and cytosine contain one heterocyclic ring each. The purines—adenine and guanine—are derivatives of pyrimidines and consist of a pyrimidine ring joined with an imidazole ring. All the bases except for uracil are found in DNA; uracil replaces thymine in RNA. The ribose monophosphate form of uracil (UMP) serves as a precursor for cytidine triphosphate (UMP →UDP →UTP →CTP). In these series of reactions, the uracil moiety is aminated with an amine contributed by glutamine, to form cytosine. UMP also gives rise to the deoxyribose monophosphate form of uridylate (dUMP), which forms deoxythymidylate in a reaction catalyzed by thymidylate synthase. In this reaction, uracil is methylated with a methylene contributed by N5,N10-methylenetetrahydrofolate (M-THF). The widely used chemotherapy drug methotrexate (amethopterin) blocks the regeneration of THF to M-THF, preventing the thymidylate synthase reaction and blocking rapidly dividing cells from synthesizing DNA. Another chemotherapeutic drug, fluorouracil, is converted to fluorodeoxyuridylate (F-dUMP) by in vivo enzymes. F-dUMP is a suicide inhibitor of thymidylate synthase, which forms an irreversible covalent complex with the enzyme.

Which base derivative can serve as a precursor for the synthesis of two of the other base derivatives shown? a. Cytidine triphosphate (CTP) b. Uridine monophosphate (UMP) c. Deoxythymidine monophosphate (dTMP) d. Adenosine triphosphate (ATP) e. Deoxyadenosine monophosphate (dAMP)

*The answer is B.* During the early phases of starvation, the catabolism of proteins is at its highest level. Anabolic enzymes, which are not utilized during starvation, are degraded and their synthesis is repressed. The deamination of amino acids for gluconeogenesis and ketogenesis results in a negative nitrogen balance. Hence, ammonia and urea levels in the urine exceed normal values. The glucose formed from gluconeogenic amino acids becomes the major source of blood glucose following depletion of liver glycogen stores. Complete oxidation of this glucose, as well as the ketone bodies formed from ketogenic amino acids, leads to a relative increase in the CO2 and H2O formed from amino acid carbon skeletons.

Which one of the following products of protein metabolism is decreased below normal levels during the early stages of starvation? a. Urea b. Anabolic enzymes c. CO2 d. NH4+ e. Glucose

*The answer is D.* Pepsin is secreted in a proenzyme form in the stomach. Unlike the majority of proenzymes, it is not activated by protease hydrolysis. Instead, spontaneous acid hydrolysis at pH 2 or lower converts pepsinogen to pepsin. Hydrochloric acid secreted by the stomach lining creates the acid environment. All the enzymes secreted by the pancreas are activated at the same time upon entrance into the duodenum. This is accomplished by trypsin hydrolysis of the inactive proenzymes trypsinogen, chymotrypsinogen, procarboxypeptidase, and proelastase. Primer amounts of trypsin are derived from trypsinogen by the action of enteropeptidase secreted by the cells of the duodenum.

Which one of the following proteolytic enzymes is activated by acid hydrolysis of the proenzyme form? a. Trypsin b. Chymotrypsin c. Elastase d. Pepsin e. Carboxypeptidase

*The answer is E.* Patients with Lesch-Nyhan syndrome develop severe gout as the free bases, guanine and hypoxanthine, can no longer be salvaged. As these bases accumulate, urate is produced in excess, leading to gout. This is frequently seen in infants with this disorder as an orange sand-like compound. Xanthine, hypoxanthine, guanine, and adenine are not accumulating in the urine, as these molecules are metabolized to produce urate.

A 1-year-old boy was brought to the pediatrician due to a developmental delay, biting of his lips and fingers, and the presence of orange crystals in his diapers. Enzymatic analysis shows loss of 99% of the activity of a particular enzyme. The orange sand in the diapers was composed of which of the following? (A) Xanthine (B) Hypoxanthine (C) Guanine (D) Adenine (E) Urate

*The answer is F.* NH2 (ammonia) is produced during protein catabolism. NH1 is toxic and must be excreted. NH2 is taken to the liver, where it is then converted to urea for excretion. In muscle tissue, NH3 is transferred from glutamate to pyruvate in transamination reaction, forming alanine and a-ketoglutarate. Alanine exits the muscle and travels in the blood to the liver, where it is taken up and undergoes the same transamination reaction in reverse, regenerating pyruvate and glutamate. The pyruvate is used in gluconeogenesis and the glutamate provides one of the NH3 groups that form the molecule urea

A 10-year-old boy is evaluated in the emergency department after recently being rescued from a national park. He was on a camping trip when he became separated from the rest of his family. He was located by a park ranger four days later. During this time he subsisted on one granola bar and some nuts and berries that he foraged. Vital signs include temperature 362C (96.8'F), heart rate 64 beats/min, blood pressure 80/56 mmHg, respiratory rate 12 breaths/min, and oxygen saturation 100% on room air. On physical exam, he appears thin and his muscles are weak. Heart, lungs, and abdomen are normal. He has numerous bruises and abrasions on his skin. During the period of starvation experienced by this young boy, protein primarily from skeletal muscle was catabolized resulting in the accumulation of nitrogenous waste. In what form was the excess nitrogen transported from his skeletal muscle tissue to his liver for excretion? A. a-ketoglutarate B. Albumin C. Ammonia D. Pyruvate E. Urea F. Alanine

*The answer is C.* The child most likely has severe combined immunodeficiency caused by adenosine deaminase deficiency. This enzyme deaminates adenosine (a nucleoside) to form inosine (another nucleoside). Hypoxanthine and xanthine are both purine bases, and the monophosphates are nucleotides.

A 12-week-old infant with a history of persistent diarrhea and candidiasis is seen for a respiratory tract infection with Pneumocystis jiroveci. A chest x-ray confirms pneumonia and reveals absence of a thymic shadow. Trace IgG is present in his serum, but IgA and IgM are absent. His red blood cells completely lack an essential enzyme in purine degradation. The product normally formed by this enzyme is A. guanine monophosphate B. hypoxanthine C. inosine D. xanthine E. xanthine monophosphate

*The answer is D.* This patient is presenting with photosensitive dermatitis, diarrhea, and central nervous system disturbances pointing to the triad of pellagra. Pellagra is the result of niacin deficiency and can occur because of insufficient dietary intake or an inherited error of metabolism. Considering the boy appears well nourished on exam with no history of insufficient dietary intake, the possibility of an inherited error of metabolism is higher on the differential. This possibility is further suggested by the boy's significant aminoaciduria. Several disorders are caused by deficient renal tubular reabsorption of amino acids that can result in high levels of amino acids lost in the urine. One of these disorders is known as Hartnup disease, which is caused by a mutation in the transport of neutral amino acids, including that of tryptophan. Tryptophan is a precursor to niacin and an essential amino acid. Hartnup disease therefore manifests as pellagra due to tryptophan, and consequently niacin, deficiency.

A 12-year-old boy is brought by his mother to the clinic because of persistent rash and peculiar behavior. She reports that over the past few years the boy has developed a rough, red rash on his face, neck, and hands that gets worse when he is outdoors. She reports that he has been "tottering like an old man" since the beginning of the year and that his hands are consistently shaky. He is not doing well in school. More often than not his stools are loose and watery. The mother reports that he eats a normal, balanced diet. On physical examination, the boy appears well nourished, with an erythematous, scaling rash like that in the image. He has bilateral nystagmus and an intention tremor. He walks with a teetering, wide-based gait. Urinalysis reveals significant aminoaciduria. Deficiency of which amino acid is likely responsible for this patient's presentation? A. Cystine B. Histidine C. Isoleucine D. Tryptophan E. Tyrosine

*The answer is B.* The amino acid methionine can be metabolized into S-adenosyl methionine (SAM), which acts as a methyl-donor for many methyltransferase reactions. After the transfer of a methyl group, SAM is converted into S-adenosyl-homocysteine, which is broken down to form adenosine and homocysteine. Subsequently, the conversion of homocysteine to cystathionine requires the enzyme cystathionine synthetase, the amino acid serine, and the cofactor vitamin B6. Cystathionine is then converted to cysteine by the enzyme cystathionase, which also requires vitamin B, as a cofactor. Alternatively, the enzyme methionine synthase uses vitamin B12 as a cofactor to revert homocysteine back to methionine. This patient most likely has homocystinuria, a condition that leads to hypercoagulability and thromboembolic occlusion. Because homocysteine is prothrombotic, individuals with complete cystathionine synthase deficiency can develop premature acute coronary syndrome, as seen in this patient (based on his troponin level and ECG Findings). Other clinical features include ectopia lends (ocular lens displacement) and intellectual disability. The most common cause of homocystinuria is a defect in cystathionine synthase. Affected patients cannot for cysteine from homocysteine; therefore, cysteine is essential in their diet In addition, homocysteine buildup results in hypermethioninemia, as seen in this patient.

A 12-year-old boy is brought to the emergency department with severe chest pain. He has had intermittent substemal chest pain for the past few months that typically occurs after heavy activity. The boy's activities have been limited due to the chest pain, and he is no longer able to play on the soccer team. The patient does not use tobacco or illicit drugs. His temperature is 36.7 C (98 F), blood pressure is 130/80 mm Hg, pulse is 132/min, respirations are 24/min, and pulse oximetry is 98% on room air. BMI is 17 kg/m^2. Physical examination shows an anxious appearing boy with a rapid but regular pulse. No abnormalities are seen. Troponin is elevated, and ECG reveals ST segment elevations in leads II, II I, and aVF. After acute stabilization and treatment, further laboratory workup shows an increased serum methionine level. Which of the following amino acids is most likely essential in this patient? (A) Asparagine (B) Cysteine (C) Isoleucine (D) Leucine (E) Tyrosine) (F) Valine

*The answer is E.* The boy is exhibiting the symptoms of homocystinuria, usually caused by a defect in cystathionine β-synthase. Cystathionine β-synthase will condense homocysteine with serine to form cystathionine. An inability to catalyze this reaction will lead to an accumulation of homocysteine, which will oxidize to form homocystine. The elevated serine can be metabolized back into the glycolytic pathway. Methionine will not increase in blood as the homocysteine produced is converted into homocystine. Phenylpyruvate is a diagnostic marker for PKU, but it is not relevant for homocysteine production or degradation. Fibrillin is mutated in Marfan syndrome, but this disorder is not Marfan syndrome.

A 12-year-old boy is brought to the pediatrician because of behavioral problems noted by the parents. Upon examination, the physician notices brittle and coarse hair, red patches on the skin, long, thin arms and legs (reminiscent of Marfan syndrome patients), scoliosis, pectus excavatum, displaced lens, and muscular hypotonia. Blood work is likely to show an elevation of which of the following metabolites? (A) Methionine (B) Phenylpyruvate (C) Cysteine (D) Fibrillin fragments (E) Homocystine

*The answer is D.* Cystathionine β-synthase is a B6 requiring enzyme (the reaction is a β-elimination of the serine hydroxyl group, followed by a β-addition of homocysteine to serine; both types of reactions require the participation of B6). In some mutations, the affinity of the cofactor for the enzyme has been reduced, so significantly increasing the concentration of the cofactor will allow the reaction to proceed. The enzyme does not require the assistance of B1, B2, B3 (niacin), or B12 to catalyze the reaction.

A 12-year-old boy is brought to the pediatrician because of behavioral problems noted by the parents. Upon examination, the physician notices brittle and coarse hair, red patches on the skin, long, thin arms and legs (reminiscent of Marfan syndrome patients), scoliosis, pectus excavatum, displaced lens, and muscular hypotonia. Treatment with which of the following vitamins may be successful in controlling this disorder? (A) B1 (B) B2 (C) B3 (D) B6 (E) B12

*The answer is C.* The endoplasmic reticulum (ER) is a continuous system of folded membranes that enclose a single cisternal space. The rough ER (RER) has ribosomes attached and is the site of synthesis of secretory, lysosomal, and integral membrane proteins. Once synthesized, many proteins undergo post-translational modification inside the RER and most are targeted tor export to the Golgi apparatus. These proteins pass from the RER to the Golgi apparatus within COP II coated transport vesicles. The Golgi apparatus sorts and distributes proteins to the cell membrane, organelles, and secretory granules. In contrast to the RER, the smooth ER (SER) contains enzymes tor steroid and phospholipid biosynthesis. All steroid-producing cells (eg, cells in the adrenals, gonads. and liver) contain a well-developed SER. ACTH primarily stimulates the cells in the adrenal cortex to produce glucocorticoids, but it also increases adrenal production of mineralocorticoids and androgens to a lesser extent. These are all cholesterol-derived steroid hormones, and the cells that synthesize them contain prominent SERs. The SER is also involved in the detoxification of numerous drugs, especially in hepatocytes. In addition, a specialized SER functions as the sarcoplasmic reticulum in striated muscle cells.

A 12-year-old girl is involved in a motor vehicle accident and brought to the emergency department An abdominal CT scan is negative for acute trauma. However, she is incidentally found to have an abnormal intraabdominal mass. Biopsy of the mass reveals ectopic tissue. On electron microscopy, these cells have a highly developed smooth endoplasmic reticulum. These cells most likely receive stimulation from which of the following agents? A. Parathyroid hormone B. Progesterone C. ACTH D. Aldosterone E. Dopamine

*The answer is D.* At least three specific small bowel enterocyte apical transport proteins appear to be involved in the absorption of amino acids from the diet. In Hartnup disease, the intestinal and renal absorption of tryptophan is defective. Tryptophan is an essential amino acid and a precursor for nicotinic acid, serotonin, and melatonin. The clinical manifestations of Hartnup disease are primarily due to the malabsorption of tryptophan, resulting in niacin (Vitamin B2) deficiency, because niacin is synthesized from tryptophan. Most children with Hartnup disease are asymptomatic, but some children experience photosensitivity and pellagra-like skin rashes as in the case described above. Neurologic involvement can occur most commonly leading to ataxia. Neurologic and skin symptoms typically wax and wane during the course of this disease. The main laboratory finding in Hartnup disease is aminoaciduria, restricted to the neutral amino acids (alanine, serine, threonirie, valine, leucine, isoleucine, phenylalanine, tyrosine, tryptophan, and histidine). The urinary excretion of proline, hydroxyproline, and arginine remains unchanged, and this important finding differentiates Hartnup disease from other causes of generalized aminoaciduria such as Fanconi syndrome. Treatment with nicotinic acid or nicotinamide and a high-protein diet generally results in significant improvement of symptoms.

A 12-year-old male Is evaluated for ataxia accompanied by episodic erythematous did pruritic skin lesions and loose stools. Laboratory evaluation reveals loss of neutral aromatic amino acids in the urine. This patient's symptoms would most likely respond to which of the following supplements? (A) Thiamine (B) Riboflavin (C) Folic acid (D) Niacin (E) Pyridoxine (F) Tocopherol

*The answer is D.* The boy has primary oxaluria type I, an autosomal recessive trait, which is a defect in a transaminase that converts glyoxylate to glycine. If this transaminase is defective, glyoxylate will accumulate. The glyoxylate will then be oxidized to oxalate, which, in the presence of calcium, will precipitate and form stones in the kidney. The enzyme that catalyzes this reaction is the D-amino acid oxidase. Alanine, leucine, and lysine metabolism do not give rise to oxalate.

A 13-year-old boy is admitted to the hospital due to flank and urinary pain. Analysis demonstrates the presence of kidney stones. The stones were composed of calcium oxalate. Family history revealed that the boy's father and mother had had similar problems. Oxalate accumulation arises in this patient due to difficulty in metabolizing which of the following? (A) Alanine (B) Leucine (C) Lysine (D) Glyoxylate (E) Glycine

*The answer is F.* This patient likely has hereditary orotic aciduria, a rare autosomal recessive disorder of de novo pyrimidine synthesis that results in physical and mental retardation (eg, low height/weight, delayed developmental milestones), megaloblastic anemia (eg, elevated mean corpuscular volume, low reticulocyte count), and elevated urinary orotic acid levels. Increased urinary erotic acid may also be seen in ornithine transcarbamylase deficiency, however, patients with this condition classically have a failure to thrive and hyperammonemia encephalopathy within the first few weeks of life (due to impaired urea synthesis). Hereditary orotic aciduria occurs due to a defect in uridine 5' monophosphate (UMP) synthase, a polypeptide containing 2 enzymatic domains (orotate phosphoribosyltransferase and OMP decarboxylase) that catalyze tile final conversion of orotic acid to UMP. Impaired conversion of orotic acid to UMP results in the excretion of large amounts of orotic acid in the urine and the clinical features described above. Uridine supplementation can bypass this enzymatic defect and improve symptoms as uridine is convened to UMP via nucleoside kinases.

A 14-month-old boy is evaluated for failure to thrive and developmental delay. His mother reports that at 12 months he could barely lift his head and had difficulty sitting unsupported. The toddler has not started babbling or forming words. He is at the 10th percentile for height and 5th percentile for weight. Laboratory results are as follows: Hemoglobin 8.6 g/dL Mean corpuscular volume 114 fL Reticulocytes 1% Ammonia, plasma 42 µg/dL (normal 40 - 80 µg/dL) Urine specimens contain large amounts of erotic acid crystals. Supplementation with which of the following substances would most likely benefit this patient? (A) Ascorbic acid (B) Folic acid (C) Guanine (D) Iron (E) Pyridoxine (F) Uridine

*The answer is C.* This patient has acute intermittent porphyria (AIP), an autosomal-dominant disorder caused by a lack of porphobilinogen deaminase (previously called uroporphyrinogen I synthetase). The buildup of toxic levels of δ-aminolevulinate and porphobilinogen lead to the associated symptoms of abdominal pain (more than 90% of cases), neuropathy, high sympathetic tone, muscle weakness, and neuropsychiatric disturbances, including anxiety, depression, seizures, and paranoia. AIP almost never presents before puberty, and it can be hard to diagnose because of its acute nature. Untreated, it can lead to paralysis and death.

A 17-year-old boy presents to the emergency department with severe abdominal pain, Laboratory tests show a deficit in porphobilinogen deaminase and excess 5-aminolevulinate and porphobilinogen in the urine. Which of the following symptoms would most likely also be present in this patient? A. Chest pain and bradycardia B. Hypotension and headache C. Neuropsychiatric disturbances and muscle weakness D. Polyphagia and polyuria E. Stiff neck and headache

*The answer is C.* Biopterin is an enzyme cofactor that can be synthesized de novo by somatic cells. It is required for the function of aromatic amino acid hydroxylases.

A 17-year-old girl is considering several vegan minimalist diets with very limited sources of nutrition. She asks her doctor which nutrients she must have in her food and which ones she can do without. He tells her that many nutritional components cannot be synthesized by the body and must be obtained in food. Under no physiologic stressors, which of the following substances can be synthesized by the somatic cells of a healthy adult? A. Leucine B. Linoleic acid C. Biopterin D. Folic acid E. Valine

*The answer is B.* The presence of an unidentified acid in the blood of a newborn should hint at a congenital metabolic disease. Identification of one of the acids as leucine (a branched-chain amino acid, along with isoleucine and valine) should lead to a diagnosis of maple syrup urine disease (MSUD). The fundamental problem in MSUD is the inability to break down these three amino acids. Normally, the enzyme complex branched-chain α-keto acid dehydrogenase would break them down, and they would then enter the tricarboxylic acid cycle as acetyl coenzyme A (CoA) or succinyl CoA. When they do not break down, they remain in the blood and induce a metabolic acidosis that manifests as intellectual disability and physical retardation, feeding difficulty, and urine with a sweet (maple syrup) odor, eventually leading to seizures, coma, and/or death if untreated. The treatment is the elimination of valine, leucine, and isoleucine from the diet.

A 2-week-old boy is being evaluated in the emergency department after being found in a coma. The mother reports her son has been lethargic. The infant's arterial pH is 6.8, partial pressure of carbon dioxide in the blood is 25 mm Hg, and bicarbonate level is 16 mEq/L. A metabolic work-up is initiated. Urine organic acid testing reveals elevated levels of leucine. A mutation in which of the following enzymes is most likely responsible for the infant's symptoms? A. Alanine aminotransferase B. Branched-chain α-keto acid dehydrogenase C. Carbamoyl phosphate synthetase I D. Glutamate dehydrogenase E. Ornithine transcarbamylase F. Phenylalanine hydroxylase

*The answer is B.* Maple syrup urine disease is associated with dystonia, which manifests as sustained muscle contractions resulting in twisting and repetitive movements or abnormal fixed postures. This usually manifests by 4 days of age but may not develop until the infant is 4-7 days old, depending on the amount of protein in the infant's feeding regimen. Additionally, breastfeeding may delay onset into the second week. The mechanism for dystonia is thought to be due to altered levels of dopamine, GABA, and glutamate, and there are also associations with high serum leucine:tyrosine ratios.

A 2-week-old infant is brought to his pediatrician for a well-baby check-up. His mother is concerned because he has not been feeding well and appears to have lost weight. This is confirmed on his growth chart; he has dropped from the 70th percentile for height and weight at birth to the 30th percentile in the past 2 weeks. His blood is found to contain excess amounts of leucine, isoleucine, and valine. This infant is most likely to exhibit which of the following additional symptoms? A. Darkening of urine B. Dystonia C. Hyperglycemia D. Recurrent urinary tract infection E. Red urine with no measurable RBCs F. Renal stones

*The answer is D.* Lesch-Nyhan syndrome is an X-Iinked recessive disorder that is characterized by a deficiency of the enzyme hypoxanthine-guanine phosphoribosyltransferase (HGPRT). HGPRT mediates the conversion of hypoxanthine and PRPP to inosine monophosphate and pyrophosphate. This reaction is an important step in the purine salvage pathway, and its deficiency leads to an increase in uric acid production secondary to increased purine breakdown. This can manifest as orange "send" sodium urate crystals in an infants diaper. Other clinical manifestations include delayed developmental milestones, intellectual disability, self-mutilating behavior, and motor symptoms (both extrapyramidal and pyramidal). uric acid overproduction in these patients can be treated with the xanthine oxidase inhibitor allopurinol.

A 2-year-old boy is brought to his pediatrician for his well-child exam. His parents are concerned because he is still unable to walk unassisted. On interview, they mention the appearance of orange "sand" in the child's diapers. The pediatrician notices that past documentation is significant for consistently delayed milestones. Family history is noncontributory. Physical exam is significant for old and new bite wounds around the child's lips and hands. Genetic testing reveals markedly low hypoxanthine-guanine phosphoribosyltransferase (HGPRT) enzyme activity This enzyme normally produces which of the following products from hypoxanthine and 5-phosphoribosyl I-pyrophosphate (PRPP)? A. Guanosine monophosphate and pyrophosphate B. Adenosine monophosphate and pyrophosphate C. Nicotinamide adenine dinucleotide phosphate D. Inosine monophosphate and pyrophosphate E. Ribose-5-phosphate

*The answer is A.* Bilirubin is a degradative product of hemoglobin metabolism. Bilirubin (pigment) stones are specifically associated with excessive bilirubin production in hemolytic anemias, including sickle cell anemia. Bilirubin stones can also be seen in hepatic cirrhosis and liver fluke infestation.

A 25-year-old woman with sickle cell anemia is brought to the emergency department because of steady pain in the right upper quadrant pf her abdomen. She states that the pain radiates to the right shoulder. especially after large or fatty meals. A diagnosis of cholelithiasis is made. Which of the following is the most likely primary composition of the stones? A. Calcium bilirubinate B. Calcium oxalate C. Cholesterol D. Cholesterol and calcium bilirubinate E. Cystine

*The answer is B.* There are 9 essential amino acids: histidine, isoleucine, leucine, lysine, methionine, phenylalanine, threonine, tryptophan, and valine. Essential means it must be obtained from the diet, whereas the rest (e.g. gtycine) can be synthesized de novo in the body.

Which of the following amino acids can be synthesized de novo? A. Isoleucine B. Glycine C. Lysine D. Methionine E. Valine

*The answer is D.* Alkaptonuria is an autosomal recessive disorder of tyrosine metabolism. Deficiency of homogentisic acid dioxygenase blocks homogentisic acid metabolism, preventing the conversion of tyrosine to fumarate. Homogentisic acid accumulates in the body and is excreted in the urine, imparting a black color to the urine if allowed to sit and undergo oxidation. In patients with alkaptonuria, the retained homogentisic acid selectively binds to collagen in connective tissues, tendons, and cartilage. This leads to "ochronosis," a blue-black pigmentation most evident in the ears, nose, and cheeks, and ochronotic arthropathy, which typically manifests during adulthood. *Educational Objective:* Alkaptonuria is an autosomal recessive disorder in which the beck of homogentisic acid dioxygenase blocks the metabolism of tyrosine, leading to an accumulation of homogentisic acid. Clinical features include a black urine color when exposed to air, a blue-black pigmentation on the face, and ochronotic arthropathy.

A 2-year-old boy is brought to the office by his parents. He is currently toilet-training during the day and at nighttime. After he urinated in the toilet last night, his parents forgot to flush the toilet and noticed that the boy's urine turned black overnight. The child has no significant past medical history and takes no medications. He can say 2-word sentences, follow 2 step directions, and jump with 2 feet off the ground. Examination shows a well-nourished child with no swelling or tenderness of any joints. Urinalysis results are as follows' Color black Specific gravity 1.022 Protein none Blood negative Glucose negative Ketones negative Leukocyte esterase negative Which of the following conversion pathways is most likely deficient in this patient? A. Leucine to acetoacetate B. Phenylalanine to tyrosine C. Serine to cysteine D. Tyrosine to fumarate E. Valine to glutamic acid

*The answer is C.* This patient has Lesch-Nyan syndrome, which is a deficiency in hypoxanthine-guanine phosphoribosyltransferase (HGPRT). Symptoms include hyperuricemia and self-mutilation.

A 2-year-old boy with mental retardation has chewed the tips of his fingers on both hands and a portion of his lower lip. His serum uric acid concentration is increased, and he has a history of uric acid renal calculi. His 5-year-old brother has similar findings. Which of the following abnormal enzyme activities is the most likely cause of these findings? (A) Decreased adenine phosphoribosyltransferase (B) Decreased adenosine deaminase (C) Decreased hypoxanthine-guanine phosphoribosyltransferase (D) Increased phosphoribosylpyrophosphate synthetase (E) Increased xanthine oxidase

*The answer is B.* Ammonia generated from the metabolism of alpha-amino acids is converted into urea by the urea cycle. The urea cycle involves five enzymatic steps, two in the mitochondrial matrix and three in the cytosol. The combination of CO, ammonia, and ATP, catalyzed by carbamoyl phosphate synthase (the rate-limiting enzyme in the urea cycle), forms carbamoyl phosphate as the first step o the urea cycle. Carbamoyl phosphate then combines with ornithine to form citrulline in a reaction catalyzed by ornithine transcarbamoylase in the mitochondrial matrix. Citrulline then enters the cytosol and is converted to argininosuccinate, which is then converted to arginine. The conversion of arginine to ornithine by the cytosolic enzyme arginase completes the urea cycle by releasing a urea molecule. Remember that urea synthesis is a cyclic process and that while ammonium ion, CO ATP and aspartate are consumed in this process, there is no net loss or gain of ornithine. citrulline, argininosuccinate. or arginine. The molecule N-acetyl glutamate serves as a regulator of the urea cycle through allosteric activation of carbamoyl phosphate synthetase I. Disorders of the urea cycle can result from defects in any of the following six enzymes: 1. Carbamoyl phosphate synthetase (CPS) 2. Ornithine transcarbamylase (OTC) 3. Arglnlnosucclnlc acid synthetase (AS) 4. Arglnlnosucclnlc acid lyase (AL) 5. Arginase (AG) 6. N-Acetylglutamate synthetase NAGS) The first five enzymes are directly involved in the urea cycle whereas the sixth enzyme is involved in the production of N-acetylglutamate, the allosteric activator of carbamoyl phosphate synthase I. Patients with urea cycle disorders display clinical symptoms of as neurological damage secondary to increased serum ammonia levels. Typically, patients present early in childhood, however, milder defects can present for the first tire during adulthood. OTC deficiency is the most common urea cycle disorder, resulting in increased levels of carbamoyl phosphate and impaired disposal of ammonia.

A 2-year-old male demonstrates periodic lethargy, vomiting, and confusion. Laboratory testing reveals increased blood ammonium levels during these episodes as well as markedly increased urine orotic acid excretion. Which of the following enzymes is most likely to be deficient in this patient? (A) Carbamoyl phosphate synthase I (B) Ornithine transcarbamylase (C) N-acetyl glutamate synthase (D) Hypoxanthine-guanine phosphoribosyltransferase (E) Adenosine deaminase

*The answer is D.* The patient has the signs of albinism, a lack of melanin. Melanin is produced from tyrosine (tyrosine is hydroxylated in melanocytes to form dopa, which then enters the pathway for melanin production). The defective enzyme in oculocutaneous albinism (the type exhibited by this patient, as opposed to ocular albinism, which only affects the eyes) is tyrosinase. This is a melanocyte-specific genetic deficiency neuronal cells also contain tyrosinase, which is needed to produce the catecholamines, and that isozyme is normal in patients with albinism. A defect in branched-chain amino acids would lead to MSUD; in phenylalanine, PKU; and in tryptophan, low serotonin levels. A defect in histidine metabolism may lead to reduced levels of histamine.

A 20-year-old male is new to your practice, and you notice he has white hair, white skin, and nystagmus. A defect in the metabolism of which one of the following compounds is responsible for this presentation? (A) Branched-chain amino acids (B) Histidine (C) Tryptophan (D) Tyrosine (E) Methionine

*The answer is B.* Tyrosine is a precursor of the catecholamines dopamine, epinephrine, and norepinephrine. The pathway for catecholamine production is normal in individuals with albinism, as it is the melanocyte isozyme of tyrosinase that is mutated, not the form in the cells that produce the catecholamines.

A 20-year-old male is new to your practice, and you notice he has white hair, white skin, and nystagmus. The amino acid(s) pathway that contained a mutation as indicated in the previous question is also a precursor for which one of the following? (A) Serotonin (B) Norepinephrine (C) Testosterone (D) Aldosterone (E) Histamine

*The answer is A.* Anemias may be classified as microcytic, normocytic or macrocytic based on the mean corpuscular volume (MCV). Orotic aciduria is a macrocytic (MCv'>100 L) anemia that is the result of inadequate de novo pyrimidine synthesis. Normally, orotic acid is converted to uridine monophosphate (UMP) by UMP synthase, but in orotic aciduria IJMP synthase is defective. This diagnosis is made in children who present with a megaloblastic anemia refractory to folate and vitamin B12 supplementation. Patients are given UMP to bypass the mutated enzyme.

A 3-month-old infant is being evaluated for developmental delay and failure to thrive. A laboratory blood work reveals the following: The patient is given folate and vitamin B12 supplementation, but he makes no improvement over the next couple of weeks. What is the most likely diagnosis A. Orotic aciduria B. Vitamin B6 deficiency C. Lesch-Nyan Syndrome D. Cystathione synthase deficiency E. Ornithine transcarbamylase deficiency

*The answer is E.* This woman has phenylketonuria (PKU), whose clinical effects depend upon the levels of phenylalanine, an essential amine acid. Aspartame (N-asparlyl-phenylalanine methyl ester), which is widely used as an artificial sweetener. must be strictly avoided by phenylketonurics. Phenylalanine crosses the placenta and, if maternal serum levels are elevated, acts as a teratogen to the developing fetus. This condition is known as maternal PKU. Although the mother can stay well with substantial elevations in serum phenylalanine concentration, the children born to such women are usually profoundly retarded and may have multiple birth defects.

A 20-year-old primigravid woman who is 2 months pregnant comes to the physician for her first routine prenatal examination. She states that she had a congenital deficiency of phenylalanine metabolism as a child and required a special diet in which she was not able to ingest anything sweetened with aspartame. Laboratory studies show markedly elevated maternal serum levels of phenylalanine. Genetic studies have not been performed on the patient's husband, Which of the following is the most appropriate response by the physician? A. Childhood phenylalanine restriction is sufficient to protect the health of the fetus in the future B. Further information is required to ascertain if the fetus is at risk C. The fetus is at no health risk if it is heterozygous for the PKU gene D. The fetus is at no health risk if phenylalanine levels are normalized by the third trimester E. The mother's hyperphenylalaninemia may already have harmed the fetus

*The answer is A.* A deficiency of any of the enzymes responsible for porphyrin synthesis can result in porphyria, which can be broadly classified as either hepatic or erythropoietic. depending on the site of the enzymatic deficiency. Heme is synthesized in the liver for use in the cytochrome p450 enzyme system, while heme in the bone marrow is generated for hemoglobin use. The synthesis pathways in the liver and bone marrow are regulated differently because the heme generated by these two separate tissues serves different functions. The clinical manifestations of porphyria result from the accumulation of precursors of porphyrins in the blood, tissues, and urine. Acute attacks of intermittent hepatic porphyry can be precipitated by administration of drugs such as phenobarbital, griseofulvin, and phenytoin. Additionally, alcohol and a low-calorie diet can induce an acute attack of porphyria. All of these factors precipitate porphyria symptoms by decreasing the hepatic concentration of heme, which causes an increase in hepatic ALA synthase activity and leads to increased formation of δ-aminolevulinic acid and porphobilinogen. Since heme serves to negatively feedback inhibit the synthesis of ALA synthase, a reduction in heme synthesis subsequently leads to increased δ-aminolevulinic acid and porphobilinogen. In patients with acute intermittent porphyries, δ-aminolevulinic acid and porphobilinogen accumulate because of a congenital blockage, and high concentrations of these intermediates are responsible for the acute abdominal pain and neurologic symptoms observed in this disease. The diagnosis of acute intermittent porphyry is made by demonstrating elevated δ-aminolevulinic acid and porphobilinogen during acute attacks. *Educational Objective:* Decreased heme concentration results in an increase in hepatic ALA synthase activity, which in tum, leads to increased formation of δ aminolevulinic acid and porphobilinogen. Increased formation of δ-aminolevulinic acid and porphobilinogen occurs because heme normally serves to inhibit the synthesis of ALA synthase.

A 22-year-old Caucasian female with recurrent abdominal pain and anxiety has had marked improvement in her symptoms after intravenous administration of a heme preparation. Rapid improvement of symptoms in this patient is most likely due to significant repression of A. δ-Aminolevulinate synthase B. δ-Aminolevulinate dehydratase C. Hydroxymethylbilane synthase D. Ferrochelatase E. Bilirubin glucuronyl transferase

*The answer is D.* This patient most likely has porphyry cutanea tarda, the most common disorder of porphyrin synthesis. Enzyme deficiencies in the early steps in porphyrin synthesis cause abdominal pain and neuropsychiatric manifestations without photosensitivity, while late step derangements (after the condensation of porphobilinogen) cause photosensitivity. More specifically, defects in URO decarboxylase, COPRO oxidase, PROTO oxidase, and Ferrochelatase result in photosensitivity. Photosensitivity induced by porphyria is thought to be mediated by the formation of porphyrin-mediated superoxide free radicals from oxygen upon exposure to sunlight. (Choice A) Deficiency of ALA-synthase will result in a decrease in the formation of all porphyrins. This deficiency will not result in porphyry but will result in a decrease in heme synthesis and concurrent hypochromic, microcytic anemia. Pyridoxal phosphate (Vitamin B6) is the cofactor required for activity of ALA synthase, thus, pyridoxine deficiency can result in microcytic hypochromic anemia secondary to decreased heme synthesis. (Choices B and C) Deficiencies in ALA dehydrase and HMB synthase do not result in photosensitivity because the metabolites that accumulate in these enzyme deficiencies are not porphyrinogens or porphyrins and are therefore unable to react with oxygen upon excitation by ultraviolet light. (Choice E) Bilirubin glucuronyltransferase is a hepatic enzyme responsible for the conjugation of bilirubin with glucuronide, improving solubility for biliary excretion. A decrease in glucuronyltransferase results in unconjugated hyperbilirubinemie. *Educational Objective:* Enzyme deficiencies of the early steps in porphyrin synthesis cause neuropsychiatric manifestations without photosensitivity, while late step derangements lead to photosensitivity. Photosensitivity in porphyrin causes vesicle and blister formation on sun-exposed areas as well as edema, pruritis, pain, and erythema.

A 22-year-old Caucasian male develops recurrent skin blistering and prefers to work as nocturnist. Laboratory evaluation shows elevated total plasma porphyrins. Which of the following enzymes is most likely deficient in this patient? (A) δ-Aminolevulinate synthase (B) δ-Aminolevulinate dehydrase (C) HMB synthase (D) Uroporphyrinogen decarboxylase (D) Bilirubin glucuronyl transferase

*The answer is E.* Deficiency of porphobilinogen deaminase (also known as uroporphyrinogen I synthetase) causes acute intermittent porphyria (AIP), a disease characterized by acute attacks of gastrointestinal, neurologic/psychiatric, and cardiovascular symptoms. A deficiency in porphobilinogen deaminase leads to a deficiency in heme synthesis, in turn causing a build-up of intermediary products such as aminolevulinic acid and porphobilinogen, which cause the symptoms of the disease. Age at onset is almost always after puberty, it is more common in women, and symptoms often are precipitated by drugs that induce heme-containing CYP450. Because of the intermittent nature of attacks, AIP can be difficult to diagnose. These patients often will present with urine that turns a dark color when exposed to air.

A 22-year-old woman presents to the hospital with severe abdominal pain, abdominal distention, and ileus, along with peripheral neuropathy. Her boyfriend notes that she has been acting strange lately, and that she "seems like a different person." Which of the following enzymes is deficient in this patient? (A) Adenosine deaminase (B) Homogentisic acid oxidase (C) Lysosomal a-1,4-glucosidase (D) Ornithine transcarbamylase (E) Porphobilinogen deaminase

*The answer is B.* This patient most likely has homocystinuria. Clinical findings associated with this condition include intellectual disability, marfanoid habitus, and lens subluxation. In addition, untreated homocystinuria increases a patient's risk of experiencing thromboembolic events in the third decade of life. In fact, almost one-third of patients with untreated homocystinuria die of thrombotic complications by age 30 years. There are three forms of homocystinuria: (1) cystathionine synthase deficiency, (2) decreased affinity of cystathionine synthase for pyridoxal phosphate and (3) methionine synthase deficiency. Cystathionine synthase, which converts homocysteine to cystathione, uses vitamin B6 as a cofactor.

A 24-year-old, intellectually disabled woman presents to the emergency department with an ECG-confirmed acute myocardial infarction. She has never seen a physician previously. Physical examination reveals a tall, lanky young woman whose lenses are displaced downwardly and inwardly, Laboratory tests show a deficiency of a particular enzyme that uses vitamin B6 as a cofactor. What is most likely deficient in this patient? A. Adenosine deaminase B. Cystathione synthase C. Glucose-6-phosphate dehydrogenase D. Methionine synthase E. Phenylalanine hydroxylase F. Pyruvate carboxylase G. Tyrosinase

*The answer is E.* The ubiquitin proteasome pathway (UPP} is essential for breakdown of intracellular proteins, both native and foreign, and helps recycle them into the amino acid building blocks. Ubiquitin functions as a tag that is attached to proteins to mark them for destruction. This process is performed by ubiquitin ligases, enzymes that recognize specific protein substrates and catalyze ubiquitin attachment. These tagged proteins are then taken up by the proteasome. where they are broken down into their constituent oligopeptides and, eventually, amino acids. The role of the UPP in the immune response is related to its ability to degrade foreign Intracellular proteins, such as viral particles. These proteins are degraded to an appropriate size. coupled to major histocompatibility class I protein complex in the endoplasmic reticulum, and then presented on the cell surface for recognition by cytotoxic CDS lymphocytes. Once the cytotoxic lymphocytes recognize non-native (eg, viral) proteins on infected cells, the presenting cells are destroyed as part of the immune response.

A 26-year-old man develops myalgia, nasal congestion, and cough. His temperature is 38.3 C (101 F). Examination shows nasal and pharyngeal hyperemia. Rapid influenza antigen testing of his nasopharyngeal secretions is positive. In the infected cells of the respiratory tract, viral proteins are degraded and attached to major histocompatibility I molecules that are then expressed on the cell surface for presentation to cytotoxic CD8+ lymphocytes. Which of the following enzymes is most likely involved in this process? (A) Acid phosphatase (B) Caspase (C) Guanylate cyclase (D) Myeloperoxidase (E) Ubiquitin ligase

*The answer is E.* Acute intermittent porphyria (AIP) - the condition described in the vignette- and porphyria cutanea tarda are the two most common porphyrias seen clinically. AlP results from a defect in the enzyme porphobilinogen (PBG) deaminase, also called uroporphyrinogen I synthase. PBG deaminase is the third enzyme in the heme synthetic pathway (illustrated here), and its absence leads to an aberrant accumulation of aminolevulinate and PBG. The *5P's* mnemonic for AlP *P*ainful abdomen, *P*or wine-colored urine, *P*olyneuropathy, *P*sychological disturbance, and *P*recipitated by drugs (alcohol and starvation) is a good method for remembering the clinical manifestations. In this case, the neurovisceral signs and symptoms that most correlate with AlP include autonomic neuropathies that produce constipation, abdominal pain, vomiting, hypertension, tachycardia, muscle weakness, as well as psychiatric manifestations such as anxiety, paranoia, depression and high PBG levels in the urine.

A 27-year-old man presents to the Emergency Department in an agitated state. He initially complains of severe abdominal pain, but he soon becomes paranoid and combative, requiring 5-point restraint. His vital signs show elevated blood pressure and tachycardia. When a straight catheter is inserted, reddish urine flows into the Foley bag. The urine is negative for RBCs, and a toxicity screen result is negative. The doctor suspects a porphyria. Laboratory tests for urine porphobilinogen are positive. Which of the following enzyme deficiencies is most likely responsible for this patient's disorder? A. Aminolevulinate dehydratase B. Aminolevulinate synthase C. Ferrochelatase D. Heme oxygenase E. Porphobilinogen deaminase F. Uroporphyrinogen decarboxylase G. Uroporphyrinogen III cosynthase

*The answer is C.* This patient has Gilbert syndrome, a mild impairment of bilirubin conjugation due to low levels of uridine diphosphate glucuronyltransferase. This is a genetic syndrome caused by a mutation in the promoter region of the UGT1A1 gene. This mutation reduces the efficiency in transcription and, thus, reduces levels of the enzyme. A decrease in conjugation may lead to a small increase in indirect (unconjugated) bilirubin, as is seen here (general, total bilirubin should not be more than 5 mg/dL). Importantly, in general, Gilbert syndrome is not clinically relevant, as patients do fine with their decreased uridine diphosphate glucuronyltransferase levels (in contrast to Crigler Najjar, where the enzyme is absent), In times of stress (like fasting), however, an increase in bilirubin may overwhelm the catabolic pathway, leading to mild icterus with no clinical risks. Fasting may lead to increased bilirubin because of its release from adipocytes.

A 27-year-old medical student with no significant medical history appears "slightly yellow." The patient describes having a single evening of "food poisoning" that left him without an appetite for the past 3 days (eating only a single piece of toast each day). Results of physical examination are unremarkable except for mild icterus. Laboratory tests reveal hematocrit of 40, RBC count of 5.0 x 1012/ L, mean corpuscular volume of 95 fL, reticulocyte index of 1.5%, lactate dehydrogenase of 100 U/L, and blood smear without abnormalities; liver function testing reveals aspartate aminotransferase of 35 IU/L, alanine aminotransferase of 30 IU/L, and alkaline phosphatase of 40 IU/L. Total bilirubin is mildly elevated at 2.5 mg/dl (direct is 0.2 mg/dL; indirect is elevated at 2.3 mg/dl). What is the primary defect in heme catabolism that led to this patient's condition? A. Decreased bilirubin excretion by the liver B. Decreased binding to albumin C. Decreased production of bilirubin conjugation enzymes D. Impaired function of heme oxygenase E. Reduced affinity for bilirubin by conjugation enzymes

*The answer is C.* This patient has a classic case of Lesch-Nyhan syndrome, an X-linked recessive disorder due to near-complete deficiency of the purine salvage enzyme hypoxanthine-guanine phosphoribosyl transferase (HPRT, or sometimes referred to as HGPRT). This enzyme recycles purine bases back into nucleotide synthesis and thereby prevents their conversion to uric acid. The deficiency of HPRT is associated with excessive de novo purine synthesis, hyperuricemia (excessive monosodium urate crystal deposition) and the clinical signs and symptoms described in the question stem. The biochemical basis of the often striking self-mutilatory behavior. which may require restraints and even tooth extraction, has never been established. Treatment with allopurinol inhibits xanthine oxidase and reduces gouty arthritis, urate stone formation. and urate nephropathy. lt does not however, modify the neurologic/psychiatric presentation.

A 3-month-old boy has delayed motor development. Nine months later, he develops spasticity and writhing movements. Two years later, he exhibits compulsive biting of fingers and lips and constantly bangs his head. By age 14, he develops arthritis and by age 20, he develops renal failure. Which of the following is most likely to be increased in serum in this patient? A. Galactitol B. Glucocerebrosidase C. Monosodium urate D. Orotic acid E. Phenylalanine

*The answer is A.* Regulation of thyroid hormone secretion involves feedback inhibition along the hypothalamus-pituitary-thyroid axis. The hypothalamus releases thyrotropin-releasing hormone (TRH), which triggers release of thyroid stimulating hormone (TSH, thyrotropin) from the pituitary. TSH stimulates release of thyroid hormone from the thyroid, which in turn inhibits release of both TRH and TSH. Thyroid hormone exists in 2 active forms, T4 (thyroxine), the primary form secreted by the thyroid, and T3 (triiodothyronine), the more active form produced primarily by deiodination of T4 in peripheral tissues. Hypothyroidism may occur due to dysfunction involving the thyroid gland (primary hypothyroidism) or, less commonly, due to disorders of the pituitary (secondary) or hypothalamus (tertiary). This patient, with fatigue, cold intolerance, and a diffusely enlarged thyroid (goiter), has common features of hypothyroidism. In the United States, the most common cause of hypothyroidism is Hashimoto thyroiditis, which is characterized by autoimmune destruction of the thyroid gland. As this autoimmune destruction of the thyroid progresses, thyroid hormone production declines. This leads to loss of feedback inhibition of TSH secretion, with low T4 and elevated TSH (Choices B and C). Because most T3 is produced in peripheral tissues under the control of multiple factors, T3 levels usually remain normal until relatively late-stage hypothyroidism (Choice H). (Choice D) Thyroid hormones circulate mostly in protein-bound form, primarily to thyroxine-binding globulin (TBG), transthyretin, and albumin. When TBG levels are elevated (eg, pregnancy or oral contraceptive use), total thyroid hormone levels are high but the free hormone levels are normal. These patients are euthyroid and have normal TSH. (Choice E) Elevated levels of TSH, T4, and T3 are consistent with hyperthyroidism due to a TSH-secreting pituitary adenoma. (Choice F) Secondary hypothyroidism due to pituitary failure is characterized by decreased levels of both thyroid hormones and TSH. This is significantly less common than primary hypothyroidism and usually associated with loss of other pituitary hormones. (Choice G) Suppressed TSH with elevated thyroid hormone levels is characteristic of thyrotoxicosis (eg, Graves disease). This patient has no clinical features of hyperthyroidism. *Educational Objective:* Primary hypothyroidism is characterized by decreased thyroxine (TO) levels and increased thyroid-stimulating hormone (TSH). Triiodothyronine (T3) is primarily produced by conversion from T4 in peripheral tissues, and levels may be normal in early hypothyroidism.

A 30-year-old woman comes to the office with a 3-week history of mild fatigue, cold intolerance, and constipation. The patient has also experienced dry skin, weight gain, and myalgias. Her mother had similar symptoms and was diagnosed with a thyroid disorder at a similar age. The patient has no history of significant past medical problems and takes no medications. She has been pregnant once and gave birth to a healthy infant 3 years ago. Examination reveals dry skin, delayed ankle jerk reflexes. and bradycardia. Her thyroid gland shows mild diffuse enlargement, with no tenderness or nodules. Laboratory evaluation of this patient is likely to show which of the following patterns?

*The answer is A.* Severe combined immunodeficiency (SCID) may be a result of a variety of genetic mutations, and one of the most common is adenosine deaminase (ADA) deficiency. Patients with ADA deficiency are prone to Candida infections (shown in the image) as well as Pneumocystis pneumonia. The ADA enzyme is necessary for purine nucleotide breakdown. Lack of ADA leads to excessive levels of deoxyadenosine triphosphate, which then inhibits production of other nucleotides. This is toxic to rapidly proliferating cells (eg, developing lymphocytes), and thus patients develop profound T and B lymphocyte immunodeficiencies. In the past, patients with SCID disorders required complete isolation ("boy in the bubble") from the outside environment in order to survive. Currently, the treatment for SCID is bone marrow transplantation.

A 3-month-old boy is brought to the pediatrician by his father. The child has had a persistent cough, tachypnea, and hypoxia. Further work-up reveals the patient's symptoms are due to underlying Pneumocystis jirovecii infection. The father also reports that the boy has had a persistent diaper rash (a culture of the causative organism is shown in the image). The child lacks lymphoid tissue. Radiologic tests are significant for a lack of a thymic shadow, and laboratory values show decreased B and T lymphocytes. Which of the following describes the pathogenesis of the condition likely present in this child? A. Adenosine deaminase deficiency B. Mutations in the gene encoding Bruton tyrosine kinase C. Mutations in the T lymphocyte CD40L protein D. Reduced nicotinamide adenine dinucleotide phosphate oxidase E. Upregulation of de novo purine synthesis

*The answer is C.* The boy has the inherited disorder tyrosinemia type I, which is a defect in fumarylacetoacetate hydrolase, the last step in the degradation pathway for tyrosine. In its acute form, this disorder will lead to liver failure and death within 1 year of life. The accumulation of intermediates in the tyrosine degradation pathway triggers apoptosis of the hepatocytes, leading to complete liver failure. The yellowing of the eyes (jaundice, due to accumulated bilirubin) is a result of liver failure. None of the other amino acids listed (alanine, tryptophan, histidine, and lysine) contribute to the formation of intermediates of the phenylalanine and tyrosine degradative pathways.

A 3-month-old boy of French-Canadian ancestry is seen by the pediatrician for failure to thrive and poor appetite. Physical exam denotes hepatomegaly and a yellowing of the eyes. The boy had been vomiting and had diarrhea, and a distinct cabbagelike odor was apparent. This disorder is due to a defect in the metabolism of which of the following amino acids? (A) Alanine (B) Tryptophan (C) Tyrosine (D) Histidine (E) Lysine

*The answer is B.* The clinical vignette strongly suggests phenylketonuria (PKU). PKU results most commonly from a deficiency of phenylalanine hydroxylase, the first enzyme in the breakdown pathway for phenylalanine. The product of this reaction is tyrosine, a nonessential amino acid in healthy patients. Infants with PKU have inappropriately high blood phenylalanine levels. High phenylalanine levels are neurotoxic, so this may lead to intellectual disability if untreated. Other symptoms include hypopigmentation (due to impaired melanin synthesis), musty odor. eczema (as shown in the image), and hyperreflexia. Treatment includes a diet low in phenylalanine with tyrosine supplementation (since it is now essential). It is most effective if instituted before the child is 3 weeks old. Most infants in North America are currently routinely screened for PKU.

A 3-month-old child is brought to his pediatrician's office for a check-up, The child has a rather pale complexion. He has a social smile but does not hold his head up on his own or make noises. Brisk reflexes are noted in the upper and lower extremities, and he has a rash, as shown in the image. The mother reports that she has to change his clothes several times daily because of a strange odor. What is the most appropriate treatment for the condition the infant most likely has? A. A diet low in isoleucine and leucine B. A diet low in phenylalanine C. A diet low in tyrosine D. A high-protein diet E. Recombinant enzyme therapy

*The answer is B.* The clinical vignette strongly suggests phenylketonuria (PKU). PKU results most commonly from a deficiency of phenylalanine hydroxylase, the first enzyme in the breakdown pathway for phenylalanine. The product of this reaction is tyrosine, a nonessential amino acid in healthy patients. Infants with PKU have inappropriately high blood phenylalanine levels. High phenylalanine levels are neurotoxic, so this may lead to mental retardation if untreated. Other symptoms include hypopigmentation (due to impaired melanin synthesis), musty odor, eczema, and hyperreflexia. Treatment includes a diet low in phenylalanine with tyrosine supplementation (since it is now essential). It is most effective if instituted before the child is 3 weeks old. Most infants in North America are currently routinely screened for PKU.

A 3-month-old child is brought to his pediatrician's office for a check-up. On examination, the physician notices that he has a social smile, but does not hold his head up on his own or make noises. The infant also has pale skin, eczema, odd odor, and hyperreflexia. What is the most appropriate treatment for the condition the infant most likely has? (A) A diet low in isoleucine and leucine (B) A diet low in phenylalanine (C) A diet low in tyrosine (D) A high-protein diet (E) Recombinant enzyme therapy

*The answer is C.* This child suffers from severe combined immunodeficiency (SCID ). In 3 months, she has been diagnosed with a number of severe infections with unusual pathogens that commonly affect immunocompromised individuals, including protozoal diarrhea, Pneumocystis jirovecii pneumonia, and oral Candida albicans. She also demonstrates failure to thrive, as evidenced by her growth charts. SCID has a variety of genetic causes; however, approximately 20% of cases (1 in 200,000 live births) are attributed to a deficiency of adenosine deaminase. The enzyme is a component of the purine salvage system, and its absence ultimately leads to an inability to complete DNA synthesis in B and T lymphocytes. The lack of functional B and T cells leads to an impaired immune response and, subsequently, to the pathophysiology seen in this disease.

A 3-month-old girl is rushed to the emergency department in respiratory distress. Since birth, she has suffered from chronic diarrhea that is culture positive for Isospora. She has had multiple hospitalizations over the past few months, including one for a severe bout of pneumonia with Pneumocystis jirovecii and another for respiratory syncytial virus. On admission, she is noted to be below the fifth percentile for weight and length and appears pale and lethargic. No facial abnormalities are noted. Her pulse is 150/min, respiratory rate is 50/min, and the temperature is 38.9°C (102°F). Her mouth is coated with whitish bumps that scrape off easily. No murmurs are detected on cardiac auscultation. This patient is most likely to have which of the following defects? A. Defect in B-cell maturation B. Defect in B-cell tyrosine kinase C. Deficiency of adenosine deaminase D. Faulty development of the pharyngeal pouches E. Impaired polymerization of microtubules

*The answer is C.* The infant has a defect in the urea cycle, resulting from ornithine transcarbamylase (OTC) deficiency. OTC deficiency would result in decreased intermediates of the urea cycle, including decreased urea formation as indicated by the decreased BUN. OTC can be diagnosed by elevated orotic acid since carbamyl phosphate accumulates in the liver mitochondria and spills into the cytoplasm entering the pyrimidine-synthesis pathway.

A 3-week-old infant has been having intermittent vomiting and convulsions. She also has had episodes of screaming and hyperventilation. The infant has been lethargic between episodes. Tests reveal an expanded abdomen, and blood values show decreased citrulline amounts as well as a decreased BUN. What other clinical outcomes would be expected in this infant? A. Decreased blood pH and uric acid crystals in urine B. Decreased blood pH and increased lactic acid in blood C. Increased blood glutamine and increased orotic acid in urine D. Increased blood ammonia and increased urea in urine E. Megaloblastic anemia and increased methylmalonic acid in blood

*The answer is E.* This child is presenting with signs and symptoms of lead poisoning. In the United States, many sources of lead poisoning (eg, leaded gasoline) have been eliminated, but lead-containing paint in older homes remains a major source of toxicity. Children who ingest paint chips or inhale paint dust may have toxic levels of lead in their blood, causing anorexia, hyperirritability, altered sleep patterns, and decreased play activity. Loss of developmental milestones, especially in speech, may be seen. Abdominal complaints such as intermittent vomiting, pain, and constipation may be the first signs of lead poisoning, which can then progress to ataxia, altered state of consciousness, coma, seizures, and encephalopathy. Permanent, long-term consequences of lead poisoning include learning and cognitive deficits and aggressive behavior. The mechanism of lead toxicity involves the inhibition of several enzymes involved in heme synthesis, including the enzyme aminolevulinic acid dehydratase. This results in the accumulation of various products in the heme synthesis pathway, causing the patient to develop a lead-induced porphyria.

A 3-year-old boy is brought to his pediatrician after complaining of abdominal pain to his mother for the past 3 weeks. He has had intermittent episodes of vomiting and constipation during this time period, and his mother has also noticed that he has become more emotionally labile and his speech seems to be regressing. The child has had no health or behavioral problems in the past. The family recently moved to the area and is currently living in a home undergoing renovation. What is the most likely biochemical disturbances account ing for the symptoms described in this patient? A. β-Glucocerebrosidase deficiency B. Depletion of glutathione stores 1C. Glucose-6-phosphatase deficiency D. Hexosaminidase A deficiency E. Inhibition of aminolevulinic acid dehydratase

*The answer is E.* Orotic acid and orotidine are intermediates in pyrimidine synthesis, Deficiency of orotidine phosphoribosyltransferase or of orotidine decarboxylase (both activities reside on the same protein) interferes with the production of all pyrimidine nucleotides-hence the megaloblastosis (because RNA and DNA synthesis is slow) Orotidine-5' -phosphate decarboxylase deficiency results in orotic acid (its substrate) buildup. In addition to orotic aciduria, orotic acid crystals form in the urinary tract, causing uropathy. Microcytic. hypochromic red blood cells may also be seen because hemoglobin levels decrease without sufficient gene expression, The first enzyme in the heme pathway δ-aminolevulinate synthase, has a short half-life (60 minutes) and must be continually synthesized to produce sufficient heme. Administration of uridine corrects the problems because, after phosphorylation in the cell, it replaces the product of the deficient reactions.

A 3-year-old boy is diagnosed with megalbolastosis and a hypochromic anemia. His hemoglobin is 4.7 g/dL. Treatment with vitamin B12. folate, pyridoxine, and iron is begun, but there is no improvement in his symptoms. Urinalysis shows white needle-shaped crystals. Treatment is begun with uridine, which corrects the megaloblastosis, returns hemoglobin levels to normal, and decreases the excretion of the urinary crystals. Which of the following enzymes is most likely deficient in this patient? A. Aspartate carbamoyltransferase B. Cytoplasmic carbamoyl-phosphate synthase C. Mitochondrial carbamoyl-phosphate synthase D. Ornithine carbamoyltransferase E. Orotidine 5'-phosphate decarboxylase

*The answer is D.* This patient's severe intellectual disability, history of seizures, and abnormal pallor of catecholaminergic brain nuclei on autopsy are suggestive of phenylketonuria (PKU). PKU results from the inability to convert phenylalanine into tyrosine, a reaction which is normally catalyzed by phenylalanine hydroxylase. This enzyme requires the cofactor tetrahydrobiopterin (BH). Which is regenerated from dihydrobiopterin (BH) by the enzyme dihydropteridine reductase. Although neonatal hyperphenylalaninemia can be caused by deficiency of either enzyme, most cases are attributable to abnormalities in phenylalanine hydroxylase. It is believed that excess phenylalanine and the presence of large concentrations of phenylalanine metabolites (eg. Phenyllactate & phenylacetate) contribute to the brain damage seen in PKU. Hypopigmentation involving the skin, hair, eyes, and catecholaminergic brain nuclei (which produce a dark pigment known as neuromelanin) results from the inhibitory effect of excess phenylalanine on melanin synthesis. The classic musty or mousy body odor is due to the accumulation of abnormal phenylalanine metabolites.

A 3-year-old boy who recently immigrated to the United States is brought to the physician by his parents because he has not yet begun to walk or speak. Assessment of his developmental milestones shows severe intellectual disability. He dies 6 months later from refractory seizures resulting in respiratory failure. Autopsy shows pallor of the substantia nigra, locus ceruleus, and vagal nucleus dorsalis. The underlying condition responsible for this patient's death is most likely caused by a deficiency of which of the following enzymes? (A) Branched-chain ketoacid dehydrogenase (B) Dopamine hydroxylase (C) Homogentisic acid oxidase (D) Phenylalanine hydroxylase (E) Tyrosinase

*The answer is B.* The child has nonclassical PKU, which is due to a defect in the biosynthesis of tetrahydrobiopterin. Tetrahydrobiopterin is required for ring hydroxylation reactions, such as the conversion of phenylalanine to tyrosine, tyrosine to dopa, and tryptophan to serotonin. However, tetrahydrobiopterin is not required for the conversion of dopa to melanin, serotonin to melatonin, and norepinephrine to epinephrine, which is a methylation reaction. A deficiency of tetrahydrobiopterin would cause phenylalanine to be converted to phenylketones rather than to tyrosine.

A 3-year-old child from Russia, after immigrating to the United States, was found to have developmental delays and severe mental retardation. Elevated levels of phenylalanine and phenylpyruvate were found in the blood. The child was placed on a low-phenylalanine diet, but there was no improvement in the child. Given this information, the child would be expected to have difficulty in undergoing which one of the following conversions? (A) Phenylalanine to phenylketones (B) Tyrosine to dopamine (C) Dopa to melanin (D) Serotonin to melatonin (E) Norepinephrine to epinephrine

*The answer is D.* The child exhibits the symptoms of Lesch-Nyhan syndrome, which is a deficiency of HGPRT activity. HGPRT will convert the free bases hypoxanthine, or guanine, plus PRPP (5′-phosphoribosyl 1′ pyrophosphate) to the nucleotides IMP, or GMP, plus pyrophosphate. The reactions in answer choices A and C are part of the pyrimidine salvage pathways, using pyrimidine nucleoside phosphorylase, in which a nucleoside is formed from the free base and (deoxy)ribose 1-phosphate. The reaction in E is the IMP dehydrogenase step, the first step on the pathway to de novo GMP production. The reaction in the choice B is catalyzed by adenine phosphoribosyltransferase, and is analogous to the HGPRT reaction, other than adenine is the substrate, and not hypoxanthine or guanine.

A 3-year-old child has mental retardation, poor muscle control, gout, chronic renal failure, facial grimacing, and lip and finger biting. This child has an inability to catalyze which one of the following reactions? (A) Thymine + deoxyribose 1-phosphate yields deoxythymidine + inorganic phosphate (B) Adenine + 5′-phosphoribosyl 1′-pyrophosphate yields AMP + pyrophosphate (C) Uracil + ribose 1-phosphate yields uridine + phosphate (D) Guanine + 5′phosphoribosyl 1′-pyrophosphate yields GMP + pyrophosphate (E) IMP + NAD1 yields XMP + NADH

*The answer is D.* Toxicities associated with lead poisoning begin at blood lead levels of only 10 pg/dl. Toxic effects include abdominal pain, peripheral neuropathy, very thin, black-blue lines along the margin of the gums at the base of the teeth (Burton lines), cognitive impairment, and basophilic stippling (see circles in image). Basophilic stippling is not specific for lead toxicity and may also be observed in arsenic toxicity, sideroblastic anemia, and thalassemia

A 3-year-old girl presents to the emergency department with 2 weeks of abdominal pain. The mother denies any nausea, vomiting, or fever in the child. The child says that her hands feel "funny," and she apparently has been stumbling while walking more frequently over the past few months. Serum laboratory tests are normal. The child's peripheral blood smear shown in the image, reveals stippling of RBCs. Poisoning with which of the following substances is most likely? A. Arsenic B. Copper C. Iron D. Lead E. Mercury

*The answer is E.* Tetrahydrobiopterin is a naturally occurring molecule that can be synthesized from GTP in humans. It is a cofactor for phenylalanine hydroxylase, which is the rate-limiting enzyme in the conversion of phenylalanine to tyrosine. This child has phenylketonuria (PKU) due to a defect in tetrahydrobiopterin synthesis, which prevents her from converting phenylalanine to tyrosine. PKU caused by tetrahydrobiopterin deficiency can be treated with supplementation with tetrahydrobiopterin in addition to a phenylalanine-free diet, high-tyrosine diet. Phenylalanine leads to a build-up of phenylketones. Children with untreated PKU display intellectual disability and growth retardation, fair skin, eczema, and a musty body odor.

A 3-year-old girl who was born at home in a rural area is brought to the local clinic because of seizures. She is pale-skinned, shows marked intellectual disability, and has a musty body odor. The doctor diagnoses her with a defect in tetrahydrobiopterin metabolism. Which of the following is a component of the correct therapeutic measure for this child? A. Branched-chain amino acid-free diet B. Fructose-free diet C. Ketogenic diet D. Low-tyrosine diet E. Phenylalanine-free diet

*The answer is F.* The patient described in the question stem exhibits features of arginase deficiency. Arginase is an enzyme of the urea cycle that produces urea and ornithine from arginine. Arginase deficiency is likely underdiagnosed because the spasticity seen commonly in this disorder may simple be attributed to cerebral palsy. Treatment of arginase deficiency consists of a low-protein diet that is devoid of arginine. Administration of a synthetic protein made os essential amino acids usually results in a dramatic decrease in plasma arginine concentration and an improvement in neurologic abnormalities.

A 3-year-old male is hospitalized with progressive spastic paresis of his lower extremities and choreoathetoid movements. Comprehensive laboratory testing reveals very high arginine levels in both the child's plasma and cerebrospinal fluid. The enzyme deficient in this patient is normally involved in the production of which of the following? (A) Serotonin (B) Glutamine (C) Orotic acid (D) Homocysteine (E) γ-aminobutyric acid (F) Urea

*The answer is B.* One of the systemic drugs given for psoriasis is methotrexate, which inhibits dihydrofolate reductase. The rationale behind using this drug is to kill the skin cells that are giving rise to the condition, thereby alleviating the symptoms. Methotrexate inhibits dihydrofolate reductase, preventing FH2 (which is produced in the thymidylate synthase reaction) from being converted to FH4. This leads to a functional loss of folic acid, and cells treated with methotrexate will not be able to synthesize dTMP from dUMP, or perform de novo synthesis of the purine ring. Since dTMP cannot be made, dTTP will not accumulate, nor will dATP (in the absence of dTTP, ribonucleotide reductase cannot generate dGTP, which means that dATP will also not be made). The lack of FH4 production from FH2 means that FH4 levels will be low, along with N5-methyl-FH4 levels.

A 30-year-old female with severe psoriasis has been nonresponsive to topical steroid creams, coal tars, and UV light, but is controlled well on a small dose of medication she takes orally once a week. This systemic treatment leads to an accumulation of which one of the following? (A) FH4 (B) FH2 (C) N5-methyl-FH4 (D) dTTP (E) dATP

*The answer is C.* The man is experiencing gout attacks, due to a buildup of uric acid. Uric acid is the end product of purine degradation. The liver contains larger levels of nucleic acids (DNA, RNA) than do the other foods listed as answers, and the intestinal epithelial cells will convert the purines in the food to uric acid, and release it into the circulation. The alcohol the man has consumed leads to dehydration, which raises the uric acid concentration to the point where it will precipitate in the blood, leading to the painful episodes.

A 30-year-old male has had multiple episodes of sudden, severe pain, redness, and swelling of metatarsophalangeal joint of his great toes. These problems seem to occur after the man has had a night out on the town with his friends, when they go barhopping, and the night usually ends with a cab ride home for the group. This problem would also be exacerbated if the man eats which one of the following during his night out? (A) Hamburger (B) Chicago hot dog (C) Chopped liver (D) Nachos and salsa (E) Chicken wings

*The answer is E.* This patient suffers from the autosomal recessive disorder alkaptonuria, which is caused by a deficiency in the enzyme homogentisic acid oxidase. As a result of the deficiency, there is an accumulation of homogentisic acid, leading to endogenous ochronosis. The first clinical manifestation of this disorder may occur in infancy and may manifest with black staining of diapers. Excess homogentisic acid is excreted in urine, impa1ting a black color to urine when oxidized. Oxidation of urine occurs at an alkaline pH, which may classically be described to occur when the urine sample is left to stand for some time. Affected patients are usually asymptomatic until middle adulthood when patients demonstrate blue-black discoloration (demonstrated in the images) that is most apparent in regions of mucosa and skin that are thin due to the dermal deposition of pigment. These regions include the sclera, axillary skin, tympanic eardrum, genital skin, the nail beds, face and the cartilage of extensor joints, ears, and the nasal tip. Ochronotic arthritis develops even later in age and occurs because excess homogentisic acid leads to inhibition of the enzyme lysyl hydroxylase, which is crucial for collagen cross-linking. Homogentisic acid is an intermediate product in the metabolism of phenylalanine and tyrosine and cannot be further metabolized without functional homogentisic acid oxidase. In patients with alkaptonuria, a diet high in tyrosine or L-phenylalanine may lead to a build-up of homogentisic acid with subsequent worsening of their condition.

A 31-year-old white man presents to his primary care physician for a health maintenance visit. His medical history is notable for low back and right shoulder pain treated with NSAIDs. He is concerned because over the last few years he has noticed progressive darkening of his ears and eyes. His examination is remarkable for brownish-gray pigmentation of his sclerae and pinnae. There is no warmth or joint erythema, but he does have stiffness and limited range of motion of his shoulders and lumbar spine. Excess intake of which of the following may lead to worsening of this patient's condition? A. Cystine B. Fat-soluble vitamins C. Homocysteine D. Triglycerides E. Tyrosine

*The answer is A.* The patient has acute intermittent porphyria, which is a defect in one of the early steps leading to heme synthesis. The buildup of the intermediate that cannot continue along the pathway leads to the dark urine, and it turns darker when UV light interacts with the conjugated double bonds in the molecule. Erythromycin is metabolized through an induced P450 system, which requires increased heme synthesis. This leads to metabolite buildup to the level where the abdominal pain appears. The defect in heme synthesis does not affect creatine phosphate, cysteine, thymine, or methionine levels.

A 34-year-old female has a history of intermittent episodes of severe abdominal pain. She has had multiple abdominal surgeries and exploratory procedures with no abnormal findings. Her urine appears dark during an attack and gets even darker if exposed to sunlight. The attacks seem to peak after she takes erythromycin, due to her penicillin allergy. This patient most likely has difficulty in synthesizing which of the following? (A) Heme (B) Creatine phosphate (C) Cysteine (D) Thymine (E) Methionine

*The answer is B.* The patient in this vignette has hepatic encephalopathy. He has a history of alcoholism and a physical exam that shows multiple stigmata of chronic liver disease including ascites, asterixis, palmar erythema, and spider hemangiomas. There is no sign of infection with a normal white blood cell count and the toxicology screens are negative. In patients with hepatic encephalopathy, ammonia is often elevated. Ammonia is metabolized and excreted via the urea cycle. Ammonia combines with bicarbonate end phosphate to form carbamoyl phosphate via the enzyme carbamoyl phosphate synthetase and its cofactor, N-acetylglutamate.

A 34-year-old homeless man with a history of alcoholism is brought into the emergency department. He was found sleeping on a park bench by the police and is unable to provide any information. His vital signs include temperature 370C (98.60F), heart rate 105 beats/min, blood pressure 100/68 mmHg, respiratory rate 18 breaths/min, and oxygen saturation is 100% on room air. On physical exam, he is awake and alert, but only oriented only to self and unable to answer any questions. His heart is tachycardic with a normal rhythm and there are crackles in the bases of his lungs bilaterally. His abdomen is distended with a positive fluid wave. He has positive asterixis. He has erythema of his palms and spider angiomas on his chest. There are multiple bruises and abrasions on his extremities. A white blood cell count is normal. Serum and urine toxicology screens are negative. Which of the following is the most immediate product in the normal metabolism of the substance that is likely to be elevated in this patients serum? A. N-acetylslutamate B. Carbamoyl phosphate C. Arginine D. Urea E. Argininosuccinate F. Citrulline

*The answer is C.* This patient has a resolving hematoma after a traumatic injury. Following the injury, hemoglobin-containing erythrocytes escape into the periorbital tissues, giving the bruise its initial purple or bluish color. Erythrocyte destruction causes the release of iron-containing heme molecules. Heme oxygenase (contained in macrophages, among other calls) degrades heme into biliverdin, carbon monoxide, and ferric iron while consuming oxygen and electrons provided by NADH and NADPH-cytochrome P450 reductase. Biliverdin is green in color and is further reduced (by the enzyme biliverdin reductase) to the yellow pigment bilirubin, which is then transported to the liver bound to albumin.

A 34-year-old man comes to the emergency department due to a facial injury. He reports getting hit on the face during a fistfight at a bar. Examination shows dark blue periorbital ecchymosis on the right side. Ophthalmic and neurologic examinations we otherwise normal. After appropriate evaluation, the patient is discharged home. Several days later, the bruise becomes greenish in color. This change in color is best explained by the activity of which of the following enzymes? A. Bilirubin glucuronyl transferase B. Ferrochelatase C. Heme oxygenase D. Porphobilinogen deaminase E. Uroporphyrinogen decarboxylase

*The answer is E.* The defective transporter of neutral amino acids is the hallmark of Hartnup disease, which is a member of the class of diseases known as aminoacidurias. These diseases are characterized by the presence of elevated levels of amino acids in the urine. In Hartnup disease, the defective transporter is expressed in both the renal and gastrointestinal tracts and results in the wasting of the neutral amino acids, with the exception of proline. Patients with Hartnup disease occasionally develop the symptoms of niacin deficiency, or pellagra (as reflected in this patient's neurologic impairment and dermatitis), because of the wasting of tryptophan. Tryptophan can be used to make niacin with vitamin B6 , pyridoxine, as a cofact or.

A 4-year-old boy is brought to the pediatrician by his parents, who are concerned about his delayed development. Physical examination reveals a skin rash, cerebellar ataxia, and nystagmus. Further work-up reveals that the child has a specific vitamin deficiency caused by an autosomal recessive disorder involving the lack of a transporter for neutral amino acids normally found in the small intestine and renal tubules. As a result, his urine has elevated levels of neutral amino acids. The reduction in transport of which amino acid most likely led to this child's vitamin deficiency? A. Cystine B. Leucine C. Lysine D. Phenylalanine E. Threonine F. Tryptophan G. Valine

*The answer is A.* This patient is experiencing a hypertensive emergency (severe hypertension, headache, blurry vision) and has signs of excessive sympathetic activity (tachycardia, diaphoresis, tremors), most likely as a result of ingesting tyramlne-containing foods (eg, aged cheeses. cured meats, draft beer). Tyramine is an indirect sympathomimetic that is usually metabolized in the gastrointestinal tract by monoamine oxidase (MAO). This mitochondrial enzyme is also responsible for degradation of monoamine neurotransmitters within the presynaptic nerve terminal. MAO Inhibitors (eg, tranylcypromine, phenelzine) function by increasing synaptic monoamine levels and are used in some patients with atypical or treatment-resistant depression. However, these medications also block the degradation of dietary tyramine, glowing it to enter the systemic circulation and cause severe hypertension and other signs of sympathetic hyperactivity.

A 34-year-old man is brought to the emergency department with severe headache and blurry vision. His symptoms began suddenly after having lunch at a new Italian deli in his neighborhood. The patient says he "ate a sandwich with lots of fancy meats and cheeses" and drank an iced tea. His past medical history is significant for severe, atypical depression. Hs has no known medication or food allergies. His blood pressure is 210/130 mm Hg and heart rate is 110/min. On physical examination, he appears tremulous and diaphoretic. The medication used to treat this patient's depression most likely affects which of the following steps of monoamine neurotransmission? (A) Monoamine degradation (B) Binding to postsynaptic receptors (C) Presynaptic non-selective monoamine uptake (D) Presynaptic selective norepinephrine uptake (E) Presynaptic selective serotonin uptake

*The answer is C.* The patient has alkaptonuria, a condition corresponding to the one described in the vignette. A deficiency of the enzyme homogentisic acid oxidase leads to deposition of homogentisic acid in the joints and cartilage, giving them a dark color (ochronosis) and resulting in degenerative changes. Classically, the urine of these patients turns black on contact with air or when the urine is made alkaline. The associated defect is on chromosome 3.

A 35-year-old man presents to the physician with arthritic pain in both knees along with back pain. He states that the pain has been present for months. In an effort to obtain relief, he has taken only aspirin, but this has been of little benefit. The patient is afebrile, and his slightly swollen knee joints are neither hot nor tender to palpation; however, the pain does restrict his motion. The cartilage of his ears appears slightly darker than normal. No tophi are present. A urine specimen is taken for analysis of uric acid content and turns black in the laboratory while standing. A defect in which of the following is the most likely underlying cause of the patient's condition? A. α-Ketoacid dehydrogenase B. Galactokinase C. Homogentisic acid oxidase D. Orotate phosphoribosyltransferase E. Phenylalanine hydroxylase

*The answer is C.* The patient has alkaptonuria, a condition corresponding to the one described in the stem. A deficiency of the enzyme homogentisic acid oxidase leads to deposition of homogentisic acid in the joints and cartilage, giving them a dark color (ochronosis) and resulting in degenerative changes. Classically, the urine of these patients turns black on contact with air or when the urine is made alkaline. The associated defect is on chromosome 3.

A 35-year-old man presents to the physician with arthritic pain in both knees along with back pain. He states that the pain has been present for months. In an effort to obtain relief, he has taken only aspirin, but this has been of little benefit. The patient is afebrile, and his slightly swollen knee joints are neither hot nor tender to palpation; however, the pain does restrict his motion. The cartilage of his ears appears slightly darker than normal. No tophi are present. A urine specimen is taken for analysis of uric acid content and turns black in the laboratory while standing. A defect in which of the following is the most likely underlying cause of the patient's condition? (A) a-Ketoacid dehydrogenase (B) Galactokinase (C) Homogentisic acid oxidase (D) Orotate phosphoribosyl transferase (E) Phenylalanine hydroxylase

*The answer is B.* Nitric oxide (NO) is a short-acting (about 100 msec) physiologic messenger that is released from endothelial cells and diffuses to a nearby smooth muscle cell, where it enters the cell and activates guanylate cyclase. Guanylate cyclase makes the second messenger cGMP, which triggers smooth-muscle relaxation by facilitating the dephosphorylation of myosin light chains. This prevents the interaction of myosin with actin. This is also the mechanism by which NO causes vasodilation, or the corpus cavernosum of the penis, where it causes penile erection (via vasodilation).

A 35-year-old man who is a marathon runner experiences cutaneous vasodilation to help cool his body while running on a not summer day. Increased blood flow leads to the release of a chemical mediator from endothelial cells causing vasodilation in arterioles. Which of the following enzymes is directly activated by this chemical mediator? A. Adenylate cyclase B. Guanylate cyclase C. Heme oxygenase D. Nitric oxide synthase (NOS) E. Phospholipase C

*The answer is E.* Acute intermittent porphyria (AIP) is one of the most common of the porphyrias. It is an autosomal dominant condition that results from partial deficiency of the enzyme porphobilinogen deaminase (PBGD), which catalyzes the reaction Porphobilinogen (PBG) → Hydroxymethylbilane (HMB) AIP is more commonly seen in women, but the penetrance of the disease is only about 15%. Phenotypically affected individuals rarely have episodes before puberty. PBGD is present in all tissues and red blood cells (RBCs). Most patients with the deficiency are clinically normal except when affected Oy certain environmental factors, drugs (especially agents such as barbiturates and alcohol that induce cytochrome P450), or changes in nutritional or hormonal state. When symptoms do occur (as in the present case), the urine in rare patients may turn a port-wine color due to porphobilin, an oxidation product of PBG, This color change may also be observed in patient urine that has been left to stand and oxidize, but this is rarely done since usually urine in suspected patients is sent for chemical testing. Affected patients excrete delta-aminolevulinic acid (ALA) and PBG, and there is an increase in serum ALA and PBG., Diagnosis usually can be made by showing a decrease in PBGD activity in RBCs, therefore choice B is incorrect.

A 37-year-old man with a history of acute intermittent porphyria comes to the physician because of acute gastroenteritis. He has severe abdominal pain, tachycardia, hypertension, and proximal muscle weakness. Which of the following laboratory findings is most likely to be seen during an episode of this illness? A. increased erythrocyte Zn-protoporphyrin B. increased porphobilinogen deaminase (PGBD) activity in erythrocytes C. increased serum porphyrins D. increased urinary coproporphyrins E. increased urinary delta-aminolevulinic acid (ALA) and porphobilinogen (PBG)

*The answer is A.* Alkaptonuria is a relatively benign childhood disorder that is marked by severe arthritis in adult life. This autos oral-recessive disorder is caused by deficiency of homogentislc acid dioxygenase, which normally metabolizes homogentisic acid into maleylacetoacetate. Accumulated homogentisic acid causes pigment deposits in connective tissue throughout the body. During adulthood, these blue-black deposits become apparent in the sclerae and ear cartilage. Deposits also occur in the large joints and spine, causing ankylosis, motion restriction, and significant pain. A distinctive characteristic of alkaptonuria is that the urine of these patients turns black when exposed to air due to oxidization of homogentisic acid. *Educational Objective:* Alkaptonuria is an autosomal-recessive disorder caused by a deficiency of homogentisic acid dioxygenase. at enzyme involved in tyrosine metabolism. Excess homogentisic acid causes diffuse blue-black deposits in connective tissues. Adults have sclerae arid ear cartilage hyperpigmentation along with osteoarthropathy of the spine and large joints.

A 38-year-old man comes to the office due to pain in multiple joints. He has a 5-year history of lumbar pain and a 2-year history of bilateral knee pain. The patient works in construction and his pain is worst altar a long day on his fest. He has taken ibuprofen intermittently, but the pain is no longer tolerable. The patient has a paternal aunt with osteoarthritis. Physical examination shows blue-black spots on his sclera and diffuse darkening of the auricular helices. Which of the following is the most likely cause of this patient's arthritis? A. Homogentisic acid dioxygenase deficiency B. Hyperuricemia C. Multifactorial articular cartilage failure D. Recent infection with Salmonella E. Tyrosinase deficiency

*The answer is B.* The patient's presentation is consistent with methylmalonic acidemia, an autosomal recessive organic acidemia resulting from a complete or partial deficiency of the enzyme methylmalonyl-CoA mutase. Catabolism of isoleucine, valine, threonine, methionine, and off-chain fatty acids normally leads to the formation of propionyl-CoA, which is then converted to methylmalonyl-CoA by biotin-dependent carboxylation. Isomerization of methylmalonyl-CoA through a vitamin B12-dependent reaction forms succinyl-CoA, which subsequently enters the TCA cycle. Mutations in methylmalonyl-CoA mutase result in buildup of methylmalonic acid and propionic acid, leading to a metabolic acidosis. Hypoglycemia results from overall increased metabolic rate leading to increased glucose utilization and direct toxic inhibition of gluconeogenesis by the organic acids. The presence of hypoglycemia leads to increased free fatty acid metabolism that produces ketones, resulting in a further anion gap metabolic acidosis. Finally, organic acids also directly inhibit the urea cycle, leading to hyperammonemia.

A 4-day-old girl is brought to the office for a routine newborn visit. She was born at 39 weeks gestation via normal spontaneous vaginal delivery to a gravida 5 para 4 woman. The patient has been breastfed exclusively but has had increasing difficulty feeding over the past 24 hours. Her parents say that she is ""too sleepy to feed" and has been vomiting. The infant has 3 healthy living siblings and a brother who died in infancy from "low sugar and acid in his blood." Physical examination shows tachypnea and signs of dehydration. The patient is responsive to painful stimuli only. After acute treatment and stabilization, urine testing reveals significantly elevated levels of methylmalonic acid. Which of the following sets fo laboratory values would most likely result from this patient's condition?

*The answer is E.* This patient's clinical presentation suggests phenylketonuria (PKU). Classic PKU (the most common form) occurs most frequently in persons of Scandinavian descent. Individuals who are homozygous for this autosomal recessive disorder have a severe deficiency of phenylalanine hydroxylase, the enzyme responsible for conversion of phenylalanine to tyrosine. Tyrosine becomes an essential amino acid in these patients as it cannot be synthesized from phenylalanine, although most patients receive adequate amounts in their diet and do not require supplementation. Furthermore, the excess phenylalanine inhibits tyrosinase, which normally results in the downstream production of melanin. The lack of melanin causes the fair complexion seen in this patient. Although development is initially normal, most untreated patients develop intellectual disability by age 6 months. Other classic findings include seizures, eczema, light hair/skin pigmentation, and a "musty," odor. However, cognitive impairment can be lessened by early detection (newborn screening) and restriction of phenylalanine Intake.

A 4-month-old boy is brought to the office for his first visit since arriving in the United States. The patient was recently adopted, and his adoptive mother says the boy is tremulous compared to her biological children. Over the pest week, the boy has also had episodes of upward eye deviation and bilateral arm and leg shaking for approximately 2 minutes at a time. Biological family history is not available. His temperature is 36.7 C (98.1 F), blood pressure is 90/40 mm Hg, pulse is 120/min, and respirations are 30/min. Examination shows a fair-skinned infant with blue eyes and a musty body odor. 'Which of the following amino acids is most likely essential in this patient? A. Cysteine B. Isoleucine C. Leucine D. Phenylalanine E. Tyrosine F. Valine

*The answer is E.* This is an infant with phenylketonuria (PKU), an inborn error of metabolism that results in a deficiency of phenylalanine hydroxylase. This enzyme normally converts phenylalanine into tyrosine. As a result of the deficiency, phenylalanine accumulates, causing intellectual disability, failure to thrive, a musty body odor, and fair skin, among other features. Infants with PKU require a special diet that is rich in tyrosine and does not contain phenylalanine.

A 4-month-old infant from a newly immigrated family is brought to the emergency department by his mother, who is concerned because the child has not been feeding well. The mother has also noticed that her son has an unusual odor, which she describes as "old and musty." She admits that she did not receive any prenatal care but adds that there were no complications during the pregnancy. On physical examination, the infant appears to be extremely pale, and his skin seems dry and flaky. He also seems lethargic. The patient's height and weight are at the 40th percentile for his age. Treatment of this patient will most likely include dietary supplementation with which of the following? A. Arginine B. Glutamate C. Isoleucine D. Tryptophan E. Tyrosine F. Valine

*The answer is E.* Orotic aciduria is the buildup of orotic acid due to a deficiency in one or both of the enzymes that convert it to UMP. Either orotate phosphoribosyltransferase and orotidylate decarboxylase are both defective, or the decarboxylase alone is defective. UMP is the precursor of UTP, CTP, and TMP. All of these end products normally act in some way to feedback-inhibit the initial reactions of pyrimidine synthesis. Specifically, the lack of CTP inhibition allows aspartate transcarbamoylase to remain highly active and ultimately results in a buildup of orotic acid and the resultant orotic aciduria. The lack of CTP, TMP, and UTP leads to a decreased erythrocyte formation and megaloblastic anemia. Uridine treatment is effective because uridine can easily be converted to UMP by omnipresent tissue kinases, thus allowing UTP, CTP, and TMP to be synthesized and feedback-inhibit further orotic acid production.

A 4-year-old girl presents in the clinic with megaloblastic anemia and failure to thrive. Blood chemistries reveal orotic aciduria. Enzyme measurements of white blood cells reveal a deficiency of the pyrimidine biosynthesis enzyme orotate phosphoribosyltransferase and abnormally high activity of the enzyme aspartate transcarbamoylase. Which one of the following treatments will reverse all symptoms if carried out chronically? a. Blood transfusion b. White blood cell transfusion c. Dietary supplements of phosphoribosylpyrophosphate (PRPP) d. Oral thymidine e. Oral uridine

*The answer is C.* The child has cystathionine synthase deficiency. also known as homocystinuria. This enzyme occurs in the pathways involving the metabolism of sulfur-containing amino acids. and normally catalyzes the conversion of homocysteine to cystathionine. When the enzyme is blocked. homocysteine (which is normally a degradation product of methionine) is remethylated by a salvage pathway, resulting in the resynthesis of methionine. Plasma methionine and homocysteine levels are elevated as a result. The clinical features illustrated in the question stem are typical and likely to be mistaken for Marfan syndrome. Note however that the history of mental retardation is not consistent with Marfan syndrome. Children with cystathionine synthase deficiencies also have limited joint mobility in contrast to the hyperektensible joints in Marfan syndrome. These patients are also vulnerable to (potentially fatal) thrombotic episodes, particularly in postoperative and postpartumperiods.

A 4-year-old girl with a history of mental retardation is brought to the physician because of poor vision. Physical examination shows long limbs, tall stature, and kyphosis. Funduscopic examination shows bilateral lens dislocations. Laboratory studies are most likely to show a selective elevation of which of the following amino acids in serum? A. Cystathionine B. Cysteine C. Methionine D. Phenylalanine E. Tyrosine

*The answer is A.* Methotrexate is an antimetabolite that inhibits dihydrofolate reductase by acting as a folic acid analog. Adverse effects associated with methotrexate include stomach it is, erythema, rash, urticaria, alopecia, gastrointestinal symptoms, and myelosuppression. Leucovorin (folinic acid) can be used to "rescue" the bone marrow after methotrexate administration. Folinic acid is reduced to methylene tetrahydrofolate, thereby bypassing the inhibited dihydrofolate reductase enzyme and alleviating the myelosuppression.

A 4-year-old white male child with acute lymphocytic leukemia is receiving intrathecal methotrexate to treat neoplastic meningitis. A complete blood cell count with differential indicates leukopenia, anemia, and thrombocytopenia. An oncologist recommends a treatment to reverse the patient's myelosuppression. What is the mechanism of action of this rescue therapy? A. Delivery of folinic acid B. Erythropoietin analog C. Inactivation of the toxic metabolite D. Inhibition of thymidylate synthase E. Inhibition of xanthine oxidase

*The answer is B.* This patient with latent tuberculosis has laboratory values and a bone marrow aspirate consistent with sideroblastic anemia due to isoniazid use. Sideroblastic anemia is diagnosed by bone-marrow examination with Prussian blue stain. Causes include X-linked sideroblastic anemia (due to a δ-aminolevulinate synthase mutation), myelodysplastic syndrome, alcohol abuse, copper deficiency, and certain medications (eg isoniazid, chloramphenicol, linezolid). IsonIazId directly inhibits the enzyme pyridoxine phosphokinase, which normally converts pyridoxine (vitamin B,) to its active form, pyridoxal 5' phosphate. Pyridoxal 5' phosphate is a cofactor for δ-aminolevulinic acid (ALA) synthase, the enzyme that catalyzes the rate-limiting step in heme synthesis. Inhibition of this enzyme produces a microcytic, hypochromic anemia. Iron is transported to developing erythrocytes that cannot form heme, and its granules accumulate circumferentially around the nucleus, forming ring sideroblasts. Because pyridoxal 5' phosphate is B cofactor for numerous enzymes, pyridoxine deficiency can also lead to dermatitis, stomatitis, neuropathy, and confusion. Therefore, pyridoxine is typically prescribed with isoniazid.

A 40-year-old woman comes to the office with a 3-month history of progressive limitation of physical activity due to fatigue. She says, "I could barely walk from my car to your office." Past medical history is significant for a positive tuberculin skin test 7 months ago with a normal chest radiograph. She has been compliant with the prescribed treatment despite its bitter taste. Physical examination shows a tired-appearing woman with conjunctival and palmer pallor. Results of complete blood count are as follows: Hemoglobin 9 g/dL Hematocrit 28% Mean corpuscular volume . 72 fL Bone marrow aspirate revealed the following representative sample under Prussian blue stain. Decreased activity of which of the following enzymes most likely explains the anemia found in this patient? A. δ-aminolevulinate dehydratase B. δ-aminolevulinate synthase C. Cystathionine synthase D. Glucose-6-phosphate dehydrogenase E. Pyruvate kinase

*The answer is D.* The presenting symptoms of this patient are classic for hypothyroidism. The lead-in asking about an amino acid deficiency suggests that the cause of this hypothyroidism is lack of synthesis of thyroid hormone. During the synthesis of thyroid hormone, iodide is first oxidized to iodine. Iodine is then added to the tyrosine residues of thyroglobulin to create monoiodotyrosine (MIT) and diiodotyrosine (DIT). MIT and DIT are then combined to create thyroxine and triiodothyronine, with thyroxine as the dominant product. Because tyrosine residues are needed to create thyroid hormone, a deficiency in phenylalanine could theoretically cause a lack of thyroid hormone production because phenylalanine is a precursor to tyrosine. Phenylalanine and tyrosine are also precursors to dopa, melanin, dopamine, norepinephrine, and epinephrine.

A 42-year-old female presents to the physician with lethargy, cold intolerance, and a 15-lb weight gain over the past 3 months, despite concerted efforts to cut down her calorie intake. If her symptoms were secondary to an essential amino acid deficiency, which amino acid would be deficient? A. Arginine B. Glycine C. Histidine D. Phenylalanine E. Tryptophan

*The answer is C.* The patient is displaying megaloblastic anemia due to a deficiency of vitamin B12. The use of omeprazole to reduce acid production in the stomach also reduces the ability of B12, bound to ingested proteins, to be released by the proteins to be bound by intrinsic factor for effective absorption into the blood. Providing injections of B12 will bypass the need for separation of B12 from its binding proteins and will allow B12 to circulate throughout the body and reach its intracellular targets and proteins. Oral B12 would also work under these conditions. The patient is unlikely to have an intrinsic factor problem (due to his age), and intrinsic factor cannot be given orally or via injection (since it needs to work in the intestine). For lack of intrinsic factor, injections of B12 are also required. The patient has normal folate levels, so giving more folate will not help the anemia, and vitamin B6 is not involved in these reactions.

A 42-year-old male has fatigue, pale skin, and shortness of breath with exercise. Blood work shows a macrocytic, hyperchromic anemia with hypersegmented neutrophils and normal folate levels. The patient has been taking omeprazole for over 3 years to treat gastric reflux disease. One method to treat this patient is to do which one of the following? (A) Give injections of vitamin B6 (B) Give injections of intrinsic factor (C) Give injections of vitamin B12 (D) Give oral folic acid (E) Give oral intrinsic factor

*The answer is E.* Ubiquitin is a protein found in all eukaryotic cells that undergoes ATP-dependent attachment to other proteins, labeling for degradation. The proteasome than recognized these ubiquinated proteins and uses ATP energy to drive them through its tubular structure, degrading them into small peptides in the process. Attachment of 4 or more ubiquitin monomers is required before most proteins are allowed entry into the proteasome. Ubiquitination plays an important role in many cell functions, including antigen processing, muscle wasting, cell cycle regulation, DNA repair, and disposal of misfolded proteins and regulatory enzymes. Impairment of the ubiqutin-proteasome system can contribute to the development of neurodegenerative disorders such as Parkinson's and Alzheimer's diseases. Failure of the system to properly degrade abnormal proteins causes protein misfolding, aggregation, and eventual obstruction of intracellular molecular traffic, leading to cell death. Together, the Parkin, PINK1 and DJ-1 genes code for a protein complex that promotes the degradation fo misfolded proteins via the ubiquitin-proteasome system. Mutations in Parkin, PINK1, and DJ-1 are each associated with autosomal recessive forms of Parkinson's disease that have an early age of onset.

A 42-year-old woman comes to the neurologist for enrollment in a research study. She has a 15-year history of resting tremor, bradykinesia, and cogwheel rigidity consistent with Parkinson's disease. One of her siblings recently started having similar symptoms. Genetic analysis is performed on the patient and her affected sibling. The result shows a loss-of-function mutation in a gene that leads to an accumulation of misfolded proteins. Which of the following biochemical processes is most likely defective in this patient? (A) Acetylation (B) Gamma-carboxylation (C) Glucuronidation (D) Phosphorylation (E) Ubiquitination

*The answer is B.* 5-FU inhibits the thymidylate synthase reaction, which produces dTMP from dUMP. It does not inhibit dihydrofolate reductase, which would block the production of FH4. Methotrexate inhibits dihydrofolate reductase. UMP is made via the de novo pyrimidine biosynthetic pathway, which is not inhibited by 5-FU. Methylcobalamin is produced from N5-methyl-FH4 and vitamin B12, and 5-FU does not block this activation of B12. CTP is produced from UTP, using glutamine as the nitrogen donor, and that reaction is also not affected by the presence of 5-FU.

A 42-year-old woman has been diagnosed with liver cancer, and is being treated with 5-FU. 5-FU is successful in destroying the tumor cells because it blocks the production of which one of the following? (A) FH4 (B) dTMP (C) UMP (D) Methylcobalamin (E) CTP

*The answer is D.* This patient has obstructive jaundice, causing her pruritus and scleral icterus. In this situation conjugated bilirubin cannot be excreted, and its levels are therefore elevated in the serum. The unconjugated bilirubin level, however, is not elevated. Alkaline phosphatase is usually elevated in cases of obstructive jaundice.

A 42-year-old woman presents to her physician with generalized itching. Physical examination reveals scleral icterus. Laboratory tests show: Total bilirubin: 2.7 mg/dL Conjugated bilirubin: 2.4 mg/dL Alkaline phosphatase: 253 U/L Aspartate aminotransferase: 36 U/L Alanine aminotransferase: 40 U/L What is the most likely mechanism underlying this patient's jaundice? (A) Absence of UDP-glucuronyl transferase (B) Decreased levels of UDP-glucuronyl transferase (C) Extravascular destruction of the patient's RBCs (D) Intrahepatic or extrahepatic biliary obstruction (E) Intravascular destruction of the patient's RBCs

*The answer is D.* The patient has pellagra due to niacin (vitamin B3) deficiency. Niacin is found in unrefined and enriched grains, cereal, milk, and lean meats. Due to poor intake of such nutrients, individuals who are homeless (such as the patient in this vignette) or alcoholics are at risk for developing niacin deficiency. Niacin is required for adequate cellular function and metabolism as an essential component of nicotinamide adenine dinucleotide and nicotinamide adenine dinucleotide phosphate. Because cellular functions in multiple organs and tissues are affected by niacin deficiency, there is a systemic clinical expression of pellagra involving the skin, gastrointestinal tract, and central nervous system. The symptoms of pellagra progress through the three D 's : Dermatitis, Diarrhea, and Dementia. If untreated, it can result in the fourth D: Death. Pellagra is an Italian word, meaning "thickened skin," and it is usually seen in sun-exposed areas of the body including the neck (as shown in the image), and thus it is also known as the Casal necklace. Since niacin is derived from tryptophan, a decrease in tryptophan absorption or an increase in tryptophan metabolism can produce similar symptoms. Finally, an inherited cause of pellagra is Hartnup disease, an autosomal recessive disorder affecting the absorption of neutral amino acids (like tryptophan) in the proximal convoluted tubules of the kidney. It will cause ni acin de ficiency and a pellagra-like syndrome .

A 43-year-old man comes to the clinic with complaints of pruritus and burning of both forearms that initially looked like sunburn. On physical examination, the affected skin appears thickened and hyperpigmented. A similar lesion is seen on the neck. The patient states that he has recently began to have diarrhea, and at times during the interview, he forgets what he was saying. For the past 6 months, he has been homeless and eating whatever he can find. A defect in the absorption of which amino acid would cause similar symptoms? A. Arginine B. Histidine C. Phenylalanine D. Tryptophan E. Tyrosine

*The answer is C.* Ammonia is a toxic byproduct of protein catabolism. It is excreted via the urea cycle in hepatocytes. Each molecule of urea contains two ammonia. The ammonia that are used to make urea come from multiple sources. One source is glutamate. Within hepatocytes, glutamate is converted to a-ketoglutarate and free ammonia via glutamate dehydrogenase. The free ammonia then combines with CO2 to form carbamoyl phosphate. Carbomyl phosphate then condenses with ornithine to initiate the urea cycle.

A 44-year-old alcoholic female with a 10 year history of chronic hepatitis C is brought into the emergency department because of recent altered mental status. Physical exam shows severe ascites and periumbilical venous distention. She has a flapping hand tremor. Lab tests show hyperammonemia. In a healthy patient the liver is able to excrete most ammonia via the urea cycle. Which of the following enzymes acts to release free ammonia immediately prior to ammonia entering the urea cycle? A. Argininosuccinate synthase B. Urease C. Glutamate dehydrogenase D. Carbamoyl phosphate synthetase E. Ornithine transcarbamoylase

*The answer is E.* This patient's macrocytic megaloblastic anemia without neurologic symptoms is most likely caused by folic acid deficiency. Folic acid plays a key role as a coenzyme for single-carbon transfer as seen in methylation reactions and is essential for the biosynthesis of purines and t pyrimidine thymidine. Deficiency of the vitamin is characterized by macrocytic megaloblastic anemia. The anemia is a result of diminished DNA synthesis in erythropoietic stem cells, leading to continued cell growth without progression to mitosis, thus presenting as macrocytosis. Large cells are seen with mean cell volumes of >100 μm3 and reduced levels of hemoglobin. Hypersegmented neutrophils (nuclei with six or more lobes), as seen in the blood smear in the vignette image, are a distinguishing feature of megaloblastic anemia, caused by folic acid and B12 deficiency. Folic acid is a water-soluble vitamin stored in small amounts by the body; thus a continuous supply is needed from foods such as green, leafy vegetables, lima beans, and whole-grain cereals. Folate deficiency is usually seen in pregnant women and alcoholics and is the most common vitamin deficiency in the United States. Folic acid supplementation by pregnant women reduces the incidence of neural tube defects. *Bottom Line:* In the form of tetrahydrofolate, folic acid participates in a number of single-carbon transfer reactions, including the synthesis of nitrogenous bases like thymidine and uridine as well as several amino acids. Deficiency can lead to megaloblastic anemia.

A 44-year-old former alcoholic woman presents with worsening fatigue for the past 2 months. She reports no neurological deficits or other symptoms, and physical examination is unremarkable. Laboratory studies reveal a hemoglobin level of 8 g/dL and mean corpuscular volume of 110 µm^3 , The blood smear shown in the image was obtained as part of her work-up. Which of the following is the function of the vitamin causing this patient 's condition? A. Catalyzes γ-carboxylation of glutamic acid residues B. Hydroxylates prolyl and lysyl residues C. Increases intestinal calcium and phosphate absorption D. Makes up the constituents of the visual pigments E. Synthesizes succinyl CoA

*The answer is B.* High homocysteine levels may be a risk factor for early atherosclerotic disease. The high levels appear to be associated with low B6, B12, and/or folate levels. Vitamin B12 is required for the methionine synthase reaction (homocysteine 1 N5-methyl-FH4 yields methionine and FH4), vitamin B6 is required for the cystathionine ß-synthase reaction (homocysteine + serine yields cystathionine), and folate is required for the methionine synthase reaction. Vitamins B3 (niacin), C, E, and B1 (thiamine) are not required for homocysteine metabolism.

A 45-year-old male is concerned about his risk of a heart attack since his brother just had a heart attack at the age of 46. His physician orders an HbA1C, lipid panel, and homocysteine level. A high homocysteine level could be associated with a deficiency of which one of the following vitamins? (A) B3 (B) B12 (C) C (D) E (E) B1

*The answer is B.* Methionine, obtained from the diet, is activated by the reaction with ATP to form SAM, the universal methyl donor. After donating a methyl group, SAM is converted to S-adenosylhomocysteine. A hydrolase removes the adenosine, generating homocysteine. Homocysteine then has two fates-it can react with N5-methyl-FH4 (with B12) to regenerate methionine, or it can react with serine (in the presence of B6) to form cystathionine, which goes on to form cysteine. A defect in either the enzyme that forms methionine (methionine synthase) or the enzyme that forms cystathionine (cystathionine ß-synthase) will lead to elevated homocysteine levels.

A 45-year-old male is concerned about his risk of a heart attack since his brother just had a heart attack at the age of 46. His physician orders an HbA1C, lipid panel, and homocysteine level. A reduction in the metabolism of which one of the following amino acids can lead to elevated homocysteine levels in the blood? (A) Alanine (B) Methionine (C) Phenylalanine (D) Glutamate (E) Cysteine

*The answer is D.* Glycine metabolism involves a variety of pathways, one of which is a reversible transamination of glycine to form glycoxylic acid. The enzyme is glycine aminotransferase (also known as alanine-glyoxylate aminotransferase), and is defective in the disorder primary hyperoxaluria type 1. Glyoxalate can be produced from glycine by two different enzymes: the first is D-amino acid oxidase, and the second is the glycine aminotransferase. Glyoxylate is oxidized to oxalate, which forms calcium salts in the kidney and precipitates, forming kidney stones. A defect in any enzyme, which may lead to an accumulation of glyoxylate, will lead to kidney stone formation.

A 45-year-old man developed severe pain in his back, which, upon going to the emergency department, turned out to be due to kidney stones. A stone chemical analysis indicated a buildup of oxalic and glyoxalic acids. These compounds can accumulate due to a problem in the metabolism of which one of the following amino acids? (A) Alanine (B) Tryptophan (C) Isoleucine (D) Glycine (E) Glutamine

*The answer is E.* The patient has an adrenal pheochromocytoma that is causing episodic hypertension (leading to headache) and arrhythmia (leading to palpitations). The tumor arises from the sympathetic chromaffin cells in the adrenal gland. Catecholamines secreted by the tumor include epinephrine, norepinephrine, and dopamine. All three of these are derived from tyrosine {which is, in tum, derived from phenylalanine).

A 45-year-old woman comes to the physician because of headaches and palpitations. Her blood pressure in the office is 120/70 mm Hg. but when she is symptomatic her pulse is 105/min and blood pressure is 190/140 mm Hg. A CT scan of the abdomen shows an adrenal mass. The hormones secreted by the adrenal mass are derived from which of the following amino acids? A. Arginine B. Glutamate C. Glycine D. Tryptophan E. Tyrosine

*The answer is E.* This patient has alkaptonuria. This is the result of a congenital deficiency of homogentisic acid oxidase, the third enzyme in the tyrosine degradation pathway (pathway shown in the image). Due to this deficiency, homogentisic acid builds up in the body and polymerizes to form alkapton bodies, which then deposit to cause dark urine and dark cartilage. Patients also sometimes suffer from arthralgias and decreased joint mobility, but are otherwise healt hy.

A 46-year-old man presents to the orthopedics clinic complaining of knee pain he has been having after playing basketball. He denies any trauma to his knees and any fevers, redness, or swelling of his knees. On examination, he has full range of motion without resistance, along with some mild crepitus on flexion/extension. He is sent home with a prescription for ibuprofen. Two months later he returns after an obvious tear of his right anterior cruciate ligament. During preoperative evaluation for an arthroscopic repair of the injury, the laboratory calls to report that his urine is unusually dark in color, but is negative for blood. In the operating room the surgeons are surprised to find that the cartilage in his knee is also black. This patient 's abnormal enzyme is involved in the degradation of which amino acid? A. Cystine B. Homocysteine C. Leucine D. Methionine E. Tyrosine F. Valine

*The answer is A.* The patient is suffering from a gout attack due to the buildup of uric acid in the blood, and precipitation of uric acid in "cold" areas of the body, such as the great toe. Uric acid has the basic ring structure of the purines and is the degradative product of adenine and guanine. The ring structure of uric acid is not at all similar to pyrimidines, nicotinamides (derived from the vitamin niacin), amino acids, or fatty acids.

A 47-year-old woman visits the emergency department due to severe pain in the metatarsophalangeal (MTP) joint of her right great toe. Upon examination, the toe is bright red, swollen, warm, and very sensitive to the touch. Analysis of joint fluid shows crystals. The patient is given indomethacin to reduce the severity of the symptoms. The crystals that are accumulating in this patient are most likely derived from which type of molecule? (A) Purines (B) Pyrimidines (C) Nicotinamides (D) Amino acids (E) Fatty acids

*The answer is D.* Alkaptonuria is caused by a deficiency of the enzyme homogentisic acid oxidase, resulting in increased homogentisic acid levels in the blood and urine. After excretion from the body, excess homogentisic acid in the urine is slowly converted to its oxidized form, alkapton, which has a characteristic dark brown to black color and is responsible for the finding described in the vignette.

A 48-year-old male is seen in clinic for a pre-operative anesthesia assessment prior to undergoing bilateral total hip replacement surgery for chronic arthritis. Other than severe arthritis, he has no known past medical history. On exam, he is noted to have dark pigmentation in the tip of his nose and pinna of his ears. He provides a urine sample for routine analysis that initially appears normal, but turns dark brown after it is left standing for a short period of time. Levels of which of the following are likely elevated in the blood? . A. Dopamine B. Tetrahydrofolate C. Uric acid D. Homogentisic acid E. Homocysteine

*The answer is B.* This patient most likely suffers from alkaptonuria, an autosomal recessive defect in tyrosine metabolism that presents with the deposition of dark blue and gray pigment throughout the connective tissues of the body, a condition called ochronosis. Though merely a cosmetic nuisance in the pinna of the ears and tip of the nose, pigment deposition in articular cartilage can lead to arthropathy and debilitating arthritis, and many patients with alkaptonuria require total joint replacement at atypically young ages. In addition, involvement of the cartilage of the heart can result in dysfunctional cardiac valves, and calcification of the intervertebral disks can result in complete ankylosis- The classic finding in alkaptonuria is a normal-appearing urine sample that Later turns dark brown or black when left standing.

A 48-year-old male is seen in clinic for a pre-operative anesthesia assessment prior to undergoing bilateral total hip replacement surgery for chronic arthritis. Other than the severe arthritis, he has no known past medical history. On exam, he is noted to have dark pigmentation in the tip of his nose and pinna of his ears. He provides a urine sample for routine analysis that initially appears normal but turns dark brown after it is left standing for a short period of time. What is the most likely diagnosis in this case? A. Dehydration B. Alkaptonuria C. Psoriasis D. Rhabdomyolysis E. Pancreatic carcinoma

*The answer is C.* Lead inhibits both ferrochelatase (increasing the zinc protoporphyrin) and ALA dehydrase (increasing δ-ALA).

A 48-year-old man developed abdominal colic, muscle pain, and fatigue. Following a 3-week hospitalization, acute intermittent porphyria was initially diagnosed based on a high level of urinary δ-aminolevulinic acid. Subsequent analysis of the patient's circulating red blood cells revealed that 70% contained elevated levels of zinc protoporphyrin, and the diagnosis was corrected. The correct diagnosis is most likely to be A. protoporphyria B. congenital erythropoietic porphyria C. lead poisoning D. barbiturate addiction E. iron deficiency

*The answer is B.* This woman suffers from β-thalassemia major, the most severe form of β-thalassemia, in which the β-chain is absent. Clinically β-thalassemia major manifests as severe hemolysis and ineffective erythropoiesis. These individuals are transfusion dependent and frequently develop iron overload. The consequences of iron overload due to transfusion dependency or secondary hemochromatosis are described in the stem. These manifestations are due to iron deposition in various tissues including the pancreas, heart, and skin. β-Thalassemia is more common among Mediterranean populations, whereas β-thalassemia is more common among Asian and African populations.

A 48-year-old woman of Mediterranean descent presents because of fatigue, arthralgias, discomfort in her right upper abdominal quadrant, and polyuria. Laboratory tests are remarkable for elevated glucose level, elevated bilirubin, low hemoglobin, elevated reticulocytes, and increased transferrin saturation. Cardiac testing shows moderate restrictive cardiomyopathy. She frequently has required blood transfusions throughout her life. Which hereditary disorder does this patient most likely have? (A) Absence of the hemoglobin α-chain (B) Absence of the hemoglobin β-chain (C) Mutation resulting in increased absorption of dietary iron (D) Mutations in the gene encoding ankyrin (E) Mutations resulting in copper accumulation

*The answer is A.* Homocysteine, the substrate for the enzyme, accumulates increasing the risk of deep vein thrombosis and disrupting the normal cross-linking of fibrillin. Deficiency of homocysteine methyltransferase would cause homocystinuria, but would also predispose to megaloblastic anemia.

A 49-year-old man with a rare recessive condition is at high risk for deep vein thrombosis and stroke and has had replacement of ectopic lenses. He has a normal hematocrit and no evidence of megaloblastic anemia. A mutation in the gene encoding which of the following is most likely to cause this disease? A. Cystathionine synthase B. Homocysteine methyltransferase C. Fibrillin D. Lysyl oxidase E. Branched chain α-ketoacid dehydrogenase

*The answer is C.* Only methionine is degraded via the homocysteine/cystathionine pathway and would be elevated in the plasma of a cystathionine synthase-deficient patient via activation of homocysteine methyltransferase by excess substrate.

A 49-year-old man with a rare recessive condition is at high risk for deep vein thrombosis and stroke and has had replacement of ectopic lenses. He has a normal hematocrit and no evidence of megaloblastic anemia. Amino acid analysis of this patient's plasma would most likely reveal an abnormally elevated level of A. lysine B. leucine C. methionine D. ornithine E. cysteine

*The answer is E.* This child is suffering from an inherited form of hyperammonemia as a result of a defect in ornithine transcarbamylase (OTC). In the urea cycle, OTC combines carbamoyl phosphate and ornithine to make citrulline. When OTC is deficient, excess carbamoyl phosphate enters the pyrimidine synthesis pathway to cause increased orotic acid in the blood, which distinguishes OTC deficiency from other urea cycle disorders. A defect in OTC causes an excess of ammonia in circulation, which leads to mental retardation, seizures, and ultimately death. Some patients with OTC deficiency also exhibit a very low blood urea nitrogen, but this is not enough to make a conclusive diagnosis. Unlike the rest of the urea cycle disorders that are autosomal recessive, deficiency of OTC is X-linked.

A 5-day-old boy is brought to the emergency department after a tonic clonic seizure at home. The infant is the product of a full-term, uneventful pregnancy, and was normal until two days prior to presentation. The mother reports irritability and poor feeding at home, and the infant was difficult to rouse this morning before suffering the seizure. On physical examination, the infant is tachypneic to 75/min, has icteric sclerae, and has poor muscle tone throughout. Laboratory studies show the following levels: plasma ammonia, 300 μmol/L (normal = 10-40 μmol/L); blood urea nitrogen, 1.5 mg/dL; and creatinine, 0.4 mg/dL. A plasma amino acid analysis fails to detect citrulline. Urine amino acids demonstrate elevated orotic acid levels. This patient suffers from a deficiency of which of the following enzymes? (A) α-Galactosidase A (B) Aldose B (C) Galactose 1-phosphate uridylyltransferase (D) Lysosomal α-glucosidase (E) Ornithine transcarbamylase

*The answer is D.* The child has the symptoms of classic MSUD. In MSUD, the branched-chain amino acids (valine, leucine, and isoleucine) can be transaminated but not oxidatively decarboxylated because the α-keto acid dehydrogenase is defective. Therefore, these amino acids and their transamination products (the corresponding α-keto acids) will be elevated. Phenylalanine and phenylketones are elevated in PKU not in MSUD. The musky sweet odor of the urine strongly suggests MSUD. Dopa levels (dopa is derived from tyrosine) should be normal in MSUD, as there is no mutation in the phenylalanine degradation pathway. Acetone, derived from acetoacetate, is exhaled and is not found in the blood. Depending on the feeding state of the child, ketosis could develop, but the acetone would still be exhaled, while blood levels of acetoacetate and ß-hydroxybutyrate might be elevated.

A 5-day-old infant initially began feeding poorly, grew irritable, then very lethargic. The diaper, when changed, had a musky, sweet odor to it. At the emergency department of the local hospital, the child was suspected of having an inborn error of metabolism, and blood work was ordered. You would expect to see an elevation of which one of the following given the conditions of the child's case? (A) Phenylalanine (B) Phenylketones (C) Dopa (D) Isoleucine (E) Acetone

*The answer is F.* Tetrahydrobiopterin (BH,) is a cofactor in the synthesis of serotonin (a major neurotransmitter), tyrosine (a precursor of DOPA), and DOPA (the antecedent of the neurotransmitters dopamine, norepinephrine and epinephrine). Dihydrobiopterin reductase enzymatically reduces dihydrobiopterln (BH2} to tetrahydrobloptertn (BH4). Serotonin (5 hydroxytryptamine or 5HT) is formed through hydroxylation and decarboxylation of the amino acid tryptophan. Most cases of phenylketonuria are due to phenylalanine hydroxylase deficiency. Less commonly, the etiology is due to BH4 deficiency secondary to dihydropteridine reductase deficiency. The consequences of defective phenylalanine and tryptophan metabolism are phenylalanine accumulation (Choice E) and low levels of serotonin and other neurotransmitters, respectively. The combination of high phenylalanine levels, which may disrupt neuronal and glial development, and low serotonin and other neurotransmitters results in progressive neurologic deterioration in untreated patients. Manifestations include developmental delay. hypotonia, dystonia, and seizures. Treatment includes both a low phenylalanine diet and BH4 supplementation.

A 5-month-old boy is brought to the office by his parents who are concerned that he has developmental delay. He has 3 older siblings, and the parents report that his siblings were much more active and interactive at the boy's age. The family has just moved to the United States from South America and did not have consistent primary care previously. Physical examination shows that the boy is unable to roll from front to back or back to front and does not seem to recognize his parents. Comprehensive laboratory evaluation reveals impaired tetrahydrobiopterin synthesis. Which of the following is most likely deficient in this patient? (A) Acetylcholine (B) Gamma-aminobutyric acid (C) Glutamate (D) Glycine (E) Phenylalanine (F) Serotonin

*The answer is B.* Hartnup disease is a transport defect, manifest in both the kidney and intestinal epithelial cells. The transporter is for large, neutral amino acids, and even though many amino acid transport systems have overlapping specificities, tryptophan uptake can be limiting with this disorder. Under such conditions, the body may not produce sufficient NAD for its needs (the nicotinamide ring can be produced from dietary niacin, as well as tryptophan), and a pellagra-like illness can develop. Giving the child niacin will allow the NAD to be regenerated, and a high-protein diet may increase tryptophan levels such that the transporter can transport tryptophan from the intestinal lumen into the bloodstream. Cystinuria is a different transport defect that will not allow cystine to be absorbed from the diet, or removed from the urine and returned to the blood in the kidney (which can give rise to kidney stones). Myasthenia gravis is due to autoantibodies directed against the acetylcholine receptor. Alkaptonuria is due to a defect in homogentisic acid oxidase, and jaundice results from an inability to add glucuronic acid residues to bilirubin in the liver.

A 5-year-old boy has had episodic periods during which areas of his skin would develop a rash, which would spontaneously resolve in a week to 10 days. An astute pediatrician told the boy's parents to give him niacin the next time this occurred and a high-protein diet, and when they did, the rash resolved in a day or two. A likely disorder that this child has is which one of the following? (A) Cystinuria (B) Hartnup disease (C) Myasthenia gravis (D) Alkaptonuria (E) Jaundice

*The answer is C.* This patient most likely has Lesch-Nyhan syndrome, a rare X-linked genetic disease caused by a deficiency of the HGPRT enzyme. Defects in the HGPRT protein result in the accumulation of uric acid in all body fluids, which explains the patient's swollen joints. Yellow-orange granules are a classic finding of Lesch - Nyhan, and represent urate crystals in the diaper. Patients with Lesch-Nyhan also present with episodes of self-mutilation and dystonia. The lack of HGPRT inactivates the purine salvage pathway, which prevents guanine and hypoxanthine from being recycled to GMP and IMP, respectively. Since guanine and hypoxanthine are being lost as uric acid, the de novo synthesis pathway for purines must compensate, which utilizes folate and vitamin B12 . This inappropriate utilization of folate and vitamin B12 can result in macrocytic anemia, whose most prominent feature in a blood smear is the presence of hypersegmented neutrophils.

A 5-year-old boy presents to the emergency department with painful, swollen joints of 4 days' duration. During his examination, his mother mentions that his diapers often contain orange granules and that he has a "bad habit" of biting his fingers and lips. What may be present on the blood smear as a sequela to this patient's enzyme deficiency? A. Acanthocytes B. Elliptocytes C. Hypersegmented neutrophils D. Ringed sideroblasts E. Schistocytes F. Target cells or codocytes G. Teardrop cells

*The answer is D.* The treatment for a positive PPD test (tuberculosis) is isoniazid, which can interfere with vitamin B6 (pyridoxine) function in cells. Pyridoxine is activated to PLP in cells (the active form of the vitamin), and isoniazid blocks this activation. A deficiency of B6 can lead to peripheral neuropathy, as B6 is required for the conversion of tryptophan to niacin. In many cases, vitamin B6 is given along with isoniazid to prevent these side effects from occurring (by providing more substrate than the isoniazid can bind to). Isoniazid does not affect thiamine (B1), riboflavin (B2), niacin (B3), or cobalamin (B12) metabolism, although riboflavin is required to activate pyridoxine.

A medical student has been exposed to a patient with tuberculosis and developed a positive tuberculin test (PPD), but exhibited a normal chest X-ray. He is placed on a 6-month course of prophylactic treatment, but subsequently develops peripheral neuropathies. Which of the following vitamins would be considered a treatment for the neurotoxicity? (A) B1 (B) B2 (C) B3 (D) B6 (E) B12

*The answer is C.* The patient's presentation is most consistent with homocystinuria, the most common inborn error of methionine metabolism. Most patients present at age 3-10 with ectopia lentis (dislocated lens). About hald of patients have intellectual disability. Other clinical manifestations include a Marfanoid habitus (eg, elongated limbs, arachnodactyly). Patients are at high risk for thromboembolic occlusion of both large and small vessels, especially those of the brain, heart, and kidneys. Thromboembolic complications are the major cause of morbidity and mortality in these patients. Homocystinuria is most frequently caused by an autosomal recessive deficiency of cystathionine beta-synthase, an enzyme that requires pyridoxine (vitamin B6) as a cofactor. Approximately 50% of the affected patients respond to high doses of pyridoxine, which improves residual enzymatic activity and reduces plasma homocysteine levels. Additional treatment includes dietary restriction of methionine.

A 5-year-old boy with developmental delay is brought to the office due to difficult "seeing the board" at school. Examination shows a boy with a tall, thin habitus with elongated limbs. Funduscopy shows bilateral lens subluxation. Four years later, the patient dies suddenly of a massive cerebrovascular accident. Autopsy shows middle cerebral artery thrombosis and old renal infarcts. His parents wish to know if anything could have been the most appropriate supplementation for this patient? (A) Ascorbic acid (B) Carnitine (C) Pyridoxine (D) Thiamine (E) Tyrosine (F) Vitamin K

*The answer is F.* Pellagra ("rough skin" in Italian vernacular) is due to niacin deficiency and is characterized by the "3 Ds": dermatitis, diarrhea, and dementia: Dermatitis is bilateral and symmetric on sun-exposed areas of the body and is characterized by rough, thick, scaly skin. Diarrhea is due to atrophy (and occasions ulceration) of columnar epithelium of the gastrointestinal tract. Dementia is due to neuronal degeneration in the brain end spinal cord, with lesions similar to those associated with pernicious anemia. Niacin (nicotinic acid or vitamin BJ is an essential component of the coenzymes nicotinamide adenine dinucleotide (NAD) and nicotinamide adenine dinucleotide phosphate (NADP), which participate in redox metabolism. Specifically, NAD functions as a coenzyme for dehydrogenases involved in the metabolism of fats, carbohydrates, and amino acids; NADP is crucial in the hexose-monophosphate shunt of glucose metabolism and for biosynthesis of cholesterol and fatty acids. Niacin can be obtained through dietary intake or synthesized endogenously from tryptophan. In developing countries, niacin deficiency is seen in populations that subsist primarily on corn products (niacin in com occurs in a bound, unabsorbable form). In developed countries, it is primarily seen in patients with impaired nutritional intake (eg, alcoholism, chronic illness). Pellagra can also be seen occasionally in those with carcinoid syndrome, prolonged isoniazid therapy, or Hartnup disease.

A 52-year-old woman comes to the office with a 2-month history of skin rash that worsens with sun exposure. Her family says that lately she has become irritable, hostile, and has had episodes of disorientation. The patient does not use tobacco or illicit drugs but has been drinking half a bottle of gin daily. Further questioning reveals poor nutritional intake and intermittent diarrhea. The patient restricts her diet for weight control. Body mess index is 17 kg/m^2. On examination, she has a well-demarcated, hyperpigmented, scaly rash on the hands, forearms, and upper chest. The cause of most of her symptoms is determined to be a lack of the precursor vitamin for the synthesis of NAD+ coenzyme. The compensatory pathway to synthesize this coenzyme uses which of the following as a precursor? (A) Arginine (B) Carotene (C) Cholesterol (D) Orotic acid (E) Phenylalanine (F) Tryptophan

*The answer is F.* Tumor lysine syndrome is an oncologic emergency that can develop during chemotherapy for high-grade lymphomas, leukemias, and other cancers with rapid cell turnover, substantial tumor burden, or high sensitivity to chemotherapy. When large numbers of tumor cells are destroyed during a short interval, intracellular ions such as potassium and phosphorous are released into the serum along with nucleic acids (which are catabolized into uric acid). Elevated potassium levels can result in arrhythmias that may be fatal. Uric acid can precipitate in the kidney, causing obstructive uropathy and acute renal failure. The risk of tumor lysine syndrome can be reduced by aggressive fluid hydration. Allopurinol (a xanthine oxidase inhibitor) or rasburicase can also be used to reduce uric acid levels during the breakdown of tumor cells. Rasburicase is a recombinant version of orate oxidase, an enzyme present in many mammals, but not in humans. lt catalyzes the conversion of uric acid to allantoin, which is 5-10 times more soluble than uric acid. Rasburicase is effective in preventing and treating hyperuricemia and the resulting renal manifestations of tumor lysine syndrome.

A 52-year-old woman comes to the physician complaining of a rapidly enlarging neck mass and persistent nighttime sweating. After the appropriate workup, she is diagnosed with diffuse large B-cell lymphoma and admitted to the hospital. She is started on combination chemotherapy without incident. On the third day of treatment, she is noted to have decreased urine output. Laboratory analysis shows increased levels of blood urea nitrogen and creatinine. Electrocardiography shows peaked T waves. Administration of which of the following agents would have most likely prevented this patient's 15 renal impairment? A. Denosumab B. Folinic acid C. N-acetylcysteine D. Prednisone E. Probenecid F. Rasburicase

*The answer is C.* Heme synthesis occurs partly in the mitochondria and partly in the cytoplasm of erythrocytes. Mitochondria are necessary for the first and the final 3 steps. Erythrocyte precursors divide a number of times before finally losing their nuclei and mitochondria and forming mature red blood cells that survive for about 120 days (4 months). When erythrocytes lose their mitochondria, they lose the ability to generate heme and therefore hemoglobin. Heme is synthesized in virtually every organ, but the principal sites of synthesis are erythrocyte precursor cells (located in the bone marrow) and hepatocytes (use heme in microsomal cytochrome P450 system).

A 54-year-old man with a history of chronic kidney disease due to hypertension develops anemia. Evaluation reveals that the cause of anemia is erythropoietin deficiency. Erythropoietin increases the numbers of erythroid precursor cells in the bone marrow and induces heme production in erythrocyte precursors. In this patient, mature erythrocytes are found that are unable to synthesize heme even though they contain detectable levels of cytoplasmic enzymes involved in heme synthesis. Lack of which of the following cellular organelles best explains this phenomenon? A. Endoptasrnic reticulum B. Golgi apparatus C. Mitochondria D. Nucleus E. Peroxisomes F. Proteasomes

*The answer is A.* This patient has hepatic encephalopathy, likely due to recent gastrointestinal bleeding and a corresponding increase in ammonia and nitrogen absorption by the gut. The pathogenesis of hepatic encephalopathy is related to increased circulatory levels of ammonia and other neurotoxins due to failure of the Ever to metabolize waste products. When ammonia levels rise acutely, astrocyte and neuron function ace affected. Within the brain, astrocytes and neurons interact to regulate the metabolism of glutamate, glutamine, and ammonia in a process known as the glutamate-glutamine cycle. Glutamate released by neurons during neurotransmission is taken up by astrocytes and converted to glutamine, a non-neuroactive compound. Glutamine is then released by astrocytes and taken up by neurons, where it is either converted back to glutamate for use as a neurotransmitter or transaminated into α-ketoglutarate for use in the Krebs cycle. Ammonia toxicity results in part from depletion of glutamate and α-ketoglutarate in the brain during the process of ammonia detoxification. Hyperammonemia increases the conversion of glutamate into glutamine by glutamine synthetase within astrocytes (Choice C). The resulting increase in glutamine leads to hyperosmolarity and mitochondrial dysfunction, causing astrocytic swelling and impairment. Increased glutamine formation also decreases total brain glutamate stores, impairing excitatory neurotransmission (mediated by NMDA, AMPA, and kainate receptors) and neuronal energy production. In addition, ammonia can be detoxified to glutamate via glutamate dehydrogenase, depleting α-ketoglutarate and further impairing energy metabolism in the brain.

A 54-year-old man with a history of cirrhosis is brought to the emergency department by his wife, who found him agitated and confused. She reports that he vomited bright red blood several times yesterday. His cirrhosis is secondary to chronic hepatitis C infection. and he has received treatment for esophageal varices in the past. Physical examination reveals abdominal distention, decreased liver span, and testicular atrophy. A flapping tremor involving his hands is seen with wrist extension. Serum studies show elevated ammonia levels. Which of the following metabolic intermediates is most likely to be deficient in this patient's brain? (A) α-ketoglutarate (B) Carnitine (C) Glutamine (D) Lactate (D) Oxindole

*The answer is E.* This patient has features of anemia (eg. exertional dyspnea, fatigue, pallor) with associated neurologic deficits that are highly suggestive of vitamin B12 deficiency (eg. megaloblastic anemia, pancytopenia) are related to impaired DNA synthesis. Vitamin B12 (hydroxycobalamin) also serves as a cofactor for methylmalonyl-CoA mutase (converts methylmalonyl-CoA to succinyl-CoA) and methionine synthase (converts homocysteine and folic acid to methionine). B12 deficiency consequently results in elevated levels of serum methylmalonic acid and homocysteine. Increased methylmalonic acid levels can disrupt myelin synthesis and result in subacute combined degeneration of the dorsal columns (eg. loss of proprioception/vibration, Romberg sign) and lateral corticospinal tract (eg. spastic muscle weakness, hyperflexia). Axonal degeneration of peripheral nerves can also be seen.

A 54-year-old woman is evaluated in the clinic for exertional dyspnea and easy fatigability. The patient has no chest pain, cough, or wheezing. She does not use tobacco, alcohol, or illicit drugs. On physical examination, her gait is unstable when her eyes are closed and there is impaired vibratory sensation in the lower extremities. Marked pallor of the conjunctivae, nail beds, and palms is present. Which of the following laboratory tests in this patient? A. Erythrocyte glucose-6-phosphate dehydrogenase activity O B. Erythrocyte glutathione reductase activity C. Erythrocytic pyruvate kinase activity D. Erythrocyte transketolase activity E. Serum methylmalonic acid level F. Serum protoporphyrin level

*The answer is D.* The fact that the urine sample turned dark is strongly suggestive that the patient has alkaptonuria, a deficiency of homogentisic acid oxidase, an enzyme in the tyrosine degradation pathway that converts homogentisic acid to maleylacetoacetic acid. Levels of homogentisic acid build up and are excreted in the urine, where exposure to alka li or oxygen results in the characteristic change of the urine color to dark black. Although the disease is generally benign, patients with alkaptonuria often have problems with arthritis. The mechanism is not entirely understood but may be caused by oxidized derivatives of homogentisic acid interfering with collagen cross- linking.

A 55-year-old man with recurrent arthritis of both knees presents to his primary care physician for a routine check-up. Results of urinalysis are negative, but the nurse noticed that the urine turned dark after several hours. Levels of what substance will be most notably elevated in the urine sample from this patient? A. Branched amino acids B. Cystine C. Homocysteine D. Homogentisic acid E. Phenylalanine

*The answer is A.* Deficiencies of vitamin B12 (cobalamin) and folate are two of the most common causes of megaloblastic anemia. Laboratory tests that can assist in differentiating between these two deficiencies include methylmalonic acid and homocysteine. Vitamin B12 deficiency results in elevation of both methylmalonic acid and homocysteine, whereas. folate deficiency results in isolated elevation of homocysteine but normal levels of methylmalonic acid. The laboratory values provided in the vignette should guide one to the correct answer. Furthermore, the patient's history of enteritis correlates best with vitamin B12 deficiency. Vitamin B12 is synthesized only by microorganisms and is obtained from foods of animal origin, It is absorbed in the terminal ileum with the help of intrinsic factor, which is secreted by gastric parietal cells. Although not mentioned in the vignette, neurologic deficits (if present) are additionally helpful in making this distinction. Lack of vitamin B12 results in increased odd-chain fatty acid fragments and decreased methionine, which leads to neurologic deficits, most notably subacute combined degeneration of the spinal cord. This is in contrast to folate deficiency, which leads to macrocytic anemia but does not result in neurologic deficits. It is important to identify the underl ying cause of megaloblastic anemia. Folate supplementation will improve the anemia of both folate and cobalamin deficiency; this may provide false reassurance if the megaloblastic anemia is due to cobalamin deficiency since folate supplementation will not treat the neurologic manifestations associated with vitamin B12 deficiency.

A 55-year-old man with type 2 diabetes mellitus, elevated cholesterol, and chronic enteritis is found to have a low hematocrit and an elevated mean corpuscular volume on a routine blood test. Blood tests reveal elevated levels of serum homocysteine and methylmalonic acid. A deficiency of which micronutrient is most likely responsible for his laboratory findings? A. Cobalamin B. Folate C. Pyridoxine D. Thiamine E. Vitamin A

*The answer is E.* Pernicious anemia describes a disease in which the body lacks the ability to absorb vitamin B12. Vitamin B12 deficiency can result in megaloblastic anemia, which is characterized by increased mean corpuscular volume (macrocytosis), and hypersegmented neutrophils, as well as peripheral neuropathy. Pernicious anemia is caused by deficiency of intrinsic factor, a protein produced by gastric parietal cells that is required to absorb vitamin B12. This deficiency can be caused by gastrectomy (eg, during bariatric surgery) or autoimmune destruction of parietal cells or int rinsic factor

A 55-year-old woman presents to the physician because of low energy, generalized weakness, and shortness of breath for several months. She is otherwise healthy and is not taking any medication. On physical exam, she looks pale, and capillary refill is delayed. Laboratory tests reveal a hematocrit of 20% and a mean corpuscular volume of 115 fL. What is the most likely cause of her abnormal laboratory findings? A. Calorie deficiency B. Chronic renal disease C. Following a lacto-ovo vegetarian diet (vegetarian that consumes dairy and eggs) for 6 months D. Iron deficiency E. Pernicious anemia

*The answer is C.* This patient is presenting with decompensated liver failure and signs of hepatic encephalopathy, His protuberant abdomen with a fluid wave, spider angiomata, and alcohol history all point toward portal hypertension caused by alcoholic cirrhosis. His confusion, slurring of speech, and flapping of hands when outstretched (asterixis) are all signs of encephalopathy due to the buildup of ammonia. Patients presenting with hepatic encephalopathy are often treated with agents to reduce ammonia levels, including lactulose and antibiotics, both of which work in the gut to bind ammonia and eliminate urease-producing bacteria, respectively. In a healthy person, ammonia (the potentially toxic by-product of protein degradation) is converted to urea in the liver. Urea is a more neutral compound that is eliminated by the kidney in the urine. The conversion from ammonia to urea constitutes the urea cycle, which takes place in hepatocytes. The initial steps of the urea cycle (the convergence of ammonia and carbon dioxide to form carbamoyl phosphate; the convergence of carbamoyl phosphate and ornithine to form citrulline) all occur in the mitochondrial matrix of hepatocytes. Citrulline is then exported from the mitochondria, and the ensuing steps occur in the cytosol, where urea is ultimately generated.

A 58-year-old man with a history of alcohol abuse presents to the hospital with a protuberant abdomen, slurring of speech, and confusion. On physical examination, there are multiple spider angiomata on his chest, and a fluid wave is elicited on abdominal examination. When his hands are extended outward, a flapping tremor appears. He is promptly treated with medications to reduce the level of toxic metabolite responsible for his symptoms. Normally, where do the initial steps for detoxification of this metabolite take place? A. Hepatocyte cytoplasm B. Hepatocyte endoplasmic reticulum C. Hepatocyte mitochondria D. Kupffer cell cytoplasm E. Portal circulation

*The answer is E.* Uridine bypasses the mutated step of the pathway, allowing UTP to be produced. UTP inhibits the rate-determining step of the pathway, carbamoyl phosphate synthetase II, which halts the production of orotic acid, thereby lowering the concentration of orotate in the urine. This is the mechanism whereby the crystals no longer form. Uridine is not inhibiting the enzyme that directly forms orotate, nor does it inhibit aspartate transcarbamoylase or CPS-I. While adding uridine does bypass the regulated step, it is the synthesis of UTP from the uridine that leads to the drop in orotate production.

A 6-month-old infant is seen by the pediatrician for developmental delay. Blood work shows megaloblastic anemia, although measurements of B12 and folate are in the high normal range. Urinalysis demonstrates, upon standing, the formation of a crystalline substance. Supplementation of the child's diet with uridine reversed virtually all of the clinical problems. After uridine treatment, the crystals were no longer found in the urine. This is due to which of the following? (A) Inhibition of the enzyme producing the crystalline molecule (B) Bypassing the mutated step of the pathway (C) Inhibition of aspartate transcarbamoylase (D) Inhibition of nitrogen fixation by carbamoyl phosphate synthetase I (E) Inhibition of carbamoyl phosphate synthetase II

*The answer is A.* There are many types of porphyria. However, the most testable ones are: 1) acute intermittent porphyria, and 2) porphyria cuteness tarda. Acute intermittent porphyria (AIP) is an autosomal dominant disorder caused by a partial deficiency of the heme producing enzyme porphobilinogen deaminase (PEG, also called hydroxymethyibilane synthase). Symptoms of AIP include psychiatric disturbances such as anxiety and depression, vague abdominal symptoms often leading to unnecessary surgery, and dark red blood. The symptoms of AIP are often vague and often lead to misdiagnosis or late diagnosis. Many medications are known to cause neurovisceral attacks in patients with AIP including some frequently prescribed to patients with mental illnesses. Some steps of heme synthesis occur in the cytoplasm while some occur in mitochondria. As mentioned, AIP is caused by a deficiency of the enzyme PBG. The downstream product of PEG is hydroxymethylbilane. This reaction occurs in the cytoplasm. Dysfunction of PEG will decrease concentrations of hydroxymethylbilane and increase concentration of porphobilinogen.

A 58-year-old woman presents with a complaint of dark-brown urine- The patient has been complaining of vague abdominal pain for the past week that occurs intermittently without provocation. A random urine porphobilinogen level is elevated. The patient is subsequently confirmed to have acute intermittent porphyria, a disorder of porph0bilinogen deaminase. Which of the following is the product of the affected enzyme and location of the affected enzyme? A. Hydroxymethylbilane, cytoplasm B. Porphobilinogen, mitochondria C. Uroporphyrinogen Ill, cytoplasm D. Hydroxymethylbilane, mitochondria E. Coproporphyrinogen Ill, mitochondria F. Porphobilinogen, cytoplasm G. Coproporphyrinogen Ill, cytoplasm H. Uroporphyrinogen III, mitochondria

*The answer is B.* Accumulation of orotic acid indicates megaloblastic anemia arises because pyrimidines are required for DNA synthesis.

A 6-month-old boy becomes progressively lethargic and pale and shows delayed motor development. Laboratory evaluation reveals normal blood urea nitrogen (BUN), low serum iron, hemoglobin 4.6 g/dL, and leukopenia. His bone marrow shows marked megaloblastosis, which did not respond to treatment with iron, folic acid, vitamin B12, or pyridoxine. His urine developed abundant white precipitate identified as orotic acid. The underlying defect causing the megaloblastic anemia in this child is most likely in which of the following pathways? A. Homocysteine metabolism B. Pyrimidine synthesis C. Urea synthesis D. Uric acid synthesis E. Heme synthesis

*The answer is D.* This patient has Lesch-Nyhan syndrome, an X-linked recessive disorder characterized by the development of dystonia, choreoathetosis, self-mutilation, and hyperuricemia within the first few years of life. The condition is caused by a deficiency of hypoxanthine-guanine phosphoribosyltransferase (HGPRT), an enzyme that normally functions in the purine salvage pathway to convert hypoxanthine back to inosine monophosphate and guanine back into guanosine monophosphate. The absence of HGPRT results in increased degradation of guanine and hypoxanthine bases into uric acid, which increases the demand for de novo purine synthesis. The first step of purine synthesis is the formation of phosphoribosyl pyrophosphate (PRPP) by PRPP synthetase. PRPP can be used by adenine phosphoribosyltransferase and HGPRT for purine salvage, or it can be converted to phosphoribosylamine by PRPP amidotransferase in the first committed step of de novo purine synthesis. Because purine salvage is impaired in Lesch-Nyhan syndrome, the activity of PRPP amidotransferase must increase to supply a sufficient quantity of purine nucleotides.

A 6-month-old boy is brought to the office by his mother out of concern that he is not developing normally. He has been feeding regularly and has had no medical problems other than mild respiratory infection a month earlier. However, the mother says "he doesn't seem to be interactive as my other children were at his age." Physical examination reveals delayed developmental milestones and hypotonia. Two years later, this child is found to have involuntary movements and demonstrates a tendency to aggressively bite his own lips and fingers. Laboratory analysis shows an elevated blood uric acid level. Activity of which of the following enzymes is most likely increased as a result of this patient's condition? (A) Aspartate carbamoyltransferase (B) Dihydroorotase (C) Hypoxanthine-guanine phosphoribosyltransferase (D) Phosphoribosylpyrophosphate amidotransferase (E) Ribonucleotide reductase (F) Thymidylate synthase

*The answer is A.*The increase in dATP, which occurs when ADA is defective, leads to the binding of dATP to the allosteric activity site of ribonucleotide reductase, which leads to the inhibition of overall enzyme activity. Thus, deoxyribonucleotides cannot be produced for the synthesis of DNA, and cells are not capable of replication. Elevated levels of dATP do not have an inhibitory effect on purine nucleoside phosphorylase, hypoxanthine-guanine phosphoribosyltransferase, APRT, or ADA.

A 6-month-old boy was brought to the pediatrician due to frequent bacterial and viral infections. Blood work shows the complete absence of B and T cells. Radiographic analysis shows a greatly reduced thymic shadow. Treatment of the child with a stabilized protein reverses the deficiencies One possible reason for the lack of immune cells is inhibition of which of the following enzymes? (A) ADA (B) Purine nucleoside phosphorylase (C) Hypoxanthine guanine phosphoribosyltransferase (D) Adenine phosphoribosyltransferase (E) Ribonucleotide reductase

*The answer is C.* Due to the lack of ADA activity, adenosine accumulates and is converted to AMP by adenosine kinase (and deoxyadenosine is converted to dAMP). The dAMP will eventually be converted to dATP, which accumulates within the cell. There is no accumulation of dUTP, dCTP, dGTP, and dTTP under these conditions. Adenosine and deoxyadenosine levels are also high in the blood, as all tissues of the body release these compounds when they can be no longer metabolized, due to ADA deficiency. This leads to accumulation of these toxic intermediates in the lymphocytes, which are the tissues that manifest the clinical aspects of the disease.

A 6-month-old boy was brought to the pediatrician due to frequent bacterial and viral infections. Blood work shows the complete absence of B and T cells. Radiographic analysis shows a greatly reduced thymic shadow. Treatment of the child with a stabilized protein reverses the deficiencies. Which metabolite will accumulate in the blood cells? (A) dUTP (B) dCTP (C) dATP (D) dGTP (E) dTTP

*The answer is C.* The patient has the symptoms of ADA deficiency, which leads to severe combined immunodeficiency syndrome. ADA catalyzes the conversion of adenosine to inosine. IMP dehydrogenase converts IMP to XMP. APRT converts adenine to AMP. HGPRT converts guanine to GMP, and there is no enzyme that can convert guanosine to inosine in one step (guanase can convert guanine to xanthine in one step, but does not work on nucleoside substrates).

A 6-month-old boy was brought to the pediatrician due to frequent bacterial and viral infections. Blood work shows the complete absence of B and T cells. Radiographic analysis shows a greatly reduced thymic shadow. Treatment of the child with a stabilized protein reverses the deficiencies. This protein has which of the following activities? (A) Converts IMP to XMP (B) Converts adenine to AMP (C) Converts guanine to GMP (D) Converts adenosine to inosine (E) Converts guanosine to inosine

*The answer is C.* The child has hereditary orotic aciduria, a mutation in the UMP synthase that leads to orotic acid accumulation in the urine. Treatment with uridine bypasses the block and allows UTP, CTP, and dTTP synthesis. Uridine treatment also has the beneficial effect of blocking further orotate production, as UTP inhibits carbamoyl phosphate synthetase II, the rate-determining step of pyrimidine production. As CPS-II is inhibited, less orotate is produced. The megaloblastic anemia is the result of inadequate DNA synthesis in the red cell precursors due to the lack of dTTP and dCTP. The crystals are made of orotate, as that is the compound that is accumulating. Uracil, thymine, and cytosine would not be synthesized in a patient with this disorder. Aspartate is very soluble and would not form crystals if it were to accumulate.

A 6-month-old infant is seen by the pediatrician for developmental delay. Blood work shows megaloblastic anemia, although measurements of B12 and folate are in the high normal range. Urinalysis demonstrates, upon standing, the formation of a crystalline substance. Supplementation of the child's diet with uridine reversed virtually all of the clinical problems. The crystalline substance was most likely composed of which of the following? (A) Uracil (B) Thymine (C) Orotate (D) Aspartate (E) Cytosine

*The answer is D.* When UMP synthesis is inhibited, there are insufficient precursors for dTMP synthesis (which is derived from dUMP via the thymidylate synthase reaction). The lack of dTTP (which is derived from dTMP) leads to an inhibition of DNA synthesis in red blood cell precursors, leading to the megaloblastic anemia. The mutation in hereditary orotic aciduria does not affect folate or B12 metabolism. Since this is a mutation in a pyrimidine biosynthetic pathway, there is no effect on adenine synthesis. This mutation also does not affect the activity of ribonucleotide reductase.

A 6-month-old infant is seen by the pediatrician for developmental delay. Blood work shows megaloblastic anemia, although measurements of B12 and folate are in the high normal range. Urinalysis demonstrates, upon standing, the formation of a crystalline substance. Supplementation of the child's diet with uridine reversed virtually all of the clinical problems. The observed megaloblastic anemia results from which of the following? (A) Interference with folate metabolism (B) Interference with B12 absorption (C) Inhibition of ribonucleotide reductase (D) Lack of thymidine for DNA synthesis (E) Lack of adenine for DNA synthesis

*The answer is C.* The child is exhibiting the symptoms of adenosine deaminase defi ciency, an inherited immunodeficiency syndrome that is a cause of severe combined immunodeficiency. The disease is caused by the lack of adenosine deaminase (a gene found on chromosome 20), which converts adenosine to inosine (part of the salvage and degradative pathway of adenosine. This disorder leads to an accumulation of deoxyadenosine and S-adenosylhomocysteine, which are toxic to immature lymphocytes in the thymus. The amino group at position 6 is deaminated and is replaced by a double bond oxygen, to produce the base hypoxanthine, and the nucleoside inosine. The same type of reaction occurs in tRNA anticodons, in which a 5′ position adenine is converted to hypoxanthine, to produce the nucleoside inosine. Inosine is a wobble base pair former, having the ability to base pair with adenine, uracil, or cytosine.

A 6-month-old infant, with a history of chronic diarrhea and multiple pneumonias, is seen again by the pediatrician for a possible episode of pneumonia. The chest X-ray shows a pneumonia, but also reveals an abnormally small thymus. Blood work shows a distinct lack of circulating lymphocytes. The most likely inherited enzymatic defect in this child leads to an inability to alter a purine nucleotide at which position of the ring structure? (A) 1 (B) 3 (C) 6 (D) 7 (E) 8

*The answer is E.* The boy has porphyria, a reduced ability to synthesize heme. The supersensitivity to the sun is due to the presence of heme precursors in skin cells that are easily converted to radical form by the energy in sunlight, and which severely damage the cell. The drug the boy is taking is metabolized via a cytochrome P450 system, which is induced when the drug first enters the circulation. Induction of P450 systems induces the synthesis of heme, leading to increased concentrations of the heme intermediates and an increased sensitivity to the effects of these intermediates as induced by sunlight. The anemia is due to reduced heme levels in the red blood cells. This disorder is not due to defects in DNA repair, glycogen metabolism, or fatty acid metabolism.

A 6-year-old boy is slightly anemic and is very sensitive to the sun, to the point where his skin blisters instead of healing normally from sunburn. His condition worsened when he was taking rifampin for a Methicillin Resistant Staph Aureus. The boy most likely has a defect in which of the following biochemical pathways? (A) Glycogen synthesis (B) Fatty acid oxidation (C) DNA repair (D) Transcription-coupled DNA repair (E) Heme synthesis

*The answer is A.* Homocystinuria is an inborn error of metabolism caused by a defect in cystathionine synthase, the enzyme that converts homocysteine to cystathionine. In addition to Marfan-like features and subluxation of the lens, these patients are at increased risk of a variety of cardiovascular derangements, including premature vascular disease and death. Atherosclerosis is rare in children. Atherosclerosis in a ch ild is highly suggestive of homocystinuria.

A 6-year-old girl exhibits vision problems during a screening at school, and the school nurse t ells the parents the child should be fitted for glasses. Her mother is upset, because her daughter is already much taller than her classmates, has an awkward gait, and was recently diagnosed with scoliosis. She is afraid that the glasses will only add to her daughter's problems at school, where her classmates frequently tease her. When the ophthalmologist observes that the patient 's right lens is dislocated, he suspects that her symptoms are in fact related to an enzyme deficiency. This patient most likely has difficulty metabolizing which substrate? A. Homocysteine B. Lysine C. Phenylalanine D. Tryptophan E. Tyrosine

*The answer is A.* The boy likely suffers from recurrent nephrolithiasis, during which a stone lodges in the urethra. The elevated creatinine level is indicative of post renal failure from the stone obstruction. The increased levels of lysine and arginine in the urine are characteristic of cystinuria, a defect in amino acid reabsorption in the proximal tubule, leading to wasting of lysine, arginine, cystine, and ornithine. The elevated cystine level in the urine leads to recurrent stone formation. Treatment involves hydrat ion and alkalization of the urine with acetazolamide.

A 7-year-old boy is brought to the emergency department by his parents because of severe pelvic pain. His mother reports that he has had similar episodes of pain in the past, but that they were not as severe and went away in a few hours. His creatinine level is 1. 7 mg/dL, and levels of lysine and arginine in the urine are elevated. A defect in which protein is likely to be the cause of this patient's symptoms? A. Amino acid transporter in the proximal tubule B. Calcium sensor in the parathyroid glands C. Parathyroid hormone receptor D. Vitamin D receptor E. Xanthine oxidase

*The answer is B.* The patient described in the question stem is consistent with the clinical manifestations of homocystinuria. Homocystinuria is the most common inborn error of methionine metabolism and is caused by cystathionine synthetase deficiency. Thromboembolic episodes involving both large and small vessels, especially those of the brain, are classically associated with this condition and may occur at any age. Other clinical manifestations resemble those of Marfan syndrome. These include ectopia lentis, elongated limbs, arachnodactyly, and scoliosis. About 50% of patients with homocystinuria respond dramatically to pyridoxine (Be) supplementation the only treatment available at this time. *Educational Objective:* Homocystinuria is caused by cystathionine synthetase deficiency. Affected individuals manifest with skeletal abnormalities resembling those of Marfan syndrome. In addition, they are also at high risk of developing thromboembolism. About 50% of affected patients respond to high doses of vitamin B6 (pyridoxine).

A 6-year-old male with vision problems and mental retardation dies of a massive stroke. Autopsy reveals middle cerebral artery thrombosis and old renal infarcts. Which of the following supplements could have prevented this patient's death? A. Tyrosine B. Pyridoxine C. Thiamine D. Ascorbic acid E. Methionine F. Vitamin K

*The answer is C.* This patient has neurologic symptoms consistent with vitamin B12 (cobalamin) deficiency, which gradually causes demyelination of the dorsal columns, spinocerebellar tract, and lateral corticospinal tract. Pernicious anemia is a common cause of vitamin B12 deficiency and is characterized by chronic atrophic gastritis and autoantibodies directed against gastric parietal cells. As a result, the parietal cells cannot produce normal amounts of intrinsic factor. Without intrinsic factor, vitamin B12 cannot be absorbed in the ileum. Patients may present with symptoms of anemia such as fatigue, dyspnea, and tachycardia. In addition, patients may have symptoms of achlorhydria such as GERD, early satiety, constipation, and weight loss. A peripheral blood smear will show macrocytic RBCs with hypersegmented polymorphonuclear leukocytes, consistent with megaloblastic anemia (as seen in the vignette image). Both methylmalonic acid and homocysteine levels are elevated in B12 deficiency. It is imperative to check for late and vitamin B12 levels before beginning treatment with vitamin B12 injections, because giving the wrong treatment may exacerbate symptoms. Also, note that genetic defects such as orotic aciduria can cause a megaloblastic anemia that does not respond to the administration of folate and vitamin B12. Causes of macrocytic anemia that are not megaloblastic include alcoholism, liver disease, reticulocytosis, and medications such as zidovudine.

A 60-year-old woman with rheumatoid arthritis presents to her doctor with a 2-month history of progressive fatigue. She also reports feeling "wobbly" lately. Physical examination reveals a pulse of 101/min and decreased light touch and vibration sense on her lower extremities. Laboratory studies show a hemoglobin level of 9 g/dL and a mean corpuscular volume of 110 fL. A peripheral blood smear for the patient is shown in the image. On further analysis, methylmalonic acid levels and homocysteine levels are elevated. The patient 's anemia most likely results from which of the following factors? A. A vitamin deficiency resulting from medication administered for her rheumatoid arthritis B. Abnormal neural crest cell migration C. Antibodies against parietal cells D. Bacterial overgrowth in the colon E. Partially occlusive embolus to the superior mesenteric artery

*The answer is D.* Sideroblastic anemia in a person being treated for tuberculosis (with isoniazid) is most likely due to vitamin B6 deficiency. δ-Aminolevulinate synthase, the first enzyme in heme synthesis, requires vitamin B6 (pyridoxine).

A 62-year-old man being treated for tuberculosis develops a microcytic, hypochromic anemia. Ferritin levels are increased, and marked sideroblastosis is present. A decrease in which of the following enzyme activities is most directly responsible for the anemia in this man? A. Cytochrome oxidase B. Cytochrome P450 oxidase C. Pyruvate kinase D. δ-Aminolevulinate synthase E. Lysyl oxidase

*The answer is E.* All the compounds listed can produce crystals in joint fluid, but only monosodium urate (associated with gout) and calcium pyrophosphate dihydrate (associated with CPPD crystal deposition disease, also called pseudogout), and to lesser degree basic calcium phosphate (apatite-associated arthropathy), have a high likelihood of being encountered on the USMLE. The crystals described are those of monosodium urate.

A 64-year-old man comes to the physician because of a painful right knee. A small amount of fluid is aspirated from the patient's knee joint. Examination of the fluid shows a yellow-white and cloudy substance and contains 200,00 WBC/mm3 (85% neutrophils). Needle-shaped, strongly negatively birefringent crystals are seen both within and outside neutrophils. These crystals most likely have which of the following compositions? A. Basic calcium phosphate B. Calcium oxalate C. Calcium pyrophosphate dihydrate D. Cholesterol E. Monosodium orate

*The answer is A.* Progressive peripheral neuropathy. A distractor may be D, but this would be the cause of a B12 deficiency, not a result of it.

A 64-year-old woman is seen by a hematologist for evaluation of a macrocytic anemia. The woman was severely malnourished. Both homocysteine and methylmalonate were elevated in her blood and urine, and the transketolase level in her erythrocytes was below normal. In response to a B12 deficiency, which of the additional conditions may develop in this patient if she is not treated? A. Progressive peripheral neuropathy B. Gout C. Wernicke-Korsakoff D. Destruction of parietal cells E. Bleeding gums and loose teeth

*The answer is B.* Methylmalonyl-CoA mutase requires B12 but not folate for activity. Macrocytic anemia, elevated homocysteine, and macrocytic anemia can be caused by either B12 or folate deficiency.

A 64-year-old woman is seen by a hematologist for evaluation of a macrocytic anemia. The woman was severely malnourished. Both homocysteine and methylmalonate were elevated in her blood and urine, and the transketolase level in her erythrocytes was below normal. What is the best evidence cited that the anemia is due to a primary deficiency of cyanocobalamin (B12)? A. Macrocytic anemia B. Elevated methylmalonate C. Low transketolase activity D. Elevated homocysteine E. Severe malnutrition

*The answer is B.* The patient most likely has pancreatic adenocarcinoma that is located at the head of the pancreas, leading to obstruction of the common bile duct. Weight loss, painless jaundice, and a palpable gallbladder (Courvoisier sign) can occur in pancreatic cancer. The obstruction from the growing tumor results in conjugated hyperbilirubinemia, increased urine bilirubin levels, and decreased urine urobilinogen levels. The majority of bilirubin results from the breakdown of heme groups in senescent RBCs. After cellular release, bilirubin binds to albumin, which delivers the molecule to the liver. Hepatocellular uptake and glucuronidation in the endoplasmic reticulum generate conjugated bilirubin, which is water soluble and excreted in the bile. Gut bacteria deconjugate the bilirubin and degrade it to urobilinogens. The urobilinogens are excreted in the feces, with some reabsorption and excretion into urine. Based on this metabolic schema, the laboratory values in obstructive liver disease become evident. Failed excretion of conjugated bilirubin leads to a direct bilirubinemia. Failure of urobilinogen production by gut flora leads to low levels of urine urobilinogen. The urine bilirubin level is elevated secondary to the increased plasma concentration of direct bilirubin, which undergoes renal excretion.

A 65-year-old African-American man presents to his physician because of jaundice. He says that in the past few months he has not had much of an appetite and has lost 13.6 kg (30 lb). Physical examination is notable for a gallbladder that is palpable. What set of characteristics is expected in this patient? (A) A (B) B (C) C (D) D (E) E

*The answer is D.* The patient has a deficiency of B12, caused by inadequate intrinsic factor production for the absorption of dietary vitamin B12, which is required for the conversion of methylmalonyl-CoA to succinyl-CoA and of homocysteine to methionine. A vitamin B12 deficiency results in the excretion of methylmalonic acid in the urine and an increased dietary requirement for methionine. The methyl group transferred from vitamin B12 to homocysteine to form methionine comes from N5-methyl-FH4, which accumulates in a vitamin B12 deficiency, causing a decrease in free folate levels and symptoms of folate deficiency, including increased levels of FIGLU and decreased purine biosynthesis. Both B12 and folate deficiencies will lead to anemia, but only a B12 deficiency will give rise to the mental-status changes and tingling in the hands and feet. As individuals age, a variety of conditions can give rise to reduced intrinsic factor production by the stomach. *Since intrinsic factor is low, B12 needs to be delivered by injection, not by the oral route of administration.*

A 65-year-old man visits his primary physician because of tingling in his hands and feet, and a sense that he is forgetting things more than usual. A CBC indicates a mild anemia. The patient states that his diet has not changed, other than eating more red meat than before. This patient can be best treated by which one of the following? (A) Oral administration of vitamin B12 (B) Oral administration of folic acid (C) Oral administration of methionine (D) Injections of B12 (E) Injections of folic acid (F) Injections of methionine

*The answer is D.* This person has homocystinuria, which is commonly caused by a deficiency in the enzyme cystathione beta synthase. Other enzymes that can be deficient include methylenetetrahydrofolate reductase, methionine synthase, and methionine synthase reductase. Therefore, it is possible that methionine can be reduced in this patient.

A 65-year-old man with coronary artery disease comes to the physician for a follow-up examination. He is 183 cm (6 ft) tall and weighs 84 kg (185 lb); BMI is 25 kg/m2. His blood pressure is 130/80 mm Hg. The lungs are clear to auscultation. Cardiac examination shows no point of maximal impulse. Serum studies show a glucose concentration of 95 mg/dL and homocysteine concentration of 19.3 μmol/L (N=5-15). Serum lipid concentrations are within the reference range. Which of the following amino acids is most likely to be decreased in this patient? (A) Arginine (B) Leucine (C) Lysine (D) Methionine (E) Ornithine

*The answer is D.* Hydroxyproline is a hallmark of collagen.

A 67-year-old man has a restricted diet that includes no fresh citrus fruits or leafy green vegetables. His teeth are loose and his gums bleed easily. This patient's disorder most likely results from a defect in collagen synthesis that involves which of the following amino acids? (A) Arginine (B) Cysteine (C) Histidine (D) Hydroxyproline (E) Leucine (F) Methionine (G) Serine (H) Tryptophan

*The answer is B.* The above case describes a patient with Parkinson's disease, which is caused by degeneration of the substantia nigra. This leads to dopamine deficiency in the brain and results in resting tremors, bradykinesia, cog-wheeling of the hand joints, and rigidity of musculature. In addition, patients are often described as having "mask-like facies." Dopamine is one of the catecholamines synthesized in a common pathway with norepinephrine and epinephrine.

A 69-year-old male presents to his family physician with a complaint of recent onset difficulty in performing activities of daily living. He is a retired factory worker who last worked 4 years ago. Upon questioning, his spouse reveals that he "hasn't been able to get around the way he used to." Physical examination reveals a well-nourished 69-year old man who walks with an exaggerated kyphosis. His gait appears to be quite slow and wide-based. He also appears to have a resting tremor. The appropriate management of his case would target which of the following? A. Amino acid degradation B. Catecholamine synthesis C. Ganglioside degradation D. Prostaglandin synthesis E. Sphingolipid degradation

*The answer is A.* Blood glucose in normal subjects does not fall in the hypoglycemic range with fasting due to decreased insulin secretion and the protective actions of multiple counterregulatory hormones. Glucagon is the primary hormone secreted in response to a rapid drop in blood glucose levels, with epinephrine acting as the major backup hormone. Cortisol and growth hormone contribute to glucose homeostasis during prolonged fasting by altering transcription of many key enzymes. In the inactivated state, cortisol receptors are found within the cytoplasm in association with heat shock proteins. Binding of cortisol to the carboxy termini portion of the receptor causes the release of the heat shock proteins and receptor dimerization. The activated homodimers are then transported to the nucleus where they control gene expression by binding to hormone~responive DNA elements in the promoter region of target genes. Cortisol increases the transcription of enzymes involved in gluconeogenesis (formation of glucose from fat and protein substrates) as well as those involved in lipolysis and proteolysis.

A 69-year-old woman with illtlzhelmer disease is brought to the emergency department after her son found her wandering in a local park after being unable to contact her for the last day. The patient says that she got lost while taking a walk and has not eaten or drunk anything for over 24 hours. On examination. she is mildly confused and dehydrated. Laboratory studies show a blood glucose level within the normal range despite her prolonged fasting. Which of the following hormones contributes to this patient's laboratory findings by binding to an intracellular receptor? A. Cortisol B. Epinephrine C. Glucagon D. Growth hormone E. Insulin F. Norepinephrine

*The answer is D.* This is a classic description of maple syrup urine disease. In this disease, there is a defect in the branched-chain a-keto acid dehydrogenase complex (BCKD). BCKD is responsible for the metabolism of the branched chain amino acids leucine, valine, and isoleucine, as well as odd-numbered fatty acids. As a result, these metabolites build up in the system. Signs and symptoms of maple syrup urine disease include encephalopathy and a characteristic "Maple Syrup" smell in the urine. BCKD requires many cofactors including vitamins B1, B2, B3, CoA (from BS) and lipoic acid. These same cofactors are required by pyruvate dehydrogenase, which converts pyruvate to acetyl-CoA.

A 7-day-old female infant is brought to the emergency department for excessive sleepiness. The child was born to a G3P2—3 mother via vaginal delivery. There were no complications during pregnancy or delivery and all of the prenatal testing was normal. The infant was discharged from the hospital With her mother when she was three days old. Over the past two days, the infant has been feeding poorly and today was difficult to rouse from sleep. Family history is unremarkable. Vital signs are as follows: temperature 37'C (98.62F), heart rate 160 beats/min, blood pressure 72/40 mmHg, respiratory rate 70 breaths/min, and oxygen saturation 100% on room air. On exam, she is minimally responsive. There is no heart murmur and lungs are clear to auscultation bilaterally Her abdomen is sot non-tender and non-distended. There are no rashes or jaundice_ You ask the nurse to draw blood and obtain a urine specimen. The nurse comments that the urine smells like burnt sugar. A complete blood cell count and differential are normal. There are 0 white blood cells in the cerebral spinal fluid and protein and glucose are normal. The urine is positive for ketones, but the leukocyte esterase and nitrites are negative. The enzyme that is defective in this patient is responsible for the metabolism of which amino acid? A. Phenylalanine B. Homocysteine C. Galactose D. Leucine E. Lactose

*The answer is B.* Maple syrup urine disease (MSUD) is caused by a defect in the branched-chain α-keto acid dehydrogenase complex. This enzyme is responsible for the catabolism of the branched-chain amino acids, leucine, isoleucine, and valine. Therefore, restriction of dietary intake of these three amino acids is the treatment for MSUD.

A 7-day-old infant is brought to his pediatrician because of lethargy and feeding problems. He is admitted to the hospital for failure to thrive. Shortly after admission, the infant has a seizure. A biochemistry evaluation reveals a deficiency in α-ketoacid dehydrogenase. To prevent intellectual disability and death, intake of which of the following amino acids should be restricted in this patient? A. Alanine B. Leucine C. Threonine D. Tryptophan E. Tyrosine

*The answer is E.* The constellation of marfanoid body habitus, brittle bones, intellectual disability, and visual problems are all clinical features suggestive of homocystinuria. Classic homocystinuria results from the reduced activity of cystathionine E-synthase (CBS), which normally condenses homocysteine with serine to form cystathionine. Serum testing of a patient with homocystinuria reveals elevated homocysteine and methionine. When CBS activity is deficient, homocysteine produced in the methionine salvage pathway can only be converted back into methionine. Because homocysteine can be elevated in other types of errors of metabolism, elevated methionine is a more specific screening test for CBS deficiency. Homocystinuria is caused by an autosomal recessive deficiency of CBS, a pyridoxine-dependent enzyme that converts homocysteine to cystathionine. In cases with phenotypic variants that are pyridoxine (vitamin B6) responsive, vitamin B6 supplementation can prevent many features of the disease. In developed countries, routine newborn testing screens for homocystinuria, and, as such, most cases that present later in life are due to a lack of access to routine medical care or cases of pyridoxine responsive variants of the disease- Patients with B6 responsive homocystinuria not detected with newborn screening often present in adolescence with thromboembolic strokes. In addition to the findings in this patient, patients with homocystinuria can also present with dislocation of the ocular lens (ectopia lentis), a variety of psychiatric manifestations, seizures, and often have clinically detectable osteoporosis by the teenage years.

A 7-year-old boy is brought to the pediatrician for the first time since immigrating to the United States. His mother reports that he has started falling and running into things. The son says that he sees differently" now. She also reports multiple fractures from minor falls on several occasions. He has always had difficulty learning in school. Examination shows a pleasant young boy wearing thick glasses with a thin body habitus, disproportionately long limbs, and mild kyphosis. The remainder of the physical examination is unremarkable. Genetic testing reveals a deficiency in cystathionine beta-synthase (CBS) activity. Laboratory studies will most likely show a serum elevation of which substance? A. Folate B. Cobalamin C. Pyridoxine D. Cysteine E. Methionine

*The answer is A.* Many clinical features that occur due to decreased hemoglobin are common for all types of anemia. These features include malaise, increased fatigability, pallor, decreased exercise capacity and congestive heart failure. The symptoms of difficulty in swallowing (dysphagia) and disfigured fingernails (spoon nails or koilonychia), as shown in the above photograph, are more specific for iron deficiency anemia. Dysphagia in a patient with iron deficiency anemia is often caused by the formation of esophageal webs and is described as Plummer-Vinson or Patterson Kelly syndrome. Red cells am microcytic and hypochromic in iron deficiency. Iron deficiency anemia is typically treated with an oral iron preparation. Vitamin C is sometimes added to improve oral iron absorption. *Educational Objective:* The symptoms of difficulty in swallowing (dysphagia) and disfigured fingernails (spoon nails or koilonychia) are specific for iron deficiency anemia.

A 72-year-old Caucasian female presents to your office complaining of difficulty in swallowing. She also complains of severe fatigue and progressive exertional dyspnea. Physical examination reveals pale conjuctiva. A photograph of her finger is shown below. Which of the following is the best treatment for this patient? A. Iron preparations B. Vitamin B12 C. Pyridoxine D. Vitamin C E. Folic acid F. Erythropoietin G. Filgrastim

*The answer is D.* Vitamin B12 is a necessary cofactor in the regeneration of folate for methyl group donation in DNA synthesis. For this reason, vitamin B12 and folate deficiencies each present with megaloblastic anemia and hypersegmented neutrophils, as all blood cell lines are affected by the defect in DNA synthesis. However, only vitamin B12 deficiency increases serum methylmalonic acid levels and impairs myelin synthesis. This myelin defect primarily impacts the posterior and lateral spinal columns, causing paresthesias and impaired proprioception. Thus, neurologic abnormality in the context of megaloblastic anemia is a USMLE-favorite hint that vitamin B12, rather than folate, is deficient.

A 78-year-old man with asthma presents to his primary care physician for an annual checkup. The physician performs a physical examination and orders routine blood work, which reveals a macrocytic anemia. Subsequent laboratory tests show an elevated serum methylmalonic acid level. A peripheral blood smear is shown in the image. If this patient's vitamin deficiency is not corrected, what neurological symptoms is he most likely to experience? (A) Confusion and confabulation (B) Deficiency in this vitamin does not cause neurological symptoms (C) Dysarthria and diplopia (D) Paresthesias and ataxia (E) Syncope and lethargy

*The answer is C.* The child has inherited mutations in the genes for adenosine deaminase, and cannot convert adenosine to inosine (and deoxyadenosine to deoxyinosine). The deoxyadenosine is toxic and will accumulate in the blood cells, eventually forming dATP through salvage reactions. The dATP will, in part, inhibit ribonucleotide reductase, and the cells with the high dATP levels will not be able to proliferate when signaled to do so due to the lack of deoxyribonucleotide precursors. This is a form of SCID. Orotic acid builds up in hereditary orotic aciduria, but immune defects are not associated with that condition. Uric acid accumulation leads to gout without affecting the formation of the immune system. NADPH is required for the ribonucleotide reductase reaction, but its levels are not altered, nor is it secreted into the blood in an adenosine deaminase deficiency. dGTP levels do not increase with an adenosine deaminase deficiency.

A 9-month-old infant had been in and out of the hospital owing to frequent infections. Blood work demonstrated the virtual lack of B and T cells, and the almost complete absence of a thymic shadow on a chest X-ray. Measurement of metabolites in the blood would be expected to show elevated levels of which one of the following? (A) Uric acid (B) Orotic acid (C) Deoxyadenosine (D) NADPH (E) dGTP

*The answer is D.* The child has PKU. The elevated phenylalanine levels in the blood are saturating the large, neutral amino acid transport protein in the nervous system (L-system), preventing other substrates from entering the brain (such as tryptophan, tyrosine, lysine, and leucine). This alters the ability of the brain to synthesize proteins, and leads to neurological problems. Providing large amounts of these large, neutral amino acids prevents saturation of the system by phenylalanine, and can be used as a treatment, along with restricted phenylalanine diet, for children with this disorder. The developmental delay does not appear to be due to acidosis, lack of tyrosine, an inhibition of hydroxylating enzymes, or inhibition of neuronal glycolysis.

A Russian child, 5 years old, was brought to the pediatrician for developmental delay. Blood analysis showed elevated levels of phenylalanine, phenyllactate, and phenylpyruvate. The developmental delay, in this condition, has been hypothesized to occur due to which of the following? (A) Acidosis due to elevated phenyllactate (B) Lack of tyrosine, now an essential amino acid (C) Inhibition of hydroxylating enzymes due to accumulation of phenylalanine (D) Lack of large, neutral amino acids in the brain (E) Inhibition of neuronal glycolysis by phenylpyruvate

*The answer is E.* Collagen has an unusual amino acid composition in that approximately one-third of collagen molecules are glycine. The amino acid proline is also present in a much greater amount than in other proteins. In addition, two somewhat unusual amino acids, 4-hydroxyproline and 5-hydroxylysine, are found in collagen. Hydroxyproline and hydroxylysine per se are not incorporated during the synthesis of collagen. Proline and lysine are hydroxylated by specific hydroxylases after collagen is synthesized. A reducing agent such as ascorbate (vitamin C) is needed for the hydroxylation reaction to occur. In its absence, the disease known as scurvy occurs. Only proline or lysine residues located on the amino side of glycine residues are hydroxylated. The hydroxylysine residues of collagen are important sites of glycosylation of disaccharides of glucose and galactose.

A child with a large head, multiple fractures, and blue scleras (whites of the eyes) is evaluated for osteogenesis imperfecta. One study involves labeling of collagen chains in tissue culture to assess their mobility by gel electrophoresis. Amino acids labeled with radioactive carbon 14 are added to the culture dishes in order to label the collagen. Which of the following amino acids would not result in labeled collagen? a. Serine b. Glycine c. Aspartate d. Glutamate e. Hydroxylserine

*The answer is C.* The only pyrimidine that requires folate for its synthesis is thymine (dUMP → dTMP). Folate is required for the incorporation of carbons 2 and 8 into all purine molecules. The synthesis of creatine phosphate and of phosphatidylcholine do not require folate. Folate deficiencies during pregnancy can lead to neural tube defects (e.g., spina bifida) in the fetus. Deoxyribonucleotide synthesis requires ribonucleotide reductase, which uses thioredoxin, and does not require a folate derivative.

A common polymorphism in the US population is a variant of N5,N10-methylene- FH4 reductase, which has a reduced activity at 37°C as compared to 32°C. A person expressing this variant enzyme would have difficulty producing which one of the following at the nonpermissive temperature? (A) Creatine phosphate from creatine (B) Pyrimidines required for RNA synthesis (C) The thymine nucleotide required for DNA synthesis (D) Phosphatidylcholine from diacylglycerol and CDP-choline (E) All deoxyribonucleotides

*The answer is A.* Trypsin cleaves and, thus, activates the pancreatic zymogens, converting chymotrypsinogen to the active form, chymotrypsin, and the procarboxypeptidases to the active carboxypeptidases. If trypsin were inactive, the other proteases could not be activated, as enteropeptidase is specific for trypsinogen. Pepsin is found in the stomach, whereas aminopeptidases are intestinal enzymes found on the brush border membrane, facing the lumen of the intestine.

A deficiency of which one of the following proteolytic enzymes would have the greatest effect on the digestion of proteins? (A) Trypsin (B) Chymotrypsin (C) Carboxypeptidase A (D) Pepsin (E) Aminopeptidase

*The answer is E.* The infant in this vignette presents with an elevated ammonia level, low blood urea nitrogen level, and respiratory alkalosis. If any portion of the urea cycle is impaired, urea will not be formed in adequate amounts and nitrogenous waste in the form of ammonia will build up in the system. If not promptly treated, irreversible neurologic deterioration and even death may occur. The most common defect of the urea cycle is a deficiency of ornithine transcarbamoylase. It is inherited in an X-Iinked recessive fashion. Ornithine transcarbamoylase is found within the mitochondrial matrix and is responsible for the condensation of ornithine and carbamoyl phosphate.

A four-day-old male infant is brought to the emergency department for feeding difficulty. The child was born to a GIPO—I mother via spontaneous vaginal delivery following an uneventful pregnancy. He was initially well and was discharged from the hospital the day prior. Over the past twenty-four hours, the infant has refused to feed and has had several episodes of vomiting. The vomiting is non-bilious. He has not had any fevers or rash. He passed meconium several times before leaving the hospital but has had a decreased number of wet diapers and bowel movements over the past day. Family history is notable for the death of a maternal uncle in infancy. Vital signs are as follows: temperature 37'C (98.6'F), heart rate 130/min, blood pressure 72140 mmHg, respiratory rate 64/min, and oxygen saturation 100% on room air. On exam, he is comatose and his breathing is fast and irregular. There is no heart murmur and lungs are clear to auscultation bilaterally. His abdomen is soft, non-tender, and non-distended. There are no rashes or jaundice. Initial lab results show an elevated plasma ammonia concentration and a normal blood glucose level. A subsequent quantitative plasma amino acid analysis reveals undetectable citrulline and arginine concentrations. Urine orotic acid is 1050 micromoVmol creatinine (normal: 1-11 micromol/mol creatinine). A complete blood cell count and the differential is within normal limits. A liver biopsy is performed and enzyme activity' analysis confirms the diagnosis. Where does the reaction catalyzed by the defective enzyme occur? A. Ribosome B. Rough ER C. Nucleus D. Cytosol E. Mitochondrial matrix

*The answer is F.* The 3 main circulating catecholamines am dopamine, norepinephrine, and epinephrine. Norepinephrine and dopamine ere produced in the central as well as the peripheral nervous system, whereas epinephrine is predominantly produced in the adrenal medulla. The first step in the synthesis of catecholamines is the conversion of tyrosine to dihydroxyphenylalanine (DOPA) by tyrosine hydroxylase (Choice G). This is the rate-limiting step in the synthesis of catecholamines. DOPA is converted to dopamine by dope decarboxylase (Choice B), which is then converted to norepinephrine by dopamine beta-hydroxylase (Choice C). In the adrenal medulla, norepinephrine is rapidly converted to epinephrine by phenylethanolamine-N-methyltransferase (PNMT). Expression of PNMT in the adrenal medulla is upregulated by cortisol. Because the venous drainage of the adrenal cortex passes through the adrenal medulla, cortisol concentrations in the medulla can be very high, and PNMT is expressed at a high level. However, following pituitary resection, ACTH secretion and subsequent cortisol production would be low. The result would be decreased PNMT activity and reduced conversion of norepinephrine to epinephrine.

A group of investigators is studying the regulation of catecholamine synthesis in response to severe stress. In the experiments, subject rats are randomly assigned to either an experimental or control group. The experimental rats undergo resection of the pituitary gland. and the control rats undergo craniotomy without pituitary resection. The experimental animals are found subsequently to have decreased production of epinephrine by the adrenal medulla compared with the control animals. Decreased activity of which of the following enzymes is most likely responsible for the observed effect? A. Catechol-O-methyl transferee B. Dopa decarboxylase C. Dopamine beta-hydroxylase D. Monoamine oxidase E. Phenylalanine hydroxylate F. Phenylethanolamine-N-methyltransferase G. Tyrosine hydroxyinse

*The answer is C.* In addition to decreasing the availability of essential amino acids, prolonged starvation and protein deficits tend to reduce the body's ability to synthesize even the "nonessential" amino acids. However, for purposes of the USMLE. you should pick out one of the essential amino acids, which include phenylalanine., valine tryptophan. threonine, isoleucine, methionine, histidine. arginine, lysine, and leucine (mnemonic: PVT TIM HALL). Not all authors include arginine, which is not usually required by the adult unless there is an increased demand for protein synthesis (pregnancy or recovery from negative nitrogen balance, as in the case of these individuals). Arginine is essential in children because they are growing and adding extra protein to their bodies All of the other amino acids listed in the choices are synthesized from glucose.

A local church sponsors a family from Somalia who wants to immigrate to the United States. Upon arrival in the U.S., the family is taken to the physician. Physical examination shows that the children are thin and have very protuberant abdomens with large palpable livers. These children would most likely have the greatest deficiency of which of the following amino acids? A. Alanine B. Glycine C. Leucine D. Proline E. Serine

*The answer is B.* PLP (from vitamin B6) is the major coenzyme of amino acid metabolism, as it participates in amino acid decarboxylation, amino acid racemization, ß-elimination reactions, ß-addition reactions, transaminations, and γ-elimination reactions. PLP is also a required cofactor for glycogen phosphorylase, but the majority of reactions that utilize this cofactor have amino acids as a substrate.

A medical student has been exposed to a patient with tuberculosis and developed a positive tuberculin test (PPD), but exhibited a normal chest X-ray. He is placed on a 6-month course of prophylactic treatment, but subsequently develops peripheral neuropathies. Considering the potential vitamin deficiency in this patient, which class of molecules would be most affected by the lack of the vitamin? (A) Carbohydrates (B) Amino acids (C) Fatty acids (D) Cholesterol (E) Phospholipids

*The answer is B.* The patient has hepatic encephalopathy from liver disease. His protuberant abdomen, palmar erythema, and pronounced gynecomastia are all a result of portal hypertension and liver disease. Hepatic encephalopathy is thougl]t to result f rom high ammonia levels exerting a toxic effect on neurons in the central nervous system. Hyperammonemia occurs because of significant liver damage and portosystemic shunting; the injured liver fails to process ammonia into nontoxic metabolites as it would normally do. Although the precise mechanism by which hyperammonemia leads to hepatic encephalopathy is not well understood, it is well known that high levels of ammonia affect various cell processes. Normally, ammonia is the natural byproduct of amino acid catabolism. In most tissues, the amino group of amino acids destined for degradation is transferred (via aminotransferases) onto α-ketoglutarate, forming glutamic acid. In all tissues (except for muscle cel ls), glutamic acid is further modified by addition of ammonia via glutamine synthetase to form glutamine (as shown in the diagram). Glutamine is the nontoxic carrier of ammonia in the bloodstream, delivering ammonia to the liver for further processing. In the liver, glutamine is deaminated by the enzyme glutaminase to produce glutamate and free ammonia, These substrates enter the urea cycle, whereby ammonia is detoxified into urea, which can then be cleared renally. One of the first steps in the urea cycle is the consolidation of ammonia, carbon dioxide, and adenosine triphosphate (ATP) to form carbamoyl phosphate.

A medical student working in the emergency department admits a 46-year-old man who appears confused and is not oriented to time or place. The student not ices t he man's tense, distended abdomen, palmar erythema, and pronounced gynecomastia. The patient also has a hand tremor. The student believes the patient's mental status to be directly related to elevated levels of a certain metabolite. Which of the following describes a consequence of high levels of this compound? A. Decrease in levels of carbamoyl phosphate B. Decreased levels of α-ketoglutarate C. Decreased product ion of glutamate D. Decreased product ion of glutamine E. Decreased utilization of glutamate

*The answer is C.* This individual suffers from porphyria cutanea tarda. The porphyrias are diseases resulting from enzymatic deficiencies in heme biosynthesis, and porphyria cutanea tarda is the most common form. This disorder is caused by deficiency of uroporphyrinogen decarboxylase, the hepatic enzyme that catalyzes the formation of coproporphyrinogen III from uroporphyrinogen III. Lack of this enzyme results in uroporphyrin accumulation in the urine (giving the urine a tea-colored appearance) and uroporphyrinogen accumulation systemically. This compound absorbs light and releases heat, causing extreme photosensitivity.

A mother brings her 6-month-old son to the pediatrician. She has noticed that he seems "afraid of light" and, after some Internet research, she is concerned that he might be an albino. Laboratory analysis reveals uroporphyrin in his urine. The child most likely has which of the following conditions? (A) Deficiency of coproporphyrinogen oxidase (B) Deficiency of porphobilinogen deaminase (C) Deficiency of uroporphyrinogen decarboxylase (D) Inhibition of ferrochelatase and δ-aminolevulinic acid dehydrase (E) Overexpression of porphobilinogen deaminase

*The answer is C.* Whenever there is a urea cycle defect, arginine becomes an essential amino acid (as its route of synthesis is the urea cycle). Benzoate, along with phenylbutyrate, is given to patients with urea cycle defects to conjugate with a nitrogen carrying molecule (benzoate conjugates with glycine while phenylbutyrate, after activation to phenylacetate, conjugates with glutamine), which is then excreted. The reactions of benzoate and phenylbutyrate with nitrogen containing amino acids are shown above. The excretion of glycyl-benzoate reduces the glycine levels of the body, forcing more glycine to be produced and providing an alternative pathway for nitrogen disposal in the absence of a functional urea cycle. Giving lysine or glutamine will not help to reduce ammonia levels in the patient.

A newborn becomes lethargic and drowsy 24 h after birth. Blood analysis shows hyperammonemia, coupled with orotic aciduria. A potential treatment for the patient is supplementation with which of the following? (A) Arginine and glutamine (B) Lysine and glutamine (C) Arginine and benzoate (D) Lysine and benzoate (E) Glutamine and phenylbutyrate

*The answer is C.* The rate-limiting step for de novo pyrimidine synthesis is carbamoyl phosphate synthetase II (CPS-II), which produces carbamoyl phosphate in the cytoplasm. In an OTC deficiency, the carbamoyl phosphate produced in the mitochondria leaks into the cytoplasm, leading to orotic acid synthesis as the regulated step of the pathway is being bypassed. The elevated ammonia is not a substrate of CPS-II, and while glutamine is also elevated, and is a substrate of CPS-II, higher glutamine concentrations will not overcome enzyme inhibition by its allosteric inhibitor, UTP. Aspartate transcarbamoylase is the regulated step of pyrimidine biosynthesis in many prokaryotic cells, but not in humans. This step is necessary for pyrimidines to be synthesized starting with carbamoyl phosphate. CPS-I is a mitochondrial enzyme not involved in pyrimidine production.

A newborn becomes lethargic and drowsy 24 h after birth. Blood analysis shows hyperammonemia, coupled with orotic aciduria. Orotic acid levels are high in this patient due to which of the following? (A) Elevated ammonia (B) Elevated glutamine (C) Bypassing carbamoyl phosphate synthetase II (CPS-II) (D) Bypassing aspartate transcarbamoylase (E) Inhibition of carbamoyl phosphate synthetase I (CPS-I)

*The answer is C.* The child has ornithine transcarbamoylase (OTC) deficiency, and cannot condense carbamoyl phosphate with ornithine to produce citrulline. The excess carbamoyl phosphate produced leaks into the cytoplasm where it bypasses the regulated enzyme of de novo pyrimidine production, leading to excess orotic acid. Thus, in an OTC defect, carbamoyl phosphate can be produced, but citrulline cannot. Since citrulline cannot be produced, the later products of the urea cycle (argininosuccinate and arginine) are also produced at lower levels than normal, which is an indirect effect due to the inability to produce citrulline.

A newborn becomes lethargic and drowsy 24 h after birth. Blood analysis shows hyperammonemia, coupled with orotic aciduria. This individual has an enzyme deficiency that leads to an inability to directly produce which of the following? (A) Carbamoyl phosphate (B) Ornithine (C) Citrulline (D) Argininosuccinate (E) Arginine

*The answer is D.* The child has albinism, a lack of pigment in the skin cells, which is produced by melanocytes. Melanocyte tyrosinase (a different isozyme than the neuronal tyrosinase that produces DOPA for catecholamine biosynthesis) is defective in albinism. The DOPA produced is then used for pigment production. A lack of phenylalanine hydroxylase leads to PKU. A lack of dihydrofolate reductase is most likely a lethal event as there are no reported cases of a lack of this enzyme. Tetrahydrofolate is not required for the conversion of tyrosine to DOPA in melanocytes. NADPH oxidase generates superoxide, which is not part of this pathway. Homogentisic acid is part of the phenylalanine and tyrosine degradation pathways, and is not involved in albinism.

A newborn has milky white skin, white hair, and red-appearing eye color (see the figure below). This disorder most often results from a defect in which of the following enzymes? (A) Phenylalanine hydroxylase (B) NADPH oxidase (C) Dihydrofolate reductase (D) Tyrosinase (E) Homogentisic acid oxidase

*The answer is E.* Partial blockage of the urea cycle leads to conditions ranging from lethargy and episodic vomiting to mental retardation. A complete block is incompatible with life. A major reason for the toxicity is the severe depletion of ATP levels caused by the siphoning off of α-ketoglutarate from the citric acid cycle in an attempt to consume ammonia. Glutamate dehydrogenase and glutamine synthetase, respectively, catalyze the following reaction: α-ketoglutarate + NH4+ → glutamate + NH4+ → glutamine As can be seen, this is the reverse order of steps whereby glutamine is successively deaminated first to glutamate and then to α-ketoglutarate by the enzymes glutaminase and glutamate dehydrogenase, respectively. It is thought that the high level of ammonia ions shifts the equilibrium of the dehydrogenase in favor of the formation of glutamate. Depending on the step in the urea cycle that is blocked, levels of arginine may be decreased.

A newborn becomes progressively lethargic after feeding and increases his respiratory rate. He becomes virtually comatose, responding only to painful stimuli, and exhibits mild respiratory alkalosis. Suspicion of a urea cycle disorder is aroused and evaluation of serum amino acid levels is initiated. In the presence of hyperammonemia, production of which of the following amino acids is always increased? a. Glycine b. Arginine c. Proline d. Histidine e. Glutamine

*The answer is A.* Jaundice refers to the yellow color of the skin and eyes caused by increased levels of bilirubin in the blood. It has many causes, including increased production of bilirubin due to hemolytic anemia or malaria, blockage in the excretion of bilirubin due to liver damage, or obstruction of the bile duct. In newborns, jaundice is normal (physiologic) due to liver immaturity. Only excess jaundice is evaluated, based on the age of the infant. High levels of bilirubin in serum (indirect bilirubin) points toward hemolysis from maternofetal blood group incompatibility, while high levels of bilirubin diglucuronide (one of several conjugated bilirubins tested as direct bilirubin) suggest liver/gastrointestinal disease. Reticuloendothelial cells degrade red blood cells following approximately 120 days in the circulation. The steps in the degradation of heme include (1) formation of the green pigment biliverdin by the cleavage of the porphyrin ring of heme; (2) formation of the red-orange pigment bilirubin by the reduction of biliverdin; (3) uptake of bilirubin by the liver and the formation of bilirubin diglucuronide; and (4) active excretion of bilirubin into bile and eventually into the stool. The change in color of a bruise from bluish-green to reddish orange reflects the heme degradation and the change in color of the bile pigments biliverdin and bilirubin. Bilirubin, which is quite insoluble, is transported to the liver attached to albumin. In the liver, bilirubin is conjugated to two glucuronic acid molecules to form bilirubin diglucuronide. Bilirubin diglucuronide is transported against a concentration gradient into the bile. If bilirubin is not conjugated, it is not excreted.

A newborn develops jaundice (yellow skin and yellow scleras) that requires laboratory evaluation. Which of the following porphyrin derivatives is conjugated, reacts directly, and is a major component of bile? a. Bilirubin diglucuronide b. Stercobilin c. Biliverdin d. Urobilinogen e. Heme f. Bilirubin g. Urobilin

*The answer is C.* This infant Is most likely suffering from *maple syrup urine disease (MSUD)*, a disorder characterized by the defective breakdown of branched-chain amino acids (leucine, isoleucine, and valine). Degradation of these amino acids first involves transamination to their respective α-keto acids, which are subsequently metabolized by an enzyme complex referred to as branched-chain α-ketoacid dehydrogenase. MSUD can result from mutations in any of the 4 genes coding for the 3 catalytic subunits of this complex. Neurotoxicity results primarily from the accumulation of leucine in the serum and tissues. A metabolite of isoleucine gives the urine of affected infants a distinctive sweet odor much like burned caramel. MSUD can be life-threatening if untreated, but dietary restriction of branched chain amino acids can lessen the severity of symptoms. Branched-chain α-ketoacid dehydrogenase, pyruvate dehydrogenase, and α-ketoglutarate dehydrogenase all require five cofactors *T*hiamine pyrophosphate, *L*ipoate, *C*oenzyme A, *F*AD, *N*AD (mnemonic. *T*ender *L*oving *C*are *F*or *N*ancy). Some patients with MSUD improve with high-dose thiamine treatment (thiamine-responsive), but most still require lifelong dietary restrictions.

A newborn develops vomiting, irritability. and lethargy several days after birth. His mother states that his diapers smell like "burned sugar." Laboratory studies confirm the diagnosis, and the patient is started on the appropriate dietary restrictions with subsequent improvement in his symptoms. The defective enzyme responsible for this patient's condition normally catalyzes a reaction involving which of the following substances? (A) Galactocerebrosidaes (B) Pyridoxine (C) Thiamine (D) Folic acid (E) Tyrosine (F) Tetrahydrobiopterin

*The answer is D.* Catabolism of isoleucine, valine, threonine, methionine, cholesterol, and odd-chain fatty acids leads to formation of propionic acid, which is then converted to methylmalonic acid by biotin dependent carboxylation. Isomerization of methylmalonyl CoA forms succinyl CoA, which subsequently enters the TCA cycle. Defects in this isomerization reaction lead to the development of methylmalonic acidemia. *Educational Objective:* Methylmalonic acidemia (also known as methylmalonic aciduria) results from a defect in the isomerization reaction that transforms methylmalonyl CoA to succinyl CoA, prior to succinyl CoA entering the TCA Cycle.

A newborn experiences lethargy, vomiting, and tachypnea soon after birth. Laboratory examination reveals a metabolic acidosis with a large anion gap, ketosis, and hypoglycemia. The methylmalonic acid concentration is markedly increased in both the plasma and the urine. Which of the following reactions is most likely deficient in this patient? A. Hydrolysis B. Carboxylation C. Hydroxylation D. Isomerization E. Transamination

*The answer is B.* Catabolism of isoleucine. valine, threonine, methionine, cholesterol, and odd-chain fatty acids lead to the formation of propionic acid. which is then converted to methylmalonic acid by biotin-dependent carboxylation. Isomerization of methylmalonyl-CoA forms succinyl CoA. which then enters the TCA cycle. A congenital deficiency of propionyl CoA carboxylase, the enzyme responsible for the conversion of propionyl CoA to methylmalonyl CoA leads to the development of propionic acidemia, as propionyl CoA accumulates. Propionic acidemia is clinically characterized by poor feeding, vomiting, hypotonia, lethargy, dehydration, and an anion gap acidosis. Propionic acid is the intermediate in the catabolism of branched-chain amino acids. such as valine and is not produced during the catabolism of the other amino acids listed.

A newborn experiences lethargy, vomiting. and hypotonia during the first few days of life. Laboratory examination reveals a metabolic acidosis with a large anion gap, ketosis, and hypoglycemia. The concentration of propionic acid is markedly Increased in the plasma and urine. Metabolism of which of the following amino acids contributes to this patient's condition? (A) Phenylalanine (B) Valine (C) Asparagine (D) Histidine (E) Proline (F) Lysine

*The answer is C.* The child has von Gierke disease, a lack of glucose-6 phosphatase activity. The lack of glucose-6-phosphatase leads to glucose 6-phosphate accumulation under conditions of glucose export from the liver, and the excess glucose 6-phosphate has three potential fates synthesize more glycogen (due to allosteric activation of phosphorylated glycogen synthase), produce lactate (glycolysis is forced to be active by high substrate concentrations, and gives rise to lactate formation), and ribose 5-phosphate (through the oxidative reactions of the HMP shunt pathway). As ribose 5-phosphate accumulates, PRPP synthetase will convert the ribose 5-phosphate to PRPP, which then initiates the de novo purine biosynthetic pathway, and the purines are overproduced. With excess purines, they are then degraded to uric acid, which increases in concentration and leads to precipitation, and gout.

A newborn infant develops its first cold, and is fussy and cannot eat. After missing a few feedings, the child becomes quite lethargic, and the parents rush the child to the emergency department. Blood analysis indicates elevated levels of lactate and uric acid, and significantly decreased levels of glucose. After stabilizing the child with glucose infusions, a glucagon challenge is given to the infant, and blood glucose levels do not increase, but decrease slightly. The accumulation of which metabolite in the liver is most responsible for the elevated uric acid seen in the circulation? (A) Glucose (B) Lactate (C) Ribose 5-phosphate (D) Thymidine (E) Acetoacetate

*The answer is A.* This patient has maple syrup urine disease (MSUD), caused by a deficiency of a-ketoacid dehydrogenase. α-Ketoacid dehydrogenase degrades branched-chain amino acids. The classic finding in this disease is a maple syrup odor in patients' urine; patients also show failure to thrive, seizures, lethargy, and severe intellectual disability. Detection of elevated isoleucine in patients' plasma is diagnostic for MSUD. Treatment includes a diet free of branched-chain amino acids. Many states screen for MSUD at birth.

A newborn is brought to the emergency department with failure to thrive, lethargy, and seizures. Urinalysis shows the following: Color: pale yellow Specific gravity: 1.025 pH : 5.0 Glucose: negative Ketones : positive Amino acids: increased leucine and isoleucine Blood: negative This patient most likely has a condition characterized by a defect in which of the following enzymes? A. α-Ketoacid dehydrogenase B. Cystathionine synthase C. Homogentisic acid oxidase D. Phenylalanine hydroxylase E. Positively-charged amino acid transporter

*The answer is D.* This constellation of symptoms is suggestive of Crigler-Najjar syndrome type I , an autosomal recessive disorder characterized by unconjugated hyperbilirubinemia that is usually fatal within 18 months of life secondary to kernicterus (bilirubin deposition in brain tissue- similar to that shown in this image of a monkey brain). Neonates have a relatively leaky blood-brain barrier and are susceptible to kernicterus during jaundice, whereas patients with a mature blood- brain barrier do not develop kernicterus even when they are quite jaundiced . Signs of kernicterus include increasing lethargy and a high-pitched cry. Multiple genetic defects in the gene for bilirubin uridine diphosphate-glucuronosyltransferase (UGT1) that completely abrogate activity can give rise to this disorder. This enzyme is responsible for conjugating bilirubin with glucuronic acid. Mutations that lead to some but less than 20% enzyme activity lead to Crigler-Najjar syndrome type 2, whereas mutations that lead to enzyme impairment but > 20% activity lead to Gilbert syndrome .

A newborn is brought to the pediatrician because of increasing lethargy and jaundice. Her mother says she is very difficult to arouse. Laboratory tests show an elevated serum unconjugated bilirubin level with a normal red blood cell count. The patient is diagnosed with a metabolic disorder and dies soon after her first birthday. This patient most likely had a deficiency in which of the following enzymes? A. Canalicular multispecific organic anion transporter B. Organic anion transporting polypeptide and organic anion transport ing polypeptide 1B3 C. Pyruvate kinase D. Uridine diphosphate-glucuronosyltransferase E. Uroporphyrinogen decarboxylase

*The answer is B.* Phenylalanine is converted to tyrosine by phenylalanine hydroxylase using tetrahydrobiopterin (BH4) as a cofactor. Tyrosine is a non-essential amino acid that becomes essential in the setting of phenylketonuria (PKU). The next step in this pathway is the conversion of tyrosine to DOPA, catalyzed by the enzyme tyrosine hydroxylase, which also uses BH, as a cofactor. When there is deficiency of dihydrobiopterin reductase, this reaction is compromised (the phenylalanine hydroxylase reaction is also compromised, but the effects are unseen in this case because the patient was receiving tyrosine supplementation, thus bypassing the impaired phenylalanine hydroxylase reaction). Once DOPA is synthesized, it is decarboxylated to dopamine by the enzyme DOPA decarboxylase. Dopamine ultimately serves as the precursor molecule to the catecholamines epinephrine and norepinephrine. Deficiency of dihydrobiopterin reductase, the enzyme responsible for the reduction of dihydrobiopterin (BH2) to BH4, is the most common cause for a deficiency of BH4 This results in what is known as atypical or malignant phenylketonuria. Tetrahydrobiopterin is a cofactor for enzymes that participate in the synthesis of tyrosine, DOPA, serotonin, and nitric oxide. Under normal conditions, dopamine from the tuberoinfundibular system tonically inhibits the release of prolactin. Decreased BH4, causes decreased levels of dopamine, which therefore cause increased levels of prolactin.

A newborn is diagnosed with hyperphenylalaninemia during newborn screening. After extensive workup, the child is placed on a special phenylalanine-restricted diet with tyrosine supplementation and is instructed to return to his physician for regular follow-up visits. The parents are extensively counseled on the child's condition, including the necessary dietary restrictions that the child must adhere to. Several months later, laboratory tests indicate that the infant has a normal serum phenylalanine level; however, after careful examination, the physician observes some neurological abnormalities. Further workup includes a serum prolactin level, which is elevated. Which of the following enzymes is most likely deficient in this patient? (A) Phenylalanine hydroxylase (B) Dihydrobiopterin reductase (C) Dopamine hydroxylase (D) Homogentisate oxidase (E) Phenylethanolamine N-methyltransferase (F) Tyrosinase

*The answer is D.* This patient has the classic presentation of a carcinoid tumor. This type of tumor secretes serotonin that causes these classic symptoms. The breakdown product of serotonin is 5-hydroxyindoleacetic acid. Elevated levels of 5-HIAA in the urine confirms a high level of serotonin and the diagnosis of a carcinoid. VMA and/or catechols would be elevated if the patient had a pheochromocytoma producing epinephrine or norepinephrine (the VMAs are degradation products of these neurotransmitters. The symptoms do not match a pheochromocytoma, particularly due to the lack of increase in heart rate or blood pressure. Dopamine is depleted in Parkinson disease, not in this condition. Cortisol levels would be high in Cushing syndrome, but not under these conditions.

A patient presents with episodes of flushing, diarrhea, abdominal cramping, and wheezing. His blood pressure and pulse rate are normal during these episodes. Physical exam is normal except for scattered telangiectasias. In order to diagnose this problem, a 24-h urine collection for which of the following would be most appropriate? (A) Vanillylmandelic acid (VMAs) (B) Catechols (C) Dopamine (D) 5-hydroxyindoleacetic acid (5-HIAA) (E) Cortisol

*The answer is B.* The patient in this scenario has atypical phenylketonuria/tetrahydrobiopterin (BH4 ) deficiency. The treatment for phenylketonuria includes diet modification to restrict phenylalanine intake. Pharmacologic doses of tetrahydrobiopterin (BH4) may be an alternative or adjunct to dietary restriction of phenylalanine in patients with hyperphenylalaninemia or mild to moderate PKU phenotypes. Considering the vignette, phenylalanine is an essential and neutrally charged amino acid and thus fits the description of amino acid X. The majority of patients with elevated phenylalanine levels have classic phenylketonuria characterized by a mutation affecting phenylalanine hydroxylase, the first enzyme in the catecholamine biosynthetic pathway, shown in the diagram. However, approximately 2% of patients with elevated (classical) phenylalanine levels have a deficiency in BH4, an important coenzyme. This deficiency can be due to a variety of mutations in any one of several enzymes responsible for BH4 production or maintenance. BH4 deficiency prevents the first and second steps of the catecholamine biosynthetic pathway. It also interferes with the synthesis of serotonin from tryptophan and nitric oxide from arginine. With catecholamine biosynthesis interrupted, one should expect lower levels of dopamine, epinephrine, and norepinephrine in the patient's CSF. Thus, dopamine is the single best answer choice.

A newborn screening test returns with abnormally high levels of a particular amino acid, amino acid X (AAX), in the infants's blood. AAX is an essential and neutral amino acid (at physiologic pH). AAX also serves as the first step in an important biosynthetic pathway. Upon further testing, all enzymes within the pathway are normal in function and amount. However, the patient has lower levels of a particular coenzyme essential for pathway functioning. It is determined that the patient will be treated with dietary restriction of AAX and supplementation of the important coenzyme. What neurotransmitter would you expect to be low in a sample of this infant's CSF? A. γ-Aminobutyric acid B. Dopamine C. Histamine D. Phenylalanine E. Tyrosine

*The answer is C.* The patient in this vignette has homocystinuria, metabolic disorder that is inherited in an autosomal recessive manner and is included in most State newborn screens. In this disorder, a defect in the enzyme cystathionine beta-synthase results in elevated levels of the amino acid homocysteine. Homocysteine is converted to cystathionine via the enzyme cystathionine beta-synthase or it can regenerate methionine. Betaine decreases the concentration of homocysteine by promoting the conversion of homocysteine to methionine.

A one-month-old female infant is brought to your clinic by her parents. They recently moved from out of state and are here to establish care. You review the patient's medical records and interview the parents. The child was born via a planned Caesarean section for breech presentation to an otherwise healthy GIPO—1 mother. Prenatal laboratory testing was unremarkable. She received a Hepatitis B vaccination and a newborn screen was sent. The infant left the hospital with her mother after three days. Since birth, she has been exclusively breastfed and has been growing and gaining weight. Physical exam is unremarkable. You call the state lab to obtain the results of the newborn screen and are told that the infant is positive for a metabolic disorder. You inform the parents that their infant is at risk for ectopia lentis, thromboembolism, intellectual disability, seizures, osteoporosis, and anemia. The infant is started on a low-protein diet and treated with betaine (N, N, N-trimethylglycine). Which of the following substances is expected to increase following treatment with betaine? A. Citrullire B. Homocysteine C. Methionine D. Tyrosine E. Phenylalanine F. Orotic acid

*The answer is A.* The patient has von Gierke disease, a lack of glucose-6-phosphatase activity. When this individual tries to produce glucose for export in the liver, glucose-6-phosphate accumulates, which then goes through either glycolysis (generating lactate) or the HMP shunt pathway, producing excess ribose-5-phosphate. The excess ribose-5-phosphate is converted to PRPP, which then stimulates the amidophosphoribosyl transferase reaction (the rate-limiting step of purine production) to produce 5′-phosphoribosyl-1′-amine. This last reaction occurs because under normal cellular conditions, the concentrations of PRPP and glutamine are significantly below the Km values for amidophosphoribosyltransferase. Any cellular perturbation that increases PRPP levels, then, will increase the rate of the reaction, producing purines that are not required by the cell. This leads to degradation of the excess purines, producing urate and leading to gout. The lactic acidosis associated with von Gierke disease also blocks the transport of urate from the blood into the urine, which contributes to the elevated uric acid levels seen in these patients. Von Gierke disease does not lead to elevated glutamine, ATP, NADH, or dTTP levels.

A patient exhibits fasting hypoglycemia and lactic acidosis under fasting conditions. Hepatomegaly is also evident. A glucagon challenge only releases about 10% of the expected level of glucose from the liver. The patient has also developed gout due to an increase in the levels of which of the following metabolites? (A) PRPP (B) Glutamine (C) ATP (D) NADH (E) dTTP

*The answer is C.* . This patient has Parkinson disease, which is a problem with dopamine synthesis in the substantia nigra. Dopamine is derived from tyrosine. Treatment with DOPA in the initial stages of the disease provides relief from the symptoms. DOPA cannot be synthesized from alanine, serine, tryptophan, or phenylalanine.

A patient has a "pill rolling" tremor, "cogwheel" rigidity, bradykinesis, speech difficulties, and a shuffling gait. The chemical that is lacking in this syndrome is a derivative of which of the following amino acids? (A) Alanine (B) Serine (C) Tyrosine (D) Tryptophan (E) Phenylalanine

*The answer is B.* 5-fluorouracil is a thymine analog (thymine is 5-methyl uracil), which, after activation in the cells to F-dUMP, binds tightly to thymidylate synthase and blocks the enzyme from converting dUMP to dTMP. By blocking thymidine synthesis, cells can no longer synthesize DNA and will not replicate. 5-FU has no direct effect on dihydrofolate reductase, amidophosphoribosyl transferase, PRPP synthase, or UMP synthase. The fi gure also indicates the effect of methotrexate on dihydrofolate reductase.

A patient has been recently diagnosed with colorectal cancer. The physician treats the patient with a combination of chemotherapeutic drugs, one of which is 5-fluorouracil (5-FU). 5-FU is effective as an anticancer drug because it inhibits which one of the following enzymes? (A) Dihydrofolate reductase (B) Thymidylate synthase (C) Amidophosphoribosyl transferase (D) 5′-phosphoribosyl 1′-pyrophosphate (PRPP) synthetase (E) UMP synthase

*The answer is B.* Tyramine is a degradation product of tyrosine (decarboxylated tyrosine), which, when elevated, will lead to norepinephrine release. Tyramine is found in red wine and aged foods such as certain cheeses. When ingested, tyramine is degraded by monoamine oxidase to a harmless compound, and excessive norepinephrine release does not occur. However, if a patient is taking a monoamine oxidase inhibitor (MAOI), it is possible that tyramine does not get degraded appropriately. MAOIs which covalently modify (as opposed to being competitive inhibitors) the enzyme are very useful medications for atypical depression that is unresponsive to other modalities. Unfortunately, MAOIs have multiple interactions with many other medications and foods. A high tyramine level leads to a greatly elevated blood pressure due to the release of norepinephrine. Patients on MAOIs need to avoid foods high in tyramine, such as cheeses (aged and processed), red wine, caviar, brewer's yeast, miso soup, dried herring, and aged meats. MAOIs have no effect on glycoproteins or cholesterol.

A patient taking a drug for depression experienced a greatly increased heart rate and sweating after eating red wine and gourmet, aged cheese. These symptoms appeared due to an inability to degrade which of the following? (A) Tyrosine (B) Tyramine (C) Serotonin (D) Glycine (E) Glutamate

*The answer is B.* Acute intermittent porphyria is an autosomal dominant disorder resulting from a partial deficiency of porphobilinogen deaminase, the third enzyme in the heme biosynthetic pathway. Symptoms tend to occur in intermittent attacks and are vague overall. Symptoms are due to effects on the viscera', peripheral, autonomic, and central nervous systems, and include those described in this vignette as well as neuropathy. Abdominal pain is the most common symptom. The neurologic symptoms can be severe and life-threatening when they progress (come, seizures, paralysis). The diagnosis is made by demonstrating an elevated urine porphobilinogen. Treatment includes symptomatic management, carbohydrate loading (glucose reduces porphyrin precursor excretion), and intravenous hemin (helps reconstitute the heme pool).

A patient with porphyria initially presented repeatedly with abdominal pain, vomiting, constipation, and red/brown colored urine An extensive work-up was negative until urinary porphobilinogen was tested and found to be very elevated, confirming the diagnosis. The patient was treated with carbohydrate loading and intravenous hemin. Select the most likely diagnosis. A. Hereditary coproporphyria B. Acute intermittent porphyria C. ALA dehydratase porphyria D. Variegate porphyria E. Erythropoietic protoporphyria F. Porphyria cutanea tarda G. Congenital erythropoietic porphyria

*The answer is C.* Sulfonamides inhibit the synthesis of THF, a compound that eukaryotic cells cannot synthesize (which is why folic acid is a required vitamin in the human diet). Via inhibition of THF synthesis, the target prokaryotic cells can no longer synthesize dTMP and purines and are unable to grow and replicate. Sulfonamides do not affect DNA polymerases directly, nor do they alter mismatch repair. Sulfonamides also have no effect on ribonucleotide reductase.

A penicillin-allergic child was given a sulfonamide for otitis media. Human cells are resistant to sulfonamides due to which of the following? (A) Sulfonamides are specific for prokaryotic DNA polymerases (B) Sulfonamides are specific for prokaryotic RNA polymerases (C) Sulfonamides inhibit a metabolic pathway not present in eukaryotic cells (D) Sulfonamides inhibit bacterial ribonucleotide reductase, but not eukaryotic ribonucleotide reductase (E) Sulfonamides inhibit prokaryotic mismatch repair, but not eukaryotic mismatch repair

*The answer is E.* Only cytidylate (CMP) has a 5′-phosphate group, a ribose (pentose) group with a 2′-hydroxyl, and a pyrimidine base. Two purines [adenine (A) and guanine (G)] and two pyrimidines [cytosine (C) and thymine (T)] occur in DNA. In RNA, uracil (U) replaces thymine. Hypoxanthine and inosine are biosynthetic precursors to the bases in nucleic acids. Bases form nucleosides through bonding of a nitrogen with the C1 carbon (1′-carbon) of a pentose sugar. Adenosine, guanosine, and cytidine can form ribonucleotides with ribose (2′- and 3′-hydroxyl groups) or deoxyribonucleotides with deoxyribose (3′-hydroxyl group only). Uridine occurs only as the ribonucleoside, thymidine as the deoxyribonucleotide (actually as thymidylate deoxyribonucleotide synthesized from uridylate by thymidylate synthetase). Ribonucleotides (adenylate, guanidylate, cytidylate, uridylate) have a phosphate ester on the 5′-hydroxyl of ribose (note that the riboprefix is usually omitted). Deoxyribonucleotides have a phosphate ester on the 5′-hydroxyl of deoxyribose (deoxyadenylate, deoxyguanidylate, deoxycytidylate, and thymidylate). Thymidylate does not need the deoxy- prefix since it only forms as a deoxyribonucleotide. The phosphate ester may be formed of one, two, or three phosphate groups (e.g., AMP, ADP, or ATP).

A pentose with a 5′-phosphate group, a 2′-hydroxyl group, and a 1′-pyrimidine group describes which of the following structures? a. Cytosine b. Guanosine c. Thymidine d. Thymidylate e. Cytidylate

*The answer is C.* Pyridoxine (vitamin B,) is necessary for the transamination and decarboxylation of amino acids, for gluconeogenesis, and for other essential biochemical processes. Transamination reactions typically occur between an amino acid and an α-keto acid. The amino group is transferred to the α-keto acid from the amino acid, and 10 the Cr-keto acid thereby becomes an amino acid. For example, glutamate (amino acid) reacts with oxaloacetate (α-keto acid) to form aspartate (the resulting amino acid) and α-ketoglutarate (the resulting α-keto acid). Transamination (aminotransferases) are the enzymes that catalyze transamination reactions. and pyridoxal phosphate (active vitamin B,) serves as an essential cofactor for the transaminase.

A research scientist is studying biochemical reactions that take place In the liver. He cultures hepatocytes in a growth media enriched with glutamate labeled with nitrogen isotopes. After some time. he finds that the nitrogen isotopes are transferred to oxaloacetate, forming aspartate in the process. Which of the following substances is most likely involved in this reaction? (A) Biotin (B) Folic acid (C) Niacin (D) Pyridoxine (E) Riboflavin (F) Thiamine

*The answer is E.* The nitrogen in a purine ring is directly derived from glycine, glutamine, and aspartic acid. Glutamate, N5-formimino tetrahydrofolate, and asparagine do not directly donate nitrogen to the ring.

A researcher wants to develop a method of labeling purines with 15N for use in future spectroscopic studies. Purine synthesis will be done in a test tube using only the enzymes necessary to synthesize purines via the de novo pathway. Which starting materials should be labeled with the heavy nitrogen in order to maximize 15N incorporation into purines? (A) Aspartate, glycine, and glutamate (B) Aspartate, glycine, and N5-formimino tetrahydrofolate (C) Asparagine, glycine, and glutamine (D) Asparagine, glutamate, and glutamine (E) Aspartate, glycine, and glutamine

*The answer is D.* The older sibling of the infant in this vignette has classic features of phenylketonuria (PKLJ). Patients with PKU are often well at birth, but later on develop developmental delay and seizures. The disorder is also associated with albinism. Treatment involves phenylalanine-free and tyrosine-enriched infant formula. Phenylalanine is a starting substrate in the synthesis of catecholamines. The pathway is phenylalanine → tyrosine → L-dopa → dopamine → norepinephrine → epinephrine. In classical phenylketonuria, there is a deficiency of the enzyme phenylalanine hydroxylase, which is responsible for the first step in this pathway. As a result phenylalanine builds up to toxic levels and the following products in the pathway ere not synthesized.

A seven-day-old female infant presents to your clinic for a routine visit. She was born to a G2P1—2 mother via spontaneous vaginal delivery. Her pregnancy was uncomplicated and all prenatal testing was normal. She is exclusively breastfed and has been gaining weight appropriately Family history is notable for consanguinity and an older sibling with albinism, seizure disorder, and developmental delay. On physical exam, the infant is noted to have blond hair, blue eyes, and a fair complexion. Her newborn screen is positive for a metabolic disorder and she is started on a special formula. If the infant was not properly treated, production of which compound would be decreased? A. Thyroid-stimulating hormone (TSH) B. Serotonin C. Melatonin D. Dopamine E. Phenylalanine F. Glutamate

*The answer is B.* This patient is suffering from phenylketonuria, an autosomal recessive deficiency in the enzyme phenylalanine hydroxylase. Patients with PKU are often well at birth, but later on develop developmental delay and seizures. The disorder can also be associated with albinism. In the affected pathway, phenylalanine is normally converted to tyrosine, which then converted to L-dopa (3.4-Dihydroxyphenylalanine). Treatment involves a phenylalanine-free end tyrosine-enriched diet.

A six-month-old female infant is brought to your clinic for evaluation. The parents are worried because compared to her older sibling she seems delayed in her developmental milestones. The infant was born at home to an otherwise healthy G2P1—2 mother via spontaneous vaginal delivery. Since birth she has not had any other medical care. Her two-year-old sister is healthy. Family history is notable for developmental delay in a maternal cousin and developmental delay and seizure disorder in several relative on the father's side of the family. On physical exam, the infant cannot keep her head steady in a seated position, roll over, reach or grasp a toy. She is also noted to have a mousy odor. A newborn screen is sent to the state lab to confirm the suspected diagnosis and the infant is started on a formula to help treat her condition. In the affected pathway, what is the immediate product synthesized from tyrosine? A. Dopamine B. L-dopa C. Valine D. Leucine E. Norepinephrine F. Phenylalanine G. Epinephrine

*The answer is C.* The enzymatic reactions responsible for catecholamine synthesis begin with the conversion of tyrosine to L dihydroxyphenylalanine (dopa) by tyrosine hydroxylase. This is the rate limiting step in catecholamine synthesis. Dopa decarboxylase then converts dopa into dopamine. Dopamine beta-hydroxylase is an enzyme located within the vesicle membrane that acts on dopamine as it enters the vesicle to produce norepinephrine. Norepinephrine is stored in the vesicles and released via exocytosis upon demand. However, norepinephrine can diffuse into the cytoplasm, where it may be converted into epinephrine by the enzyme PNMT. Epinephrine may then return to the vesicle. diffuse from the cell or undergo catabolism. In periods of stress, the release of cortisol stimulates expression of the PNMT gene, thus increasing epinephrine synthesis The transfer of a methyl group to norepinephrine is catalyzed by PNMT and involves S-adenosyl methionine transferase (SAM) as the methyl donor. The animals in this experiment were unable to produce PNMT; therefore epinephrine production was inhibited Because PNMT is involved only in epinephrine synthesis, choices A, B, D, and E are incorrect.

A strain of Knockout mice in which the gene encoding phenylethanolamine-N-methyltransferase (PNMT) has been deleted is used to study catecholamine synthesis. The mice are stressed and adrenal glands are dissected and analyzed for catecholamine content. This analysis will most likely show the absence of which of the following in the adrenal glands? A. Dopa B. Dopamine C. Epinephrine D. Norepinephrine E. Tyrosine

*The answer is D.* Maple syrup urine disease is an autosomal recessive disorder caused by a defect in the enzyme α-ketoacid dehydrogenase, which is involved in the metabolism of the branched chain amino acids isoleucine, leucine, and valine. In this disease, degradation of these amino acids is blocked, leading to the accumulation of α-ketoacids in the blood and urine that cause sweet smelling urine, central nervous system defects, mental retardat ion, and death.

A variety of inherited biochemical enzymatic deficiencies present in infants and young children as central nervous system defects, mental retardation, and early death. Which genetic condition is caused by a defect in the enzyme α-ketoacid dehydrogenase? A. Alkaptonuria B. Homocystinuria C. Lesch-Nyhan syndrome D. Maple syrup urine disease E. Phenylketonuria

*The answer is A.* Pregnant woman with megaloblastic anemia and elevated serum homocysteine strongly suggests folate deficiency. Iron deficiency presents as microcytic, hypochromic anemia and would not elevate homocysteine. B12 deficiency is not most likely in this presentation.

A woman 7 months pregnant with her first child develops anemia. Laboratory evaluation indicates an increased mean cell volume (MVC), hypersegmented neutrophils, and altered morphology of several other cell types. The most likely underlying cause of this woman's anemia is A. folate deficiency B. iron deficiency C. glucose 6-phosphate dehydrogenase deficiency D. cyanocobalamin (B12) deficiency E. lead poisoning

*The answer is C.* The conversion of propionyl-CoA to methylmalonyl CoA requires biotin, and the conversion of methylmalonyl-CoA to succinyl-CoA requires vitamin B12. The boy is consuming large amounts of avidin, which binds to biotin and inhibits biotin-containing enzymes, such as propionyl-CoA carboxylase. The hypoglycemia results from the inhibition of pyruvate carboxylase, blocking a key enzyme necessary for gluconeogenesis.

A young boy has been eating five to six raw eggs a day in an attempt to be like Rocky, the fictitious prize fighter. The boy, however, becomes tired between meals, and during one of these lethargic periods, he is taken to the emergency department. His blood glucose levels were 50 mg/dL. Which one of the following would be expected to be elevated in this boy's blood? (A) Methylmalonic acid (B) Malonic acid (C) Propionic acid (D) Succinate (E) Acetic acid

*The answer is A.* This child's language regression and anemia are most likely due to lead poisoning. Lead toxicity is most prevalent among impoverished children residing in deteriorating urban housing built before 1978. Young children are particularly susceptible to lead poisoning via inhalation and ingestion of lead-based paint dust or chips due to normal crawling and mouthing behaviors. The incomplete blood-brain-barrier in children is vulnerable to the neurotoxic effects of lead, which include long-standing behavioral problems and developmental delay or regression. Anemia in lead poisoning results from inhibition of δ-aminolevulinic acid (ALA) synthase, ALA dehydratase, and ferrochelatase in the heme biosynthesis pathway. Because protoporphyrin IX cannot combine with iron (Fe) to form heme due to ferrochelatase inhibition, it instead incorporates a zinc ion, leading to elevated zinc protoporphyrin levels. In addition, ALA levels are increased. Lead poisoning also commonly coexists with iron deficiency anemia, and severe lead poisoning can also induce hemolytic anemia.

An 18-month-old boy is brought to the office due to language regression. He said several words at his 1-year appointment but now no longer speaks any words at all. His moods have also become more unpredictable over the past 4 months with frequent tantrums. The parents tried to bring him in sooner for evaluation, but they live in an impoverished part of the city and experienced financial difficulties with transportation to the office. On physical examination, the boy is quiet and maintains appropriate eye contact throughout the visit. Hemoglobin is 9 g/dL. Which of the following enzymes is most likely inhibited in this patient? (A) δ-Aminolevulinate dehydratase (B) Bilirubin glucuronyl transferase (C) Porphobilinogen deaminase (D) Pyruvate kinase (E) Uroporphyrinogen decarboxylase

*The answer is E.* The child has purine nucleoside phosphorylase deficiency, which, for reasons not yet fully elucidated, specifically reduces T-cell counts but not B-cells. Purine nucleoside phosphorylase is one of the salvage enzymes that converts guanosine or inosine to the free base plus ribose-1-phosphate (adenosine is not a substrate for this enzyme). HGPRT deficiency leads to Lesch-Nyhan syndrome, whose symptoms are quite different (there is no immune deficiency with an HGPRT defect). APRT deficiency leads to a buildup of an insoluble metabolite (2, 8-dihydroxyadenine) that precipitates in the kidney and will lead to renal failure. ADA deficiency will lead to an immune disorder, but in ADA deficiency, both B and T cells are deficient. An adenosine kinase deficiency has not been reported in humans.

An 18-month-old infant has had a history of recurrent bacterial and viral infections. The child has failure to thrive, developmental delay, and tremors. Physical exam shows a lack of peripheral lymphoid tissue. Blood work shows lymphopenia, but normal levels of B-cells and circulating immunoglobulins. This child most likely has a defect in which of the following enzymes? (A) Hypoxanthine guanine phosphoribosyltransferase (HGPRT) (B) Adenine phosphoribosyltransferase (APRT) (C) Adenosine deaminase (ADA) (D) Adenosine kinase (E) Purine nucleoside phosphorylase

*The answer is D.* With a purine nucleoside phosphorylase deficiency, guanosine will accumulate, which will inhibit the conversion of GMP to guanosine via the actions of 5′-nucleotidase (this is also true for dGMP). As dGMP accumulates, it will be phosphorylated to form dGTP. Concurrently, inosine will accumulate, blocking the conversion of adenosine to inosine and also leading to an increase in dATP levels. The combination of dATP and dGTP leads to inhibition of ribonucleotide reductase in the thymocytes, leading to T-cell depletion. It has also been reported that the accumulation of deoxyguanosine triggers apoptosis in T cells, providing another mechanism for T-cell depletion. This does not affect the B cells in this disorder. None of the other nucleotides listed (dCTP, dTTP, dIMP, and dUTP) will accumulate in this disorder.

An 18-month-old infant has had a history of recurrent bacterial and viral infections. The child has failure to thrive, developmental delay, and tremors. Physical exam shows a lack of peripheral lymphoid tissue. Blood work shows lymphopenia, but normal levels of B-cells and circulating immunoglobulins. Which one of the following metabolites would you expect to accumulate in the thymocytes? (A) dCTP (B) dTTP (C) dIMP (D) dGTP (E) dUTP

*The answer is C.* The infant described in the question Is most likely suffering from maple syrup urine disease, a disorder characterized by a defective breakdown of the branched chain amino acids leucine, isoleucine. and valine. The specific defect of maple syrup urine disease occurs in the enzyme branched-chain α-keto acid dehydrogenase. Because their degradation is inhibited at the α-keto acid stage, tissue and serum levels of these branched-chain α-keto acids increase, which leads to neurotoxicity. Maple syrup urine disease usually manifests within the first few days of life, and classically, the urine of affected infants has a distinctive sweet odor, much like burned caramel. Maple syrup urine disease can be life-threatening if left untreated, but dietary restriction of branched-chain amino acids. such as leucine, can lessen the severity of symptoms.

An infant born to a 23-year-old female develops vomiting and lethargy several days after birth. Physical examination reveals hypertonicity and muscle rigidity. The mother also notices an odor of burnt sugar in her diapers. Which of the following amino acids should most likely be restricted in this infants diet? (A) Phenylalanine (B) Tyrosine (C) Leucine (D) Methionine (E) Histidine (F) Lysine

*The answer is B.* The boy has cystinuria, elevated levels of cystine in the urine, due to a defect in a kidney transporter that removes cystine from the urine and sends it back into the blood. Due to this, the concentration of cystine in the urine is higher than normal and reaches levels close to its solubility limit. Cysteine is derived from methionine, so a reduction in methionine levels will reduce cysteine levels, which then leads to a reduction in cystine levels. Increasing ethanol content will lead to dehydration, which will increase the concentration of cystine in the urine, leading to increased precipitation. This would also be the case if the urine were acidified (acidification also reduces the solubility of the cystine stones). Restricting glycine is not effective, as glycine is not a precursor of cysteine biosynthesis. Prescribing diuretics would force the boy to urinate more frequently, and would raise a risk for dehydration, which would lead to possible elevation of cystine concentrations.

An 18-year-old boy was brought to the hospital by his mother due to a sudden onset of flank pain in his left side, radiating toward his pubic area. His urine was reddish-brown in color, and a urinalysis showed the presence of many red blood cells. When his urine was acidifed with acetic acid, clusters of flat, hexagonal transparent crystals were noted. A radiograph of the abdomen showed radio-opaque stones in both kidneys. The boy eventually passed a stone whose major component was identified as cystine. A suggestion for treatment is which of the following? (A) Increased ethanol consumption (B) Restriction of dietary methionine (C) Utilize drugs that acidify the urine (D) Restrict dietary glycine (E) Prescribe diuretics

*The answer is B.* Acute intermittent porphyria (Al P) is a blood disorder caused by a deficiency of porphobilinogen deaminase, an enzyme involved in the heme synthesis pathway. As a result, porphobilinogen accumulates in the body, leading to various physiologic disturbances. Patients often present with abdominal pain, polyneuropathy, red wine-colored urine, neuropsychiatric symptoms, and hyponatremia due to hypothalamic involvement and syndrome of inappropriate secretion of ADH. Barbiturates and other P-450 inducers can precipitate AlP attacks by promoting the activation of δ-aminolevulinic acid (ALA) synthase, causing an accumulation of downstream products in the heme synthesis pathway. ALA synthase is inhibited by glucose, so a lack of glucose caused by the patient's starvation diet contributed to the overactivity of ALA synt hase. This is why glucose and hemin (also an ALA synthase inhibitor) are given to patients with AIP attacks. Dieting leads to mobilization of fatty acids from triglyceride stores, thus lowering fat depots in the body. Since the accumulated intermediates from the AIP enzyme deficiency are hydrophobic, they can be stored in fat tissue. When a person undergoes an acute loss of fat tissue, the stored intermediates of heme biosynthesis are released and cause and acute onset, ie, Al P. Dark or reddish-brown urine is a common symptom of ALP that is due to the accumulation of porphyrins and porphyrin precursors that accumulate in the urine during episodes. An acute presentation of red urine is likely a sign of an ongoing AIP attack.

An 18-year-old woman presents to the emergency department with acute onset of severe abdominal pain. She says she had a similar attack 1-year earlier after taking some phenobarbital. At that time she underwent an exploratory laparotomy, which revealed nothing. The patient no longer takes barbiturates but recently started an extremely low-carbohydrate and low-calorie diet for weight loss. Her temperature is 37°C (98 .6°F), respiratory rate 16/min, and blood pressure 128/83 mm Hg. On physical examination, it is noted that, in addition to severe abdominal tenderness, the patient has numbness in her fingers and is actively hallucinating. Laboratory values are notable for serum Na+ 127 mEq/L. Her urine sample is dark red . What is the most likely additional laboratory finding? A. Abnormal hemoglobin forms on gel electrophoresis B. Increased urine porphobilinogen C. Increased urine uroporphyrinogen D. Ringed sideroblasts on peripheral smear E. Spherocytes on peripheral smear

*The answer is E.* In the synthesis of cysteine, the following sequence of steps occurs, where SAM is S-adenosylmethionine, CS is cystathionine synthase, cys is cysteine, and α-KG is α ketoglutarate: methionine → SAM → homocysteine + adenosine homocysteine + serine → cystathionine → cys + α-KG + NH4+ (CS, B6) Cystathionine synthetase, a pyridoxal phosphate (vitamin B6) enzyme, catalyzes the condensation of serine and homocysteine to form cystathionine. A deficiency of this enzyme leads to a buildup of homocysteine, which oxidizes to form homocystine. This may result in mental retardation, but sometimes causes dislocated lenses and a tall, asthenic build reminiscent of Marfan's syndrome. Patients with homocystinuria also have a clotting diathesis, requiring care to avoid dehydration during anesthesia. Their cysteine deficiency must be made up from dietary sources. In some cases, dietary intake of vitamin B6 (pyrixodal phosphate) may alleviate symptoms because of its requirement by the crucial enzymes.

An adolescent female develops hemiballismus (repetitive throwing motions of the arms) after anesthesia for a routine operation. She is tall and lanky, and it is noted that she and her sister both had previous operations for dislocated lenses of the eyes. The symptoms are suspicious for the disease homocystinuria. Which of the statements below is descriptive of this disease? a. Patients may be treated with dietary supplements of vitamin B12 b. Patients may be treated with dietary supplements of vitamin C c. There is deficient excretion of homocysteine d. There is increased excretion of cysteine e. There is a defect in the ability to form homocysteine from methionine by methylation

*The answer is D.* The patient has diabetes-induced nephropathy. As the kidneys lose function, the ability of creatinine to be absorbed into the urine decreases, and its excretion would be low. Creatine is synthesized from glycine, arginine, and SAM. In muscle, creatine is converted to creatine phosphate, which is nonenzymatically cyclized to form creatinine. The amount of creatinine excreted by the kidneys each day depends on the body muscle mass. Weight lifting and increasing muscle mass would increase the levels of creatinine in the urine. A low-protein diet would not reduce the muscle mass, nor affect the creatinine excretion.

An individual who has been treated for Type 2 diabetes for the past 24 years had, as part of his annual physical, a 24-hour urine collection. Reduced levels of creatinine were found, which is most likely due to which one of the following? (A) A decreased dietary intake of creatine (B) A higher-than-normal muscle mass resulting from weight lifting (C) A genetic defect in the enzyme that converts creatine phosphate to creatinine (D) Kidney failure (E) A lower-than-normal muscle mass due to a low-protein diet

*The answer is B.* Ammonia is generated from the metabolism of alpha-amino acids and is converted into urea through the hepatic urea cycle. Defects in any of the urea cycle steps, including the transport of ornithine from the cytosol into the mitochondria, resulting in disorders of the urea cycle. The common problem resulting from disorders of the urea cycle is increased blood concentration of ammonia, leading to central nervous system dysfunction. Severe defects manifest during early infancy and childhood, while milder defects may not manifest until adulthood. The treatment of urea cycle disorders consist of balancing dietary protein intake and protein output, such that the body receives the essential amino acids needed for growth and development but not in excess such that excessive ammonia is formed protein restriction is the main form of therapy for urea-cycle disorders. Medications that provide alternative pathways for the removal of ammonia from the blood can be combined with protein restriction.

An infant born to a 23-year old female is diagnosed with an inherited condition that results in impaired transport of ornithine from the cytosol to the mitochondria. Restriction of which of the following substances in the diet can improve this patient's condition? (A) Fatty acids (B) Protein (C) Cholesterol (D) Monosaccharides (E) Disaccharides (F) Purines

*The answer is A.* Ammonia is generated from the metabolism of alpha amino acids and is normally converted to urea by the urea cycle. The urea cycle involves five enzymatic steps: two in the mitochondrial matrix and three in the cytosol. One more enzyme that indirectly participates in the urea cycle is N-acetylglutamate synthetase (NAGS). The first step of urea cycle combines Co,, ammonia, and ATP to form carbamoyl phosphate in a reaction catalyzed by the enzyme carbamoyl phosphate synthetase I, the rate-limiting step in the urea cycle. Carbamoyl phosphate synthetase I (CPS) requires the presence of N-acetylglutamate (NAG), a molecule formed by NAGS, as this molecule acts as an allosteric activator of CPS. None of the other steps in the urea cycle require NAG as an activator. The symptoms seen in the patient in this question stem are the result of toxic effects of ammonia accumulation within the child's blood and tissues. The first few feedings provide a protein load to the infant that results in amino acids being available for metabolism, but the defect in the urea cycle prevents the disposal of toxic ammonia from the child's body, leading to lethargy, vomiting, and seizures.

An infant born to a 32-year old female demonstrates lethargy, vomiting, and seizures after her first few feedings. Initial laboratory evaluation demonstrates a markedly increased blood ammonium level. Liver biopsy suggests impaired formation of N-acetylglutamate as the cause of the child's problems. Which of the following reactions is most likely impaired in this patient?

*The answer is D.* Pyruvate dehydrogenase (PDH) is an allosteric enzyme that converts pyruvate into acetyl-CoA in the presence of oxygen (i.e. during aerobic metabolism). In the absence of oxygen or with a deficiency of PDH, pyruvate is alternatively converted to lactate by the enzyme lactate dehydrogenase. Excessive lactate production in these states results in lactic acidosis. PDH deficiency has a wide spectrum of presentations. Because carbohydrates may aggravate lactic acidosis, a ketogenic diet is recommended in these patients. Amino acid catabolism following removal of the amino group results in the formation of intermediates that are either glycogenic (producing intermediates of the citric acid cycle or pyruvate) or ketogenic (producing acetoacetate or its precursors). Some amino acids such as phenylalanine, isoleucine, and tryptophan are both glycogenic as well as ketogenic. Leucine and lysine (Choice D) are exclusively ketogenic and would not lead to increased formation of lactic acid. Lysine is an essential amino acid that is totally ketogenic.

An infant suffering from severe neurological symptoms and lactic acidosis is diagnosed with pyruvate dehydrogenase complex deficiency. When placed on a special diet, the patient demonstrates some decrease in the blood lactate level. Which of the following substances can be most safely supplemented to this patient? A. Glycerol B. Alanine C. Galactose D. Lysine E. Asparagine F. Serine

*The answer is C.* The patient has a defect in argininosuccinate synthetase. Citrulline, the substrate for the reaction, accumulates and can be measured in the blood. A carbamoyl phosphate synthetase I deficiency would block carbamoyl phosphate formation, and citrulline would neither be synthesized nor accumulated. An ornithine transcarbamoylase deficiency would lead to orotic acid accumulation (carbamoyl phosphate made in the mitochondria would diffuse into the cytoplasm, thereby activating pyrimidine synthesis and overproducing ortoic acid). An argininosuccinate lyase deficiency would lead to elevated argininosuccinate, which is not observed, and an arginase deficiency would lead to elevated arginine, which was also not observed. Defects in argininosuccinate lyase and arginase also do not have as elevated ammonia levels as do defects in previous enzymes of the cycle, since two nitrogens have been disposed of in the synthesis of argininosuccinate.

An infant who appeared normal at birth began to develop lethargy, hypothermia, and apnea within 24 hours. An analysis of blood components indicated high levels of ammonia and citrulline, and low levels of urea. The most likely defective enzyme in this child is which one of the following? (A) Carbamoyl phosphate synthetase I (B) Ornithine transcarbamoylase (C) Argininosuccinate synthetase (D) Argininosuccinate lyase (E) Arginase

*The answer is D.* Digestion and absorption of nutrients primarily occurs in the small intestine. Small intestinal epithelial cells produce several enzymes responsible for nutrient absorption. Proteins in ingested food exist primarily as polypeptides and require hydrolysis to dipeptides, tripeptides and amino acids for absorption. Hydrolysis of these polypeptides is accomplished by proteolytic enzymes such as pepsin and trypsin. These enzymes are secreted as the inactive proenzymes pepsinogen and trypsinogen from the stomach and pancreas, respectively. Trypsin, in tum, activates other proteolytic enzymes including chymotrypsin, carboxypeptidase and elastase. Activation of trypsinogen to trypsin is achieved by enteropeptidase (or enterokinase), an enzyme produced in the duodenum. Enteropeptidase deficiency results in defective conversion of the proenzyme trypsinogen to the active enzyme trypsin. Typical clinical manifestations of enteropeptidase deficiency are diarrhea, growth retardation and hypoproteinemia.

An infant who fails to gain weight has no enteropeptidase activity on the surface of her duodenal epithelium. Formation of which of the following substance is most likely impaired by this patients condition? (A) Lipase (B) Pepsin (C) Amylase (D) Trypsin (E) Lactase (F) Secretin

*The answer is C.* Based on the patient's skin discoloration, absence of other clinical signs and symptoms, and elevated direct bilirubin level, the picture is most consistent with hereditary conjugated hyperbilirubinemia, Moreover, given the asymptomatic conjugated hyperbilirubinemia, a seeming autosomal recessive mode of inheritance, and the characteristic brown-to-black discoloration of the patient's grandmother's liver, the most likely diagnosis is Dubin-Johnson syndrome. Because this syndrome is characterized by defective bilirubin transport, the correct answer would be impaired excretion of conjugated bilirubin. The impaired excretion results in a back-up of conjugated bilirubin, which then results in an increase in serum conjugated and total bilirubin levels.

An otherwise healthy 14-year-old white boy presents to his physician because of chronic yellow discoloration of his skin. His mother reports that everyone in his family has a normal complexion with the exception of his deceased maternal grandmother, who had similar skin color and was found to have brown-to-black discoloration of her liver at autopsy. Laboratory tests show his liver function is within the normal range, but direct and total bilirubin levels are increased. Which of the following is the most likely reason for this patient's skin discoloration? A. Decreased or absent uridine diphosphate-glucuronyl transferase activity B. Excessive iron absorption C. Impaired excretion of conjugated bilirubin D. Increased excretion of conjugated bilirubin

*The answer is B.* This patient has gout, characterized by painful joints due to the precipitation of sodium orate crystals caused by either underexcretion (majority) or excessive production of uric acid (minority). Acute gouty arthritis is seen most commonly in males. The crystals, identified as negatively birefringent and needle-shaped, initiate neutrophil-mediated and acute inflammation, often first affecting the big toe (although other joints such as the ankle, heel, and wrist can also be affected). Kidney disease is also seen due to accumulation of uric acid in the tubules. leading to repeated episodes of renal colic (severe flank pain radiating to the groin). Acute attacks are treated with colchicine or indomethacin to reduce the inflammation. Overproduction of uric acid and chronic gout are treated with allopurinol, which inhibits xanthine oxidase and also can reduce purine synthesis by inhibiting PRPP-amidotransferase. Xanthine oxidase catalyzes the sequential oxidation of hypoxanthine to xanthine to uric acid

An otherwise healthy 47-year-old man comes to the physician because of a swollen and extremely tender left ankle. A review of his records shows that he has had 4 episodes of severe flank pain radiating to the groin over the past 5 months. Polarized light microscopy examination of synovial fluid from the ankle shows negatively birefringent, needle-shaped crystals. Which of the following conditions is most likely in this patient? A. A defect in urea synthesis B. An abnormality of the purine salvage pathway C. An abnormality of the pyrimidine synthesis pathway D. Calcium pyrophosphate deposition disease E. Ethylene glycol poisoning

*The answer is D.* This patient has orotic aciduria and has a deficiency in UMP-synthase. This person is unable to produce UMP, which can be used to synthesize uridine.

An underweight, anemic 10-year old male has a crystalline substance-in-his-urine. A bone marrow aspirate revealed a large number of meqaloblastic cells. Serum levels of vitamin B12 and folic acid are normal and dietary supplementation with additional amounts of these vitamins produced no improvement in his anemia. Which of the following dietary supplements might logically be explored to improve this situation? A. Adenosine B. Inosine C. Sodium Benzoate D. Uridine E. Vitamin B6

*The answer is C.* This patient's synovial fluid analysis shows negatively birefringent crystals (ie, monosodium urate crystals) under polarized light, which is diagnostic for gouty arthritis. Neutrophils are the primary cells responsible for the intense inflammatory response seen in patients with gout. Phagocytosis of urate crystals by neutrophils causes the release of various cytokines and inflammatory mediators that lead to further neutrophil activation and chemotaxis, resulting in a positive feedback loop that amplifies the inflammatory response. Nor steroidal anti-inflammatory drugs (NSAIDs) are first-line therapy for treating acute gouty arthritis. They inhibit prostanoid biosynthesis (eg, prostaglandins, prostacyclin, thromboxanes), exerting a broad anti-inflammatory effect that includes inhibition of neutrophils. Patients with contraindications to NSAIDs (eg, peptic ulcer disease, renal impairment) are often treated with colchicine, which impairs neutrophil migration and phagocytosis by interfering with microtubule formation. Colchicine also decreases tyrosine phosphorylation in response to monosodium urate crystals, resulting in decreased neutrophil activation. *Educational Objective:* Non-steroidal anti-inflammatory drugs (NSAIDs) are first-line therapy for treating acute gouty arthritis. They inhibit cyclooxygenase and exert a broad anti-inflammatory effect that includes inhibition of neutrophils When NSAIDs are contraindicated (eg, peptic ulcer disease, renal impairment), colchicine is useful in the acute management of gout as it inhibits neutrophil chemotaxis and phagocytosis by preventing microtubule formation.

As part of a long-term cohort study, members of a large extended family undergo periodic analysis of multiple serum markers. Many male participants are found to have abnormal laboratory results despite no obvious signs of disease. Further analysis shows that these men have an X linked mutation affecting the phosphoribosyl pyrophosphate (PRPP) synthetase gene, resulting in greatly increased substrate conversion. Incidentally, one of the male patients followed in the study is hospitalized with right knee pain and swelling. A sample of his synovial fluid shows negatively birefringent crystals under polarized light microscopy. To achieve rapid improvement in this patient's symptoms, therapy should be directed toward inhibiting which of the following types of cells? A. Eosinophils B. Lymphocytes C. Neutrophils D. Synovial cells E. Mast cells

*The answer is C.* Gout is a disease caused by tissue deposition of monosodium urate crystals. Elevated uric acid levels are a known risk factor for gout and increased purine metabolism is one possible cause of hyperuricemia. Phosphoribosyl pyrophosphate (PRPP) synthetase is the enzyme responsible for the production of the activated ribose necessary for de novo synthesis of purine and pyrimidine nucleotides. The mutation described in the question stem will cause increased production of purines due to feed-forward activation of the purine synthesis pathway. As a result, more purine molecules will undergo degradation, resulting in hyperuricemia and an increased risk of gout. *Educational Objective:* Gout occurs with increased frequency in patients with activating mutations involving phosphoribosyl pyrophosphate synthetase due to increased production and degradation of purines.

As part of a long-term cohort study, members of a large extended family undergo periodic analysis of multiple serum markers. Many male participants are found to have abnormal laboratory results despite no obvious signs of disease. Further analysis shows that these men have an X linked mutation affecting the phosphoribosyl pyrophosphate (PRPP) synthetase gene, resulting in greatly increased substrate conversion. Which of the following organs is most likely to develop pathology secondary to this mutation? A. Aorta B. Head C. Joints D. Liver E. Pancreas

*The answer is E.* In the liver, the urea cycle converts excess NH4+ to a form amenable to excretion by the kidneys. Free NH4+ condenses with CO2 to form carbamoyl phosphate in a reaction catalyzed by carbamoyl phosphate synthetase. This is an energy-expensive, essentially irreversible reaction requiring two molecules of ATP. Carbamoyl phosphate (compound E in the urea cycle diagram) then combines with ornithine to produce citrulline in the first step of the urea cycle proper. The second nitrogen of urea is derived from the amino acid aspartate, which condenses with citrulline to form arginosuccinase in the second step of the cycle. This step is catalyzed by arginosuccinase synthetase and also requires a molecule of ATP.

In the urea cycle diagram below, which compound is derived from a condensation of CO2 and NH4+?

*The answer is E.* The target of allopurinol, the enzyme xanthine oxidase, catalyzes two reactions. The first is the conversion of hypoxanthine (which is produced during the degradation of adenine) to xanthine and the second is the conversion of xanthine (which is produced during the degradation of guanine) to uric acid. Thus, in the presence of allopurinol, hypoxanthine accumulates from the degradation of adenine and xanthine accumulates via the guanine degradative pathway. Both of these compounds are more soluble than urate, thus alleviating the major problem in gout.

Individuals with gout are given allopurinol for long-term management of the disease. In such individuals, which of the following bases would accumulate in the urine? (A) Urate and xanthine (B) Guanine and adenine (C) Hypoxanthine and guanine (D) Xanthine and guanine (E) Hypoxanthine and xanthine

*The answer is A.* Lack of the enzyme homogentisate oxidase causes the accumulation of homogentisic acid, a metabolite in the pathway of degradation of phenylalanine and tyrosine. Homogentisate, like tyrosine, contains a phenol group. It is excreted in the urine, where it oxidizes and is polymerized to a dark substance upon standing. Under normal conditions, phenylalanine is degraded to tyrosine, which is broken down through a series of steps to fumarate and acetoacetate. The dark pigment melanin is another end product of this pathway. Deficiency of homogentisate oxidase is called alkaptonuria (black urine), a mild disease discovered by Sir Archibald Garrod, the pioneer of biochemical genetics. Garrod's geneticist colleague, William Bateson, recognized that alkaptonuria, like nearly all enzyme deficiencies, exhibits autosomal recessive inheritance.

Parents bring in their 2-week-old child fearful that he has ingested a poison. They had delayed disposing one of the child's diapers, and noted a black discoloration where the urine had collected. Later, they realized that all of the child's diapers would turn black if stored as waste for a day or so. Knowing that phenol groups can complex to form colors, which amino acid pathways are implicated in this phenomenon? a. The phenylalanine, tyrosine, and homogentisate pathway b. The histidine pathway c. The leucine, isoleucine, and valine pathway d. The methionine and homocystine pathway e. The arginine and citrulline pathway (urea cycle)

*The answer is E.* The child has maple syrup urine disease, a defect in the branched-chain α-keto acid dehydrogenase step that utilizes all three branched-chain α-keto acids as substrates. The reaction catalyzed by this enzyme is an oxidative decarboxylation reaction, which requires the same five cofactors as do pyruvate and α-ketoglutarate dehydrogenase; thiamine, NAD+, FAD, lipoic acid, and coenzyme A. A subset of patients with this disorder has a mutation in the E1 subunit of the enzyme, which has reduced the affinity of the enzyme for vitamin B1. Increasing the concentration of B1 can, therefore, overcome the effects of the mutation and allow the enzyme to exhibit sufficient activity to reduce the buildup of the toxic metabolites. While niacin and riboflavin are required for the enzyme, the mutation in the enzyme is such that the affinity of these cofactors for the enzyme has not been altered. B12 and B6 are not required for this reaction.

Routine newborn screening identified a child with elevated levels of α-ketoacids of the branched-chain amino acids. A certain subset of such children will respond well to which of the following vitamin supplementation? (A) Niacin (B) Riboflavin (C) B12 (D) B6 (E) Thiamine

*The answer is A.* This patient has several hallmark symptoms of homocystinuria, including progressive intellectual disability, marfanoid habitus (tall and thin, elongated limbs, scoliosis), and subluxation of the ocular lens (ectopia lentis). Elevated homocysteine levels in blood and urine are diagnostic of homocystinuria. Homocystinuria is an inborn error of methionine metabolism. Homocysteine can combine with serine to form cystathionine; this reaction is catalyzed by the enzyme cystathionine synthase. The autosomal recessive mutation of this enzyme results in the accumulation of homocysteine and the many manifestations of homocystinuria. Infants with homocystinuria are normal at birth. Clinical manifestations during infancy are nonspecific and may include failure to thrive and developmental delay. Such children usually have fair complexions, blue eyes, and a peculiar flushing of the cheeks. Generalized osteoporosis is the main finding on radiography. Thromboembolic episodes involving both large and small vessels, especially those of the brain, are common and may occur at any age.

Parents bring their 10-year-old son to the clinic with concerns about his worsening vision. On physical examination, a downward dislocation of the lens is noted. The boy has long, thin extremities and generalized decreased muscle tone. The parents also mention that their son was developmentally delayed and demonstrates slight intellectual disability. The physician informs them that the boy is at increased risk for thromboembolic events. What is the most likely cause of the patient's condition? A. Cystathionine synthase deficiency B. Defect in collagen synthesis C. Mutation in the fibrillin gene D. S-Adenosylmethionine synthase deficiency E. Vitamin B12 deficiency

*The answer is B.* The boy is suffering from lead poisoning, which he obtained from eating the flaking paint chips. Lead inhibits the δ-aminolevulinic acid dehydratase step of heme synthesis, leading to reduced heme levels. In addition, the ferrochelatase step (in which iron is inserted into the newly synthesized heme ring) is also inhibited by lead. The reduced heme levels reduce the amount of functional hemoglobin synthesized, leading to the microcytic anemia observed in the child. Lead does not interfere with iron transport or inhibit part of the phosphatidylinositol cycle (lithium is the metal that does that). DNA synthesis is not impaired by lead, nor does lead inhibit gene expression of the globin chains. Cytochrome synthesis is also decreased and may contribute to the lethargy observed in the child.

Parents bring their 6-year-old son to the pediatrician due to the parents being concerned about "mental retardation." Blood work demonstrated a microcytic anemia and basophilic stippling. During the patient history, it became apparent that the boy often stayed with his grandparents, who owned a 150-year-old apartment. The boy admitted to eating paint chips from the radiators in the apartment. The boy's anemia is most likely the result of which one of the following? (A) Inhibition of iron transport (B) Reduction of heme synthesis (C) Inhibition of the phosphatidyl inositol cycle (D) Blockage of reticulocyte DNA synthesis (E) Inhibition of β-globin gene expression

*The answer is A.* The girl has the disorder alkaptonuria, which is a defect in homogentisic acid oxidase, part of the phenylalanine/tyrosine degradative pathway. Homogentisic acid accumulates, and is removed from the body in the urine. Upon contact with the atmosphere, homogentisic acid is oxidized, and turns black. The constant presence of homogentisic acid in the circulation can lead to slow, but steady, deposits in the spine and joints, leading to arthritis in early adulthood. Calcification of the coronary arteries may also occur in these patients.

Parents of a newborn baby girl were concerned when they saw black spots in her diaper after the child had urinated. At their next meeting with the pediatrician, they were told that the disorder is one that can lead to arthritis in the spine and large joints, and the child may have heart problems and a propensity for kidney stones. The child has inherited an inborn error in the metabolism of which one of the following amino acids? (A) Phenylalanine (B) Tryptophan (C) Proline (D) Methionine (E) Histidine

*The answer is E.* Patients with congenital albinism have problems producing melanin. This can be due to a deficiency in the tyrosine (precursor to melanin) transporters or a deficiency in the tyrosinase enzyme. Either way, patients will have generalized decreased pigmentation in the skin, eyes, and hair. Without aggressive photoprotection, most albinism patients will eventually develop skin cancer. The other major problems are ocular, as the lack of melanin causes poor development of the retinal pigment epithelium. The lack of pigment in the iris also causes problems. Nystagmus, strabismus, and impaired visual acuity

Patients with albinism appear white-pink (skin color), have white hair, and have nonpigmented or blue irises. In many cases, these individuals may have melanocytes, but lack melanin in their skin. What is the most useful advice to give to a guardian of a child diagnosed with albinism? (A) To avoid foods with lactose (B) To avoid foods with phenylalanine (C) To avoid strenuous activity (D) To give growth hormone to help the child grow to a normal height (E) To wear clothing and sunscreen that protect from the sun when outside

*The answer is B.* HPRT is required for activation of 6-mercaptopurine to its ribonucleotide and inhibition of purine synthesis. The other enzymes listed are not targets for this drug.

Resistance of neoplastic cells to the chemotherapeutic effect of 6- mercaptopurine would most likely involve loss or inactivation of a gene encoding A. thymidylate synthase B. hypoxanthine phosphoribosyltransferase C. purine nucleoside pyrophosphorylase D. orotic acid phosphoribosyltransferase E. adenosine deaminase

*The answer is B.* The child has nonclassical phenylketonuria (PKU). Classical PKU is due to a defect in phenylalanine hydroxylase, leading to accumulation of phenylalanine derivatives. These interfere with amino acid transport into the brain and can lead to cognitive disorders if not treated, usually, by a low-phenylalanine diet. However, in nonclassical PKU, the required cofactor for the phenylalanine hydroxylase reaction, tetrahydrobiopterin, is deficient. This will lead to similar biochemical symptoms (elevation of phenylalanine derivatives), but, in addition, the catecholamines (dopamine, epinephrine, and norepinephrine) and serotonin cannot be synthesized as those pathways require tetrahydrobiopterin. Giving 5-hydroxytryptophan bypasses the block in serotonin biosynthesis, and would have to be a supplement for these children along with dihydroxyphenylalanine (DOPA), which is the hydroxylated precursor for catecholamine biosynthesis. Providing tyrosine will not overcome the block in neurotransmitter biosynthesis. Providing phenylalanine just makes the problem worse. Neither melanin nor alanine will bypass the metabolic block of this disease.

Routine newborn screening identified a child with elevated levels of phenylpyruvate and phenyllactate in the blood. Despite treating the child with a restricted diet, evidence of developmental delay became apparent. Supplementation with which of the following would be beneficial to the child? (A) Tyrosinea (B) 5-hydroxytryptophan (C) Melanin (D) Phenylalanine (E) Alanine

*The answer is B.* The antioxidant activity of glutathione is dependent upon maintenance of its reduced state. The enzyme glutathione reductase transfers electrons from NADPH via FAD to oxidized glutathione. Oxidized glutathione is composed of two glutathione molecules held together by a disulfide bridge. Reduced glutathione is a tripeptide with a free sulfhydryl group. It is the presence of the free sulfhydryl group that is of importance to the antioxidant activity of glutathione. In red blood cells, the function of cysteine residues of hemoglobin and other proteins is maintained by the reducing power of glutathione.

The important reactive group of glutathione in its role as an antioxidant is a. Serine b. Sulfhydryl c. Tyrosine d. Acetyl coenzyme A (CoA) e. Carboxyl

*The answer is C.* The steps of the urea cycle are divided between the mitochondrial matrix and cytosol of liver cells in mammals. The formation of ammonia, its reaction with carbon dioxide to produce carbamoyl phosphate, and the conversion to citrulline occur in the matrix of mitochondria. Citrulline diffuses out of the mitochondria, and the next three steps of the cycle, which result in the formation of urea, all take place in the cytosol. Peroxisomes have single membranes, in contrast to the double membranes of mitochondria. They house catalase and enzymes for medium- to long-chain fatty acid oxidation.

The reactions of the urea cycle occur a. In the cytosol b. In the mitochondrial matrix c. In the mitochondrial matrix and the cytosol d. Only in lysosomes e. In peroxisomes

*The answer is E.* In the kidney double deamination of glutamine reaction, two ammonia molecules are produced for excretion. In the first step, glutamine is deaminated to glutamate in a monoxidative reaction catalyzed by glutaminase. Then, glutamate dehydrogenase oxidatively deaminates glutamate to α-ketoglutarate. Glutamate dehydrogenase can use either NAD+, as shown, or NADP+ in the reactions. This second reaction is also found in the liver, where α-amino groups originally transaminated from many amino acids are funneled to glutamate to free ammonium ion for ultimate conversion to urea.

The second and final enzymatic step in the reaction pathway shown is most correctly described as a. Amination b. Aminotransferase c. Transamination d. Amidation e. Oxidative deamination

*The answer is B.* Thyroxine is a derivative of tyrosine. It is formed by the iodination and joining of peptide-linked tyrosyl residues of thyroglobulin. Proteolysis of thyroglobulin yields thyroxine. Thyroxine is also called tetraiodothyronine, or T4, because of the four iodine atoms of the thyroid hormone.

The thyroid hormone thyroxine (T4) is derived from a. Threonine b. Tyrosine c. Thiamine d. Tryptophan e. Tyramine

*The answer is C.* urine turning black when left standing is a defining feature of alkaptonuria- In this disorder, a defect in the enzyme homosentisate dioxygenase results in a build-up of homogentisate. Normally tyrosine is degraded to fumarate, with homogentisate being an intermediate in this pathway. The most serious complication is a potential for joint disease later in life.

The urine sample from a patient turns black when allowed to stand. The patient has no known health issues, and a blood smear shows normal findings. What product is likely elevated in the patient's urine? A. Red blood cells B. Tyrosine C. Homogentisate D. Urobilinogen E. Leucine

*The answer is G.* In Hartnup disease, a defect in the transport process for neutral amino acids is most pronounced in intestinal and renal transport. Neutral aminoaciduria is observed as well as increased fecal excretion of indole derivations due to bacterial conversion of unabsorbed dietary tryptophan. Pellagra-like symptoms can be seen due to the lack of tryptophan for niacin biosynthesis. Hyperammonia can be caused by a variety of urea cycle defects including carbamoyl phosphate synthase I deficiency. The high levels of NH4+ in the blood lead to very high levels of glutamine synthesis that may well be responsible for the subsequent brain damage. Deficiency of liver phenylalanine hydroxylase causes phenylketonuria. Consequently, high levels of phenylalanine are not converted to tyrosine. Phenylalanine and its metabolites accumulate in blood, leading to mental retardation if infants are not placed on a phenylalanine-restricted diet. Urine darkens upon standing due to the high levels of homogentisate that result from of a deficiency in homogentisate oxidase in the disease alkaptonuria. This is a deficiency in the pathway of tyrosine breakdown.

Which clinical laboratory observation below is suggestive of Hartnup disease (neutral amino acid transport deficiency)? a. Burnt-sugar smell in urine b. High plasma phenylalanine levels c. Extremely high levels of citrulline in urine d. Elevation of glutamine in blood and urine e. Elevated plasma tyrosine and methionine levels f. Dark urine g. High fecal levels of tryptophan and indole derivatives

*The answer is E.* Aspartate is produced by AST and carbamoyl phosphate by CPS-I.

Which enzymes are responsible for producing the direct donors of nitrogen into the pathway producing urea? A. Arginase and argininosuccinate lyase B. Xanthine oxidase and guanine deaminase C. Glutamate dehydrogenase and glutaminase D. Argininosuccinate synthetase and ornithine transcarbamoylase D. Aspartate aminotransferase and carbamoyl phosphate synthetase

*The answer is C.* PRPP is involed in de novo synthesis of all nucleotides. In purines, PRPP + Gly + Asp + Gln makes IMP (inosine monophosphate), which is then converted into ribonucleotides. In pyrimidines, orotic acid + PRPP makes OMP, which is then converted to ribonucleotides. Glycine is required for purine synthesis but not pyrimidine synthesis.

Which molecule is required for de novo synthesis of both purines and pyrimidines? A. Glycine B. Inosine monophosphate C. 5-phosphoribosyl pyrophosphate (PRPP) D. Orotic acid E. Orotidine monophosphate

*The answer is E.* The activated form of glucose utilized for the synthesis of glycogen and galactose is UDP-glucose, which is formed from the reaction of glucose-1-phosphate and UTP. The vitamin lipoic acid is covalently bound to the ε-amino group of a lysine residue of the enzyme dihydrolipoyl transacetylase. The amide-linked lipolysine residue is known as lipoamide and is an activated carrier of acyl groups derived from the hydroxyethyl derivative of thiamine pyrophosphate. In this manner, lipoamide functions as one of the coenzymes in oxidative decarboxylation reactions. In reductive synthesis such as fatty acid synthesis, NADPH is the major electron donor. This may be contrasted to NADH, which is utilized for the generation of ATP via electron transport.

Which of the activated groups or units is most closely associated with uridine diphosphate (UDP)? a. Electrons b. Phosphoryl c. Acyl d. Aldehyde e. Glucose

*The answer is B.* Methionine is the only sulfur-containing amino acid of the group, and is a precursor for the sulfur containing amino acid cysteine. Threonine has an aliphatic side chain with a hydroxyl group, glutamine and lysine have amino group side chains, and alanine has a methyl side chain. Homocysteine is a sulfur-containing amino acid that is not found in proteins, being an intermediate in the formation of cysteine from the sulfur-containing amino acid methionine. Homocysteine and methionine are also components of the activated methyl cycle in which S-adenosylmethionine is regenerated. S-adenosylmethionine is one of the major donors of methyl groups. Methionine is an essential amino acid and must be derived from the diet.

Which of the following amino acids is a precursor to cysteine? a. Threonine b. Methionine c. Glutamine d. Lysine e. Alanine

*The answer is D.* 5′-phosphoribosyl-1-pyrophosphate (PRPP) donates the ribose phosphate unit of nucleotides and is absolutely required for the beginning of the synthesis of purines. In fact, the enzymes regulating the synthesis of PRPP and the subsequent synthesis of phosphoribosylamine from PRPP are all end product-inhibited by inosine monophosphate (IMP), adenosine monophosphate (AMP), and guanosine monophosphate (GMP), the products of this reaction pathway. Allopurinol, an analogue of hypoxanthine, is a drug used to correct gout. It accomplishes this by inhibiting the production of urate from hypoxanthine and in doing so undergoes suicide inhibition of xanthine oxidase. 5-fluorouracil is an analogue of thymidine that inhibits thymidylate synthetase and is used in cancer chemotherapy. Cytosine arabinoside (Ara C), an inhibitor of RNA synthesis, also takes advantage of rapid nucleic acid synthesis in cancer cells and is used in chemotherapy.

Which of the following compounds is a required substrate for purine biosynthesis? a. 5-methyl thymidine b. Ara C c. Ribose phosphate d. 5-phosphoribosylpyrophosphate (PRPP) e. 5-fluorouracil

*The answer is B.* The degradation of purines to urate can lead to gout when an elevated level of urate is present in serum, causing the precipitation of sodium urate crystals in joints. The excessive production of urate in many patients seems to be connected to a partial deficiency of hypoxanthine-guanine phosphoribosyl transferase (HGPRT). Allopurinol, an analogue of hypoxanthine, is a drug used to correct gout. It accomplishes this by inhibiting the production of urate from hypoxanthine and in doing so undergoes suicide inhibition of xanthine oxidase. Ribose phosphate and PRPP are required for purine synthesis. 5-fluorouracil (5-FU) and cytosine arabinoside (Ara C) are cancer chemotherapy agents, the former being an analogue of thymine that inhibits thymidylate synthetase and the latter an inhibitor of RNA synthesis.

Which of the following compounds is an analogue of hypoxanthine? a. Ara C b. Allopurinol c. Ribose phosphate d. 5-phosphoribosylpyrophosphate (PRPP) e. 5-FU

*The answer is E.* All the compounds listed are intermediates of the citric acid cycle. However, only fumarate is an intermediate of both the citric acid and urea cycles. It and arginine are produced from argininosuccinate. Once produced by the urea cycle, fumarate enters the citric acid cycle and is converted to malate and then oxidized to oxaloacetate. Depending upon the organism's needs, oxaloacetate can either enter gluconeogenesis or react with acetyl CoA to form citrate.

Which of the following compounds serves as a primary link between the citric acid cycle and the urea cycle? a. Malate b. Succinate c. Isocitrate d. Citrate e. Fumarate

*The answer is D.* Once bile is excreted into the gut, bilirubin diglucuronide is hydrolyzed and reduced by bacteria to form urobilinogen, which is colorless. Much of the urobilinogen of the stools is further oxidized by intestinal bacteria to stercobilin, which gives stools their characteristic brown color. Some urobilinogen is reabsorbed by the gut into the portal blood, transported to the kidney, and converted and excreted as urobilin, which gives urine its characteristic yellow color.

Which of the following porphyrins gives stools their characteristic brown color? a. Biliverdin b. Urobilinogen c. Heme d. Stercobilin e. Urobilin

*The answer is E.* In OTC deficiency, orotic acid accumulates because carbamoyl phosphate will accumulate and go through the pyrimidine synthesis pathway, which produces orotic acid.

Which of the following results from a deficiency of ornithine transcarbamoylase but not of carbamoyl phosphate synthetase? A. Cerebral edema B. Decreased BUN C. Hyperammonemia D. Increased blood glutamine E. Orotic aciduria

*The answer is D.* Carbamoyl phosphate (CAP) synthase I is found in mitochondrial matrix and is the first step in urea synthesis, condensing CO2 and NH4+. Hyperammonemia occurs when CAP is deficient. CAP synthase II forms CAP as the first step in pyrimidine synthesis. Its complete deficiency would probably be a lethal mutation. When its activity is decreased, purine catabolism to uric acid is decreased, decreasing the possibility of hyperuricemia. In contrast, gout, Lesch-Nyhan syndrome, high xanthine oxidase activity, and von Gierke's disease all lead to increased urate production and excretion.

Which of the following would rule out hyperuricemia in a patient? a. Lesch-Nyhan syndrome b. Gout c. Xanthine oxidase hyperactivity d. Carbamoyl phosphate synthase deficiency e. Purine overproduction secondary to Von Gierke's disease

*The answer is E.* In humans, tyrosine can be formed by the hydroxylation of phenylalanine. This reaction is catalyzed by the enzyme phenylalanine hydroxylase. A deficiency of phenylalanine hydroxylase results in the disease called phenylketonuria. In this disease, it is usually the accumulation of phenylalanine and its metabolites rather than the lack of tyrosine that is the cause of the severe mental retardation ultimately seen. Once formed, tyrosine is the precursor of many important signal molecules. Catalyzed by tyrosine hydroxylase, tyrosine is hydroxylated to form L dihydroxyphenylalanine (dopa), which in turn is decarboxylated to form dopamine in the presence of dopa decarboxylase. Then, norepinephrine and finally epinephrine are formed from dopamine. All of these are signal molecules to some degree. Dopa and inhibitors of dopa decarboxylase are used in the treatment of Parkinson's disease, a neurologic disorder. Norepinephrine is a transmitter at smooth-muscle junctions innervated by sympathetic nerve fibers. Epinephrine and dopamine are catecholamine transmitters synthesized in sympathetic nerve terminals and in the adrenal gland. Tyrosine is also the precursor of thyroxine, the major thyroid hormone, and melanin, a skin pigment.

Which of the metabolites below is a precursor of tyrosine? a. L-dihydroxyphenylalanine (dopa) b. Dopamine c. Norepinephrine d. Epinephrine e. Phenylalanine

*The answer is C.* Pyrimidines are first built as a free base (orotic acid) before being converted to a nucleotide (OMP), which is the opposite of that of purine synthesis, in which the purine is built upon ribose 5 phosphate to first produce the nucleotide IMP. FH4 does not provide carbons for pyrimidine synthesis (the precursors are carbamoyl phosphate, derived from glutamine and carbon dioxide, and aspartic acid). Carbamoyl phosphate synthetase II, which produces carbamoyl phosphate in the cytoplasm for pyrimidine synthesis, does not fix ammonia into the product, but rather uses glutamine as the nitrogen donor. Aspartate is required to build the pyrimidine ring.

Which one of the following best represents de novo pyrimidine synthesis in a liver cell?

*The answer is C.* Valine, isoleucine, and leucine (the branched-chain amino acids) are transaminated (which requires vitamin B6) and then oxidized by an α-keto acid dehydrogenase (that requires lipoic acid as well as vitamin B1 [thiamine], coenzyme A, FAD, and NAD1). Four of the carbons of valine and isoleucine are converted to succinyl-CoA, which is a glucogenic product. Isoleucine also produces acetyl-CoA, a ketogenic product. Leucine is converted to HMG-CoA, which is cleaved to acetoacetate and acetyl-CoA, and is strictly ketogenic. Branched-chain amino acid metabolism occurs primarily in the muscle, as muscle contains the highest levels of the transaminase and dehydrogenase. Tetrahydrobiopterin is required for ring hydroxylations, which is not applicable to the metabolism of the branched-chain amino acids, which do not contain ring structures.

Which one of the following best represents the events that occur during the metabolism of the branched-chain amino acids?

*The answer is C.* Glutaminase acts on glutamine to release ammonia, which enters the urine and serves as a buffer by forming NH4+. The kidney produces guanidinoacetate, which travels to the liver to produce creatine. The muscle and brain then utilize the creatine to produce creatine phosphate. The kidney takes up glutamine and releases serine and alanine into the blood. Most of the urea that is excreted by the kidney is produced in the liver; the kidney has a limited ability to produce urea.

Which one of the following best represents the kidney's utilization of amino acids?

*The answer is D.* Bilirubin is produced by the oxidation of heme after its iron is released; CO is produced in this reaction. Bilirubin diglucuronide, which contains two glucuronic acid (not glucose) residues, is excreted into the bile (not the urine) by the liver. The bilirubin is modified in the intestinal tract and eventually excreted in the feces.

Which one of the following best represents the metabolism of bilirubin?

*The answer is E.* Aspartate is a glucogenic amino acid that is also used to carry NH4+ into the urea cycle. Aspartate aminotransferase catalyzes the direct transamination of aspartate to oxaloacetate: aspartate + α-ketoglutarate → oxaloacetate + glutamate Oxaloacetate may either be utilized in the citric acid cycle or undergo gluconeogenesis. Argininosuccinate synthetase catalyzes the condensation of citrulline and aspartate to form argininosuccinate: citrulline + asparate + ATP → argininosuccinate + AMP + PPi In this manner, one of the two nitrogens of urea is introduced into the urea cycle.

Which one of the following can be converted to an intermediate of either the citric acid cycle or the urea cycle? a. Tyrosine b. Lysine c. Leucine d. Tryptophan e. Aspartate

*The answer is A.* During purine ring biosynthesis, the amino acid glycine is completely incorporated to provide C4, C5, and N7. Glutamine contributes N3 and N9, aspartate provides N1, and derivatives of tetrahydrofolate furnish C2 and C8. Carbon dioxide is the source of C6. In pyrimidine ring synthesis, C2 and N3 are derived from carbamoyl phosphate, while N1, C4, C5, and C6 come from aspartate.

Which one of the following contributes nitrogen atoms to both purine and pyrimidine rings? a. Aspartate b. Carbamoyl phosphate c. Carbon dioxide d. Glutamine e. Tetrahydrofolate

*The answer is B.* Two of the major hormones are derived from the amino acid tyrosine: the adrenal hormone epinephrine and the thyroid hormone thyroxine (tetraiodothyronine). Epinephrine is the catabolic antagonist of insulin, a polypeptide hormone, and is similar in action to glucagon, a liver-specific polypeptide hormone. Thyroxine is important in governing the basal metabolic rate.

Which one of the following hormones is derived most completely from tyrosine? a. Glucagon b. Thyroxine c. Insulin d. Prostaglandins e. Endorphins

*The answer is D.* Carbamoyl phosphate within the mitochondria is formed from NH4+, CO2, and ATP. Carbamoyl phosphate synthetase II catalyzes carbamoyl phosphate synthesis from glutamine for pyrimidine synthesis in the cytoplasm. Carbamoyl phosphate reacts with ornithine to form citrulline, which reacts with aspartate to form argininosuccinate. Fumarate is released from argininosuccinate, and arginine is formed. Urea is produced from the guanidinium group on the side chain of arginine, not from the amino group on the α-carbon. Ornithine is regenerated. N-Acetylglutamate is an allosteric activator of carbamoyl phosphate synthetase I. Ornithine transcarbamoylase is not a regulated enzyme in mammals, and in bacteria N-acetylglutamate is not an allosteric effector of ornithine transcarbamoylase.

Which one of the following occurs in the urea cycle? (A) Carbamoyl phosphate is derived directly from glutamine and CO2. (B) Ornithine reacts with aspartate to generate argininosuccinate. (C) The α-amino group of arginine forms one of the nitrogens of urea. (D) Ornithine directly reacts with carbamoyl phosphate to form citrulline. (E) N-acetylglutamate is a positive allosteric effector of ornithine transcarbamoylase.

*The answer is D.* Xanthine oxidase catalyzes the last two steps in the degradation of purines. Hypoxanthine is oxidized to xanthine, and xanthine is further oxidized to uric acid. Thus, xanthine is both product and substrate in this two-step reaction. In humans, uric acid is excreted via the urine. Allopurinol, an analogue of xanthine, is used in gout to block uric acid production and deposition of uric acid crystals in the kidneys and joints. It acts as a suicide inhibitor of xanthine oxidase after it is converted to alloxanthine. Guanine can also be a precursor of xanthine.

Which statement best describes xanthine? a. It is a direct precursor of guanine b. It covalently binds to allopurinol c. It is a substrate rather than a product of the enzyme xanthine oxidase d. It is oxidized to form uric acid e. It is oxidized to form hypoxanthine

*The answer is C.* Cystathionine β-synthase has a requirement of pyridoxal phosphate, and in about 50% of the cases of defective synthase enzymes, increasing the concentration of B6 can overcome the effects of the mutation on the enzyme. While a defect in methionine synthase will lead to elevated homocysteine, this enzyme requires B12, not B6. A defect in N5, N10 methylene tetrahydrofolate reductase will also lead to elevated homocysteine, but that enzyme has a requirement for NADH, not vitamin B6. A defect in cystathionase (another B6 requiring enzyme) will block the degradation of cystathionine, which will accumulate, but will not lead to significantly elevated homocysteine. S-adenosyl homocysteine hydrolase is the enzyme that converts S-adenosyl homocysteine to homocysteine and adenosine; lack of its activity will lead to a reduction, not an increase, in homocysteine levels.

You have an elderly patient with a history of heart attacks (MIs) and strokes (CVAs). Blood work indicates an elevated homocysteine level, which is reduced by the patient taking pharmacological doses of pyridoxamine. An enzyme that would benefit from such treatment in lowering homocysteine levels is which of the following? (A) Methionine synthase (B) N5, N10 methylene tetrahydrofolate reductase (C) Cystathionine β-synthase (D) Cystathionase (E) S-adenosyl homocysteine hydrolase

*The answer is A.* This person has gout. Gout is caused by uric acid crystallization into a joint and an intense inflammatory reaction to those crystals. The X-ray demonstrated soft-tissue swelling over the first metatarsophalangeal joint and typical gouty erosion. Uric acid is an insoluble breakdown product of purines (adenine, hypoxanthine, or guanine). Pyrimidines (thymine, uracil, and cytosine) breakdown to different water soluble products that do not crystallize. Ribose-5-phosphate is also degraded to very water-soluble products.

Your 56-year-old male patient presents with intense redness, heat, and pain over his right great toe at the metatarsophalangeal joint. Fluid from this joint shows bifringent crystals. An X-ray of the foot is shown below. This disease is caused by the degradation of an excessive amount of which of the following? (A) Adenine (B) Thymine (C) Uracil (D) Cytosine (E) Ribose-5-phosphate

*The answer is A.* Methotrexate acts by inhibiting dihydrofolate reductase such that THF cannot be formed (either from folate or dihydrofolate), and a functional folate deficiency results. The folate defi ciency then results in a macrocytic anemia due to the lack of DNA synthesis. Red cell precursors increase in mass but cannot divide due to the lack of precursors for DNA replication. As a result, larger than normal cells are released into the circulation, although the overall red cell number decreases, resulting in an anemia. Both thalassemia and spherocytosis lead to microcytic anemia. Vitamin B12 levels would not be affected, and the normal occult blood tests and colonoscopy indicate that there is no bleeding leading to the anemia.

Your 60-year-old female patient has psoriasis and has been treated with methotrexate for several years. She has no other medical problems and her preventive screenings, including fecal occult blood tests and colonoscopy, have all been normal. She has developed an anemia. Which of the following would you expect to find when working up her anemia? (A) A macrocytic anemia (B) A microcytic anemia (C) Thalassemia (D) Spherocytes (E) A low vitamin B12 level

*The answer is E.* Hydroxyurea, in addition to inducing γ-chain synthesis of hemoglobin, is also an inhibitor of ribonucleotide reductase. If ribonucleotide reductase is inhibited, the cells' ability to generate deoxyribonucleotides will be impaired, and DNA synthesis will be hindered. Since blood cells are regenerated at a rapid rate, they are one of the first cells affected by an inhibition of DNA synthesis, and the result is a decrease of blood cells in the patient. Of the answers listed, the synthesis of only dUMP requires the activity of ribonucleotide reductase. Hydroxyurea does not interfere with the synthesis of N5-methyltetrahydrofolate, 5′-phosphoribosyl 1′-amine, PRPP, or adenosylcobalamin.

Your patient has sickle cell disease and is being treated with hydroxyurea. After 2 weeks on the drug, you find greatly reduced levels of most blood cell types, and the patient is removed from the drug to allow his blood cell counts to stabilize. One potential reason for this side effect of hydroxyurea treatment is its ability to alter the synthesis of which of the following metabolites? (A) N5-methyltetrahydrofolate (B) 5′ phosphoribosyl 1′ amine (C) PRPP (D) Adenosylcobalamin (E) dUMP


Conjuntos de estudio relacionados

Human Nutrition Midterm Chapters 1-10

View Set

BIOL 151 - Chapter 8 SmartNotes 2.0

View Set

Structure and function of the skin

View Set

Life, Health, and Annuities License

View Set

Question: Which cryptographic solution in WPA2 replaces TKIP?

View Set

Psykoterapi bara PP:n typ bara copy past

View Set